【sin】高校生のための数学質問スレPart10【cos】

このエントリーをはてなブックマークに追加
1ポリゴンだいすき☆ ◆iMPWS/vD/.
夜、明日提出の宿題をやっているとき

(・∀・)やった!あと1問!



(゚Д゚)ポカーン
(゚Д゚)ハァ?ナニコノモンダイ?
ヽ(`Д´)ノウワァァン!!ワカンナイヨォ!!!

・・・てな時に、頼りになる質問スレです。

・長い分母分子を含む分数はきちんと括弧でくくりましょう。
  (× x+1/x+2 ;  ○((x+1)/(x+2)) )
・質問者は名前を騙られたくない場合、トリップを付けましょう。
  (トリップの付け方は自分で探すこと)
・質問者はあらゆる回答者に敬意を表しましょう。(荒らしはスルーでおながい)
2KingOfKingMathematician ◆H06dhKnt9A :04/07/21 16:29
■過去ログ
【sinθ】高校生のための数学質問スレ【cosθ】
http://science3.2ch.net/test/read.cgi/math/1067107835/
【sin】高校生のための数学質問スレPart2【cos】
http://science3.2ch.net/test/read.cgi/math/1073922555/
【sin】高校生のための数学質問スレPart3【cos】
http://science3.2ch.net/test/read.cgi/math/1078407825/
【sin】高校生のための数学質問スレPart4【cos】
http://science3.2ch.net/test/read.cgi/math/1081959097/
【sin】高校生のための数学質問スレPart5【cos】
http://science3.2ch.net/test/read.cgi/math/1083770090/
【sin】高校生のための数学質問スレPart6【cos】
http://science3.2ch.net/test/read.cgi/math/1085143978/
【sin】高校生のための数学質問スレPart7【cos】
http://science3.2ch.net/test/read.cgi/math/1086024283/
【sin】高校生のための数学質問スレPart8【cos】
http://science3.2ch.net/test/read.cgi/math/1087659852/
【sin】高校生のための数学質問スレPart9【cos】
http://science3.2ch.net/test/read.cgi/math/1088663754/
3132人目の素数さん:04/07/21 16:38
3
4132人目の素数さん:04/07/21 16:38
KingOfKingMathematician ◆H06dhKnt9A さんありがとう。
いつもいつも感謝してます。
毎日勉強が大変でしょうが応援してますYO!!
5KingOfKingMathematician ◆H06dhKnt9A :04/07/21 16:43
>>4
おまえも頑張れな。
6FeaturesOfTheGod ◆UdoWOLrsDM :04/07/21 18:59
Re:>2,5
お前誰だよ。
7132人目の素数さん:04/07/21 19:12
1/xとx/2の小数部分が等しくなるような正の数xをすべて挙げるという問題が全くわかりません。
教えて下さい〜
8132人目の素数さん:04/07/21 19:16
>>7
言い換えると、x/2 − 1/x が整数と云うこと。1 から順に当て嵌めて行くと
方針が見えてこよう。
9132人目の素数さん:04/07/21 19:34
>>8
なるほど〜
少し出来そうな気がしてきました。
ご教授ありがとうございます。
10KingOfKingMathematician ◆H06dhKnt9A :04/07/22 00:20
>>6
私は私だ。
11前スレの859:04/07/22 00:24
平方完成して、グラフも書けました!!たぶん合ってると思うのですが…
12KingOfKingMathematician ◆H06dhKnt9A :04/07/22 00:44
>>11
不安なら、繰り返し検算してください。
その方法は身体に覚え込ませなければなりません。
>>1 氏ね。ついでにQも氏ね、つーか最優先で市ね。
14132人目の素数さん:04/07/22 00:58
心配なんですが859の問題に合わせて図を書くと、三角形の中に四角形はいりますよね?
15132人目の素数さん:04/07/22 01:04

1辺が10-xの正方形入りますよね??xは人によって違うから図は決まってないですかね?
16132人目の素数さん:04/07/22 01:09
>>15
問題は、
△ABCにおいて、∠C=90゚、AC=BC=10とする。ABの中点をMとし、辺BC上に点P、辺CA上に点QをBP=CQとなるようにとる。
(1)BP=xとおくとき、△MPQの面積Sをxで表せ
(2)Sの最大値、最小値を求めよ

これに入る正方形の一辺は5以下じゃないのか?
17132人目の素数さん:04/07/22 01:15
なんで5以下なんですか?
18132人目の素数さん:04/07/22 01:22
直角2等辺三角形で、等辺の長さが10
言い換えれば、一辺の長さ10の正方形を対角線で切ってできる三角形
19132人目の素数さん:04/07/22 01:30
意味がよくわかりません…
20132人目の素数さん:04/07/22 01:31
>>19
図を描いて、一辺が5より大きな正方形が入ったら
またおいで。
21132人目の素数さん:04/07/22 01:43
思ったんですけど、PとQ を点Cに近づけたら、正方形はでてこないですよね。逆に点Cに遠ざけた場合も。だから、図は人によって違うのでしょうか?これ!といった図は存在しないのでしょうか?
22132人目の素数さん:04/07/22 01:53
>>21
正方形は出てきません。
正方形のこと言い出したのが859なのか?
わけわからんから、859は名前に859って書いてくれ
>21
PとQは同時にCに近づくことはない。同時にCから遠ざかることもない。
図は人それぞれ違う図を描くだろうが同じ計算になるはずだ。
Sが最大値をとる場合の図とか
Sが最小値をとる場合の図とかには、これ!っていう図はあるよ
だからそれをいきなり書ければいいんだけどさ、書けないじゃん。わかんないし。

だから、xを適当な長さにしてみて書いてみる、と分かりやすくなる、のが(1)ね

(2)は、図をほっとけ。忘れろ。a(x-p)^2+qまでたどり着いたんだよね?
最小値とか最大値とかはグラフ。…を使わなくてもいいから、代入してミレ
0≦x≦10さえきちっとやれば、自ずとわかると思いますよ

と、前スレのまとめ
って俺質問しに着たんじゃなかったっけ・・・マアイイヤ
正方形は、出てくるっちゃー出てくるような
25FeaturesOfTheGod ◆UdoWOLrsDM :04/07/22 08:47
Re:>13 お前が先に氏ね。
>>25
>>13は真だ。さあ、お前の番だ。
27FeaturesOfTheGod ◆UdoWOLrsDM :04/07/22 08:53
Re:>26 お前が先に氏ね。
28Philanthrope (元 KingOfMathKingdom) ◆NlBVr1vWAA :04/07/22 08:54
From Philanthrope:


      Could it be possible, I cogitate myself: I love humanity!

Imagination and creation do I adhere: only out of his imaginative magnanimity
     are they to be created -- music, art and imperative ideas!
His profundity do I place my hope within; I shan't degenerate myself into the
               flux of sensualism.

            "Philanthrope," do I call myself.
Named anew is a pointer to myself; I shall be a lover of mankind thereafter!

29132人目の関数さん:04/07/22 09:36
適当な整数pが存在して、y≦p^2-(2p-5/4)^2が成り立つようなyの値を求めよ。

まったく持って分かりません。。。
30859:04/07/22 09:40
結局は(1)の問題の図を書こうとすると、三角形の中に正方形(1辺はいくつになるんでしょうか…)は出てくるんですか?みんなの言うことが違うのでよくわかりません
31132人目の素数さん:04/07/22 10:46
以下の問題がわかりません。よろしくお願いします。

x^2+y^2+Z^2=1 が成り立つ時、
x*y^2*z^3の最大値を求めよ。
32132人目の素数さん:04/07/22 12:05
>>31

大学入試の問題?
$x = ¥cos a , y = ¥sin a ¥cos b , z = ¥sin a ¥sin b $ と変数変換する
と、$x y^2 z^3 = ¥cos a ¥sin^5 a ¥cos^2 b ¥sin^3 b$ とでもすれば、あ
とは、2変数関数の偏微分と三角関数の公式でなんとかなりそう。

もう一つの解法は、$(x y^2 z^3)^2 = x^2 y^4 z^6$ として、$X = x^2 ,
Y = y^2 , Z = z^2$ と変換して、条件
$X + Y + Z = 1 (0 < X , Y , Z < 1)$ の下で、$X Y^2 Z^3$ の最大値を
求める方法。これは、$X = 1 - Y - Z $ を$X y^2 Z^3 $ に代入するだけ
なのでこっちのほうが良いかもわからん。

どのみち、答えは、$|x| = 1/¥sqrt(6) , |y| = 1/¥sqrt{3} ,
|z| = 1/¥sqrt{2}$ のとき、最大値 $¥sqrt{3}/36$
3331:04/07/22 12:15
>>32
ありがとうございます。
ただ、>>31が文字化けしているようなのですが、これはこちら側のミスでしょうか・・・
あるいは特殊な文字や記号がこちらで上手く表示できなかったのでしょうか。
>>32
TeX なのは構わんが、何故うにコードの円記号なのだ?
そのままでは TeX に通らんぞ。 Safari か?
35FeaturesOfTheGod ◆UdoWOLrsDM :04/07/22 12:56
&#92; \
&#165 ¥
36FeaturesOfTheGod ◆UdoWOLrsDM :04/07/22 12:57
&#92; \
&#165; ¥
374:04/07/22 15:02
>>2,5
あれっ、KingMathematicianさんじゃないんですか?
すみません、>>4の発言撤回させてください。
まちがえました。
38132人目の素数さん:04/07/22 18:10
0 ≦ θ < 2π の範囲で、式を満たすθの範囲
2*(sinθ)^2 > 1

これで、 sinθ > ±(1/√2)
ってするのはだめなんですか? これじゃ答えと一致しないんですが
>>38
二次不等式 x^2 < 1 を解いてご覧よ。
40132人目の素数さん:04/07/22 18:16

理解できました
m(_ _)m
点(-1、4)から円 x~2 +y~2-8x-2y+8=0に引いた接線を求めよ の問題で
x軸方向に-4、y軸方向に-1平行移動して接線を求めたんですが、
もっとスマートなやり方はありませんか?教えてください
42132人目の素数さん:04/07/22 23:14
放物線y=x^2と円x^2+(y-2)^2=r^2(r>0)がある。
四個の交点を持つrの値の範囲を求めよ

この問題で放物線と円の方程式の二つを使ってxを消去
そしてyだけの式(1)y^2-3y+4-r^2=0にして判別式D>0はわかるのですが
式(1)で y=0のとき4-r^2>0より大きいという条件は何を表すか分かりません。


44132人目の素数さん:04/07/22 23:30
>>41
 x^2+y^2-8x-2y+8=0
⇔(x-4)^2-16+(y-1)^2-1+8=0
⇔(x-4)^2+(y-1)^2=3^2

あとは、x軸方向に1、y軸方向に-4平行移動して
原点を通り点(5, -3)との距離が3となる直線を求めてから
x軸方向に-1、y軸方向に4平行移動する、くらいかな?
45132人目の素数さん:04/07/23 02:00
行列の積の定義についての質問ですが
A=M[a,b]M[c,d],B=M[p,r]M[q,s]であるとき
AB=C=M[ap+bq,ar+bs]M[cp+dq,cr+ds]となっていますが、これはAの行ベクトル
とBの列ベクトルから積の要素を考えたものですよね。では、Aの列ベクトル
とBの行ベクトルから考えた場合はダメなのでしょうか?つまり、
C=M[pa+rc,pb+rd]M[qa+sc,qb+sd]とはならないのでしょうか?
A*B=B*Aとはならないのでしょうか?
46132人目の素数さん:04/07/23 02:07
>>43
受験板の方に書くのは別にいいだろ。いろんな解法が知りたいのかもしれないし。
この問題には当てはまらないかもしれないが、問題によっては様々な解法
が考えられることもある。俺もそれ見ると勉強になることも多いし。
47132人目の素数さん:04/07/23 02:25
>>46
いや、受験板もここも両方回答してる回答者も多いよ。
そんな何度も何度も見せるのやめろってば。
>>46
じゃ、マルチポストはお前が全部答えてやればいい。
俺達はスルーしておく。
49132人目の素数さん:04/07/23 02:28
そうだな。
自分で解くのが一番の勉強だな。
5046:04/07/23 02:56
>>47
それは確かにそうだなあ・・・。まあ問題が面白くて、ためになると思ったら
漏れは答えるけど・・・。なんとも。
51132人目の素数さん:04/07/23 03:16
>>45
Aのi行目と,Bのj列目の積をij成分とする行列を,普通はABと書きます(定義します).
この定義によると,BAはBのi行目と,Aのj列目の積をij成分とする行列になります.

キミの様に行列の積を定義しても全く構いませんが,そのときは
AB=Bのi行目と,Aのj列目の積をij成分とする行列
BA=Aのi行目と,Bのj列目の積をij成分とする行列
となるはずで,それは最初に定義したAB,BAという行列を
それぞれBA,ABと書いたに過ぎません.

出て来るものに本質的な違いが無いのなら,
どちらの定義にしたがってもよいはずですが
もちろん,どちらかに統一(どの数学の本でも)しておいたほうが便利に決まっています.
前者の定義が採用されているのは慣習的なものです.
5245:04/07/23 04:08
>>51
丁寧な解説ありがとう。分かりやすかったです。
>>41
求める接線は
a(x+1)+b(y-4)=0とおける。
で、>>44のように円の方程式を変形すると
中心及び半径がわかる。

後は点と直線の距離の公式にぶち込む、と。

ちなみに、接線の方程式を
y=ax+bとか
y-4=m(x+1)とか…とおいた場合
x=kの形の検討が別に必要となる。

また、上記第2式から
yを円の方程式に代入して
xの2次方程式で判別式=0を使う手もあるが
計算がやや煩雑となろう。
>>41
スマートかどうかわからんが
接点を(p,q)とおくと接線は (p-4)(x-4)+(q-1)(y-1)=25とおける
これが点(-1、4)を通るので代入すると -5(p-4)+3(q-1)=25
つまり、直線 -5(x-4)+3(y-1)=25 は接点を通る直線
で、この直線と円の交点をだして接線の式に代入
てな方法もある
>44,53,54
 ありがとうございました
x-1/x=1(x>0) のとき x+1/x の値を求めよ  がわかりません。お願いします。
>>56
条件を二次方程式に直して、解の公式でxを求めて、
求める式をゴリゴリ計算すべし。
若干スマートな方法もあるが、見違えるほどでもないし。
58FeaturesOfTheGod ◆UdoWOLrsDM :04/07/23 10:57
Re:>56 x=(1+√(5))/2を使ってすぐに行けるかな?
59132人目の素数さん:04/07/23 11:18
>>56
両辺を自乗汁。
60132人目の素数さん:04/07/23 11:42
条件aを満たす関数を見つけたい、(ア)〜(エ)を埋めよ。
f(x)=xのときf(a)=(ア) f(b)=(イ)
f(a)+f(b)=(ウ) f(a+b)=(エ)

の答え教えてください。
61132人目の素数さん:04/07/23 11:54
>>60
f(x)=xということは、小学校風に書けばf(□)=□
6230:04/07/23 14:57
>>30
について誰かお願いします…
>62
前スレが参照し難いから断りを入れて、もう一度書いたら良いよ。問題を精確にね。
64132人目の素数さん:04/07/23 15:38
>>63
問題は >16
この問題自体には正方形は関係ない。
>14 が突然正方形の事を聞いたのでレスが付いただけ
>>62
話がそれすぎて混乱しているっぽいけど、
とりあえずこの方針で進めてみて。

前スレ900から引用
>>895
多分859だと思うけど、
次に(1)の三角形の面積を求める。
問題文から図が書ければ、ΔMPQの面積をxで表すことが出来る。
直接求めるのは難しいから、
ΔMPQ=ΔABC−ΔAQM−ΔPCQ−ΔBPM
を計算すると、ΔMPQがxの式で表される。
66132人目の素数さん:04/07/23 16:07
(1)の答えは出たんですが、問題の図が書けません…
>>66
インターネットで聞くより、紙と鉛筆を持ってリアルで友達に聞け。
これはマジレス。
68132人目の素数さん:04/07/23 16:12
>>67
友達がいない場合は?
69132人目の素数さん:04/07/23 16:30
友達もわかんないって言うんですよ…
まあ類は友を呼ぶっていうしな
71132人目の素数さん:04/07/23 17:39
下の3台のとき方教えてください

∫(0〜π)dx/(1+acosx) (0<a<1)

∫(0〜π)log(1+acosx)dx (lal<1)

∫(0〜2π)dθ/(1−2αcosθ+α^2)  (0<α<1)
72king:04/07/23 17:54
>>71
このスレは高校生ですよ
73132人目の素数さん:04/07/23 18:17
>>66
>>69
図がかけてないとすると、(1)の答えも怪しい。
問題よく見ればわかるけど、点Mは中点だけど、点Pも点Qも中点とは書いてない。
適当にずらして図を書かないと見えてこない。ずらして書けば、正方形は見えてこない。
ちなみにずらして書けば分かるけど、ΔAQMとΔBPMをたした面積はいつでも一定になるはず。そうすれば、ΔPCQの面積によって、ΔMPQの面積が増減するのはわかる。
y=x^3-xとy=(x-a)^3-(x-a)が共有点を持つような正の実数aの範囲はどうやって求めればいいのでしょうか?
>>74
x 、y についての連立方程式と見て解の存在する範囲を求めれば良い。
>>75
元が三次関数でも三次の項が消えたときは二次方程式の解の判別式が使えますか?
>>76
ちゃんと「二次」方程式である限りは使える。
>76
まずやる。検算する。グラフを描く。納得できるまで色々やって見ろ。
なるほど!
ご教授どうもです。
>>73
質問者ではないが、正方形云々がなんとなく分かってきた。

直線CMの延長上にCM=MDとなるように点Dを取ると□ACBDは正方形になる。
BD上に点R、DA上に点SをBP=CQ=DR=ASとなるように取ると、
対称性から□PQSRは正方形になり□PQSR=4△MPQ
以下略って辺りの話かな?

ただ、こういう発想は「頭の良い中学生」の発想であって、
「並の高校生」の発想ではないと思う。
少々発想力が弱くても力押しでゴリゴリ答が出せるのが
代数とか座標とか中学高校で習う数学のありがたみだし、
このスレ向きでは無いかもしれないと思う。
確かに面白いけどね。
81HBB:04/07/23 22:30
限界きたんでお願いします。

p,q,a,bは定数でp>0,q>0とする。XとYがX>=Yを満たしながら
動くとき、             
   
-pX^2 -qY^2 +aX+bY
の最大値、およびそのときのX、Yを求めよ。

XとYについてへーほーかんせーしてみてそれぞれの軸の大小関係について場合分け
するらしいのですが。-p(X-a/2p)^2+a^2/4p−q(Y-b/2q)^2+b^2/4qみたいな感じで。
a/2p>=b/2q →a/p>b/qの時はX>=Yより、
X=a/2p Y=b/2q 最大値a^2/4p+b^2/4qで相違ないと思うのですが
a/2p<b/2qの時がいまいちよく分かりません。
82132人目の素数さん:04/07/23 22:44
f(x^2)-1=x^3{f(x+1)-2}を満足する多項式f(x)を求めよ。
何ですが、問題からして意味が分かりません。どなたかよろしくお願いします。
>>82
まずは、fがn次式であるとして、
両辺がそれぞれ何次式になるか考える。
多項式として等しくなるのだから、両者が等しいとおく。
8482:04/07/23 22:52
>>83 すみません。もっと詳しく明快に書いていただけないでしょうか?
当方高一で、今日いきなり宿題で出されたんです。。。
>>84
めっちゃ明快やんけ。
>>84
夏休みの宿題ってことはだ、じっくり参考書読んで自学自習せよってことじゃないのけ?
8782:04/07/23 22:56
>>86補習の宿題で、僕たちに夏休みなどないです。 あるのは8月末に4日だけ。。。
88HBB:04/07/23 22:57
81もお願いします・・・
>>88
よくわからんが X=Y=b/2q とかまではいけるんじゃないか?
>>87
補習なんやったらイキナリっつーこたないわけやな。
>>81
X 、Y の一方について頂点を選べないなら頂点に一番近い所を探る。
92HBB:04/07/23 23:07
>>89
いや違います。
X=Y=a+b/2(p+q)の時なんです。この辺の事も教えてくれたら幸いっす。
93HBB:04/07/23 23:08
間違った。
X=Y=(a+b)/2(p+q)でした。

>>91
探るとはどんな感じにすりゃいいんですかね・・・
94132人目の素数さん:04/07/23 23:34
空間ベクトルの問題で、次の3点が一直線上にあるように定数a,bの値を求めよ。
という問題の解き方教えてくださーい(^_-)
95132人目の素数さん:04/07/23 23:35
>>94
定数って・・・なんの。
96132人目の素数さん:04/07/23 23:45
>>95
空間ベクトルの問題で、次の3点が一直線上にあるように定数a,bの値を求めよ。
A(−3,2,−1),B(2,−5,3),C(a,b,−5)
書き忘れてました(^_^;)
(^_-)
(^_-)
(^_-)
>>94

AとBを通る直線の式を求める。そんでその式を満たすようにa,bを決める。
9991:04/07/23 23:50
>>93
X ≧ Y 、a/2p<b/2q だから、X=a/2p を採った時、Y ≦ X = a/2p
なので、頂点 b/2q に一番近い Y = a/2p を採った時が最大。
先に Y= b/2q を採った時は、X= b/2q を採った時が最大。
つまり、X=Y の直線上で最大となるから、その条件下で考える。
つまり、X=Y とおいて、一変数の場合の最大値問題に帰着する。
100132人目の素数さん:04/07/23 23:53
>>96
・・・A、Bの座標から直線の式出てくるからなぁ・・・。
でも3変数座標系で直線の方程式出しても、Cの座標は二つ分からないわけだから解けないね。

ということは、2変数座標系で直線の方程式出せば出るんじゃないかな?
>>96
>>98より簡単に。

AB↑=kAC↑とおけるので
先にkが求まって、次いでa,bもわかる。
102132人目の素数さん:04/07/24 00:06
>>101
それじゃきっと>>96さんは原理が分からないですよ。
基礎問題なんだから最初は地道にやるべきですよ。
103132人目の素数さん:04/07/24 00:16
96ですけどやっぱだめです。どうしても解けません。
>>103

ABを通る直線の式は出たか?どこまでは分かったか書いてくれ。
105132人目の素数さん:04/07/24 00:19
>>104
ABをとおる直線式さえでてません。馬鹿なのですんません
106101:04/07/24 00:21
>>102
いや、まあこれは空間ベクトルの問題だから
平面ベクトルにおける類題は既習だと思うんだが。
>>96はまず平面ベクトルを復習した方がいいのかな。
>>105

そしたら、「空間の直線の式」とかそのあたり復習してからその問題に
取り掛かると良いよ。ここで答えだけ教えてもらっても、次の問題で
また躓くし。
108132人目の素数さん:04/07/24 00:25
>>106
テスト間近なのに結構わかってません。かなりピンチです。土日は、猛勉強します
109132人目の素数さん:04/07/24 00:27
>>107
わかりました。もう1度復習して、もしわからなかったら後日お願いいたします<(_ _)>
>>105
一般に直線がどの様な式で表現できるかを調べるべし。
a+bt
112132人目の素数さん:04/07/24 09:50
数Uの問題です

直線y=ax(a>0)と直交する直線l(L)が点(2.0)を通るとき、y=axとl(L)との交点Pの座標を求めよ。
また、原点Oと点Pとの距離をaで表せ。

上記の問題をお願いします
>>112
おまえなー

直交する ⇔ 二直線の傾きの積は-1

基本だろうが
114132人目の素数さん:04/07/24 13:49
>>112
y=ax, (y-0)=□*(x-2)
の連立方程式。□に何が入るかは
>>113
のを参考に自分で考える。
長さは教科書を読む。
115132人目の素数さん:04/07/24 15:19
場合の数の正しい読み方は、「ばあいのかず」もしくは「ばあいのすう」の
どちらでしょうか?基本的な質問ですみません。
>>115
どっちでも同じ。考えうる "場合" の総数という意味。
11782:04/07/24 17:20
>>82もおねがい。 やっぱりできない。。。
>>117
何が?
次数が決まったら、各係数を未知数にして与式を計算し、
多項式として等しくなるように次数ごとに比較すればいい。
>>82
f(x)が一次式なら
左辺は二次式 右辺は四次式 不適
二次式なら
左辺は四次式 右辺は四次式 適する

f(x)が二次式、すなわちax^2+bx+cとおけるわけだ
実際やってみると、すぐに決まってしまう箇所もあり、そんなに面倒でもなかった。

>>119
違うだろ・・・
アヒャヒャヒャ
122112:04/07/24 20:11
>>113
すいません、基本がなってなくて・・。私、数学の偏差値が35なので・・・。
>>114
ありがとうございます。なんとか解けました。
123FeaturesOfTheGod ◆UdoWOLrsDM :04/07/24 20:17
数学の偏差値80の男が聞いてあきれる。
(さすがに80は大袈裟か?)
124132人目の素数さん :04/07/24 21:05
すいません、数IIIの範囲です。
関数y=f(x)のx=aにおける微分係数lim_[x→a]{f(x)-f(a)}/(x-a),
lim_[h→0]{f(a+h)-f(a)}/hの値f'(a)が存在するとき、
f(x)はx=aで微分可能であるということですけど、
逆にf(x)はx=aで連続であってもx=aで微分可能であるとは
限らないんですよね?
どのようなときがf(x)が連続で微分不可能であるのかが
よく解らないのですが、それとも、関数の連続性というものは、
大学生になるまで追求しない方がいいのでしょうか?
>>124
y=|x-a|なんかは代表的な奴。

とりあえずxがaに近づくときに
右からと左からで微分係数が異なる場合を考える。
関数f(x)について
x=aで微分可能でない⇒x=aにおいて連続でない は偽だから対偶の

x=aにおいて連続⇒x=aで微分可能 も偽

証明は反例がすぐあがる
127132人目の素数さん:04/07/24 21:14
>>125
即レスありがとうございます。
左右から考えるんですか。(左から右ばっかり考えてました。)
128124:04/07/24 21:25
>>126
ありがとうございます。反例を示すという手間を怠ってました。
129132人目の素数さん:04/07/24 21:59
△ABCにおいて、辺BC、CA、ABを2:1に内分する点をそれぞれL,M,Nとするとき、
△ABC,△LMNの重心は一致することを誰か証明して下さい。


130132人目の素数さん:04/07/24 22:04
>>129
重心をベクトルで表すことはできる?
131132人目の素数さん:04/07/24 22:06
>>129
A,B,Cの位置ベクトルを a, b, cとすると
△ABCの重心は (1/3) (a+b+c)

また、
Lの位置ベクトルは (1/3)(b+2c)
Mの位置ベクトルは (1/3)(c+2a)
Nの位置ベクトルは (1/3)(a+2b)

であり、△LMNの重心も (1/3)(a+b+c)とわかる。
132132人目の素数さん:04/07/24 22:11
>>131
どうもです(^o^)
133132人目の素数さん:04/07/24 22:28
A=(1,3)、B=(−2,0)、C=(−1,−2)のとき、△ABCの面積を求めよ。
HELP ME!
134132人目の素数さん:04/07/24 22:50
>>133
S=ABxAC/2=(B-A)x(C-A)/2
=(-3,-3,0)x(-2,-5,0)/2=3(1,1,0)x(2,5,0)/2
=(0,0,5-2)3/2=(0,0,9/2)->|S|=9/2=4.5
135132人目の素数さん:04/07/24 22:53
>>133
方眼紙に三角形描いてみれ
>>133
とても高校生レベルとは思えないのですが。
137132人目の素数さん:04/07/24 23:03
直線の式ださずに座標だけでやる方法があったようなー・・・あー忘れた!!
138132人目の素数さん:04/07/24 23:09
普通にxy平面に点を取って、長方形とか、三角形の面積の求め方を知っていれば
馬鹿でもできる問題
>>133は学校に行ったことすらないのかもしれない。
>>137>>134が見えないのか?
140132人目の素数さん:04/07/24 23:13
それってこんな感じのじゃない?
ベクトルA=(A1,A2),ベクトルB=(B1,B2)とおいたら
S=|A1*B2−A2*B1|
間違ってたらスンマセン
141132人目の素数さん:04/07/24 23:16
>>139
単に更新押してなかっただけじゃないの?
そんなのいちいちつっこむなよw
142132人目の素数さん:04/07/24 23:16
>>140
平行四辺形の面積求めてどうするの?
ヤコビアーーーーーーーーーン
143132人目の素数さん:04/07/24 23:17
>>142
半分にしたら三角形
144132人目の素数さん:04/07/24 23:22
>>143
そんなの誰でも分かるしw
マジレスカコワルイ
145132人目の素数さん:04/07/24 23:26
と、汗をかきながら言う142
146132人目の素数さん:04/07/24 23:30
と、うんこもらしながら言うヤコビアンも知らない145
147132人目の素数さん:04/07/24 23:33
>>138
馬鹿はそれもできない
148132人目の素数さん:04/07/24 23:33
いや、馬鹿の下にカスってランクがあってさ
149132人目の素数さん:04/07/25 01:21
アホみたいな質問で恐縮ですが、1次の行列uと実数uは違いますよね?
例えば積の行列計算で(u)M[a,b]M[c,d]=M[ua+uc,ub+ud]
u*M[a,b]M[c,d]=M[ua,ub]M[uc,ud]となり値が異なります。
教科書には1次の行列(u)を単にuと表記することが多いと書いてあるので、
一度混同してしまいました・・・。
M[a,b]ってなんだ?
151132人目の素数さん:04/07/25 01:28
>>150
A=M[a,b]M[c,d]とおいて、仮に1次の正方行列とAとの積を考えてみました。
いや、だからM[a,b]という記号の意味がわからないんだけど。
153132人目の素数さん:04/07/25 01:30
>>150
すいません。A=[M[a,b]M[c,d]]ということです。括弧忘れてますた・・・。
154訂正:04/07/25 01:32
アホみたいな質問で恐縮ですが、1次の行列uと実数uは違いますよね?
例えば積の行列計算で(u)[M[a,b]M[c,d]]=[M[ua+uc,ub+ud]]
u*[M[a,b]M[c,d]]=[M[ua,ub]M[uc,ud]]となり値が異なります。
教科書には1次の行列(u)を単にuと表記することが多いと書いてあるので、
一度混同してしまいました・・・。


ようするに

a c
b d

みたいに並んだ2*2行列の事?だとすると、1*1行列と2*2行列の
積なんて無いけど。
156訂正:04/07/25 01:34
更に(u)=[u]でした。たびたびすいません。行列記号書いたの初めてで・・・。
157132人目の素数さん:04/07/25 01:35
>>155
ああそうか・・・。積の場合、列数と行数が一致しない場合は考えないもの
とするんだった。迂闊でした。すいません。ありがとう・・・。
158132人目の素数さん:04/07/25 01:42
ということは1次の正方行列[u]と1×2行列A=[a,b]の積は、[ua,ub]となる
ことから考えても1次の正方行列[u]は実数uですね?
>>158

自然に同一視できると思うけど。
160132人目の素数さん:04/07/25 01:53
∈と⊂のちがいがよく分かりません。教えてもらえますか??
>>160
a∈Aは元aが含まれる集合Aに含まれる
A⊂Bは集合Aが集合Bに含まれる
162161:04/07/25 01:55
訂正:
>>160
a∈Aはaが集合Aに含まれる(aは集合Aの元である)
A⊂Bは集合Aが集合Bに含まれる
163132人目の素数さん:04/07/25 01:58
たとえば、A={1,2,3}で、1∈A,{1}⊂Aというのはわかるのですけど、
1と{1}のちがいがよくわかりません。
1は要素であって集合の一部ではない。{1}は集合の一部であって要素ではない。
「A⊂B⇔集合Aが集合Bに含まれる」とは、集合Aが集合Bの一部であるということである。
>>163
だーかーらー、>>162が書いてるだろ。

1はAの要素、{1}はAの部分集合っつーこと。
166165:04/07/25 02:05
あ、かぶった。
167132人目の素数さん:04/07/25 02:06
すいません。
要素と集合というのがわからないのかもしれません。
もう少し考えて見ます。
>>161さんと>>164さんありがとうございました。
168132人目の素数さん:04/07/25 02:07
あ、あと>>165さんも。
169165:04/07/25 02:13
>>167
ベン図でも書くか?

まず、円を書いてその中に
1、2、3と数字を書いてみ。
その1っつーのが要素。

んで、次に1の回りに小さい円を書くと
集合の中に小さい集合が含まれた形を表現できる、つーのは
イメージできるか?
その内側にある小さい集合が部分集合だ。オケ?

普通、集合を表すのにいちいち円なんか書いてられんから
{}の中に要素なり条件なりを書いて進めるわけだな。
170132人目の素数さん:04/07/25 02:21
うーん?1と、1が入った小さい円のちがいは何ですか??
それが要素と集合のちがいということですか??
171132人目の素数さん:04/07/25 02:23
あ、わかりました。
わあ、どうもありがとうございました。
172165:04/07/25 02:25
>>170
そういうこと。
この図で書いてる円が集合を表してる。

ちなみに空集合というのは
どの要素も含まないように書いた円だと思えばヨシ。
2個の赤球と2n個の白球を一列に並べる時
白球が偶数個連続して並ぶような並べ方は何通りあるんでしょうか?
すみません。
白球が偶数個ずつ連続して並ぶような並べ方でした・・・
問題の意味がよく分かりません!
数列 a_n は実数とし、以下の2002個の連立方程式を満たす。
--------------------------------------------------
Σ_[k=1,2002] {(a_k)/(k+1)} = 4/3
Σ_[k=1,2002] {(a_k)/(k+2)} = 4/5
Σ_[k=1,2002] {(a_k)/(k+3)} = 4/7
Σ_[k=1,2002] {(a_k)/(k+4)} = 4/9
Σ_[k=1,2002] {(a_k)/(k+5)} = 4/11








Σ_[k=1,2002] {(a_k)/(k+2001)} = 4/4003
Σ_[k=1,2002] {(a_k)/(k+2002)} = 4/4005
--------------------------------------------------
このとき、
Σ_[k=1,2002] {(a_k)/(2k+1)}
は何になるか教えてください。
176132人目の素数さん:04/07/25 14:37
新課程青チャート 数T+Aの基本例題14(因数分解)の(3)番

a^2(b-c)+b^2(c-a)+c^2(a-b)

となっていますが、回答の手順に

(与式)=(b-c)a^2-(b^2-c^2)a+(b-c)bc

とありました、でも、どうしてこうなるのかが分かりません。
宜しければ教えてください。
>>176
a について整理しただけやんけ。
178176:04/07/25 15:42
展開してみたら分かりました。
>>173
例えばn=2の場合を考えると
●○○●○○は白が2個(偶数個)ずつ並んでいるからOKだが、
○●○○●○だと白が1個(奇数個)しか並んでいない部分があるからダメ
という意味。

問題の解き方のヒントは、偶数個の白玉を半分の数の大玉に置き換えると…
180132人目の素数さん:04/07/25 22:20
>>16 の(1)の図が書けません…
181132人目の素数さん:04/07/25 22:42
>>180
とりあえずΔABCと点M、点P、点Qを書いてみる。
xは適当に決めて書けばいい。
182132人目の素数さん:04/07/25 22:47
書くと、正方形が出ます。そしたらみなさんは、正方形は出ないとか出るとかで、ずっとこんがらがってるんです…助けてください
>>182
そりゃーBP=CPとか
PをBCの中点とかに取れば
正方形も出るだろうが
それじゃ問題を正確に読み取ってないだけだろ。

つか、四日も五日も考えて
まだそんなとこでストップしてる、つーのが…
>182  P は B から C に向かう。Q は C から上に向かう。
三角形 ABC の面積 S(x) が最大になる時は 4点 QMPC は 5×5 の正方形を成すが
問題を解くことと関係はない。何を迷っているのか?
任意の P に対して S(x) を式で表し計算で最大値を求めるだけだ。
 A
  |\   図が巧く見えれば良いが。
  |  \
  |    \ M
  |    \
Q |       \
x | _____ _\C
 B     P x
185132人目の素数さん:04/07/25 23:25
>>184
おいおい・・・・
余計に混乱させるだけだろ・・・・
>>180
ttp://dokuo-ha-hitori.dyndns.tv/~dokuo/cgi-bin/zuru/source/dokuo5306.gif

△MPQは△ABCから三つの周りの三角形を引いたものだろ
図よりMD=5
187183:04/07/25 23:34
ちなみに、△AMQとか△BMPとかの面積を
個別に求める必要がない、つー事実を
理解しとるんか?質問者は。
188132人目の素数さん:04/07/25 23:35
>>180
(1)は出来たって言ってなかったか??
(1)が出来たら
求めた面積が二次式なんだから平方完成して終わりだろうがYO

0≦x≦10でな
190132人目の素数さん:04/07/26 16:26
AとBの二人が硬貨を投げるゲームをしている。
表が出ればAはBから1点もらえる裏が出ればBがAから1点もらえる
ABともに持ち点5から始めて先に10点を得たら勝ちとする。
硬貨を10回投げる間にAが勝つ確率を求めてください。
191190:04/07/26 16:38
偶数回投げて決まる事はないから5回7回9回投げた場合を考える。
5回投げて決まる場合の数は1通り
7回の場合は最後に裏が出た場合と最後から二番目に裏が出た場合を引いて5通り
9回の場合も7回の場合とほぼ同様に解けばよろしいですかね?

問題参照用のレスアンカー>>190


192132人目の素数さん:04/07/26 18:57
数Vで質問なんですけど
An=1・3+3・5+・・・(2n−1)(2n+1)とおくとき、
lim = An/n^3を求めよ。
n→∞
のとき方教えてください。明日テストなのになんにもわかりません…
だれか助けて・・・
193132人目の素数さん:04/07/26 19:06
z=cos360°/7+isin360°/7 とおく。

複素数平面において1,z,z^2,z^3,z^4,z^5,z^6が表す点をそれぞれP0,P1,P2,P3,P4,P5,P6とする。
三角形P1P2P4の重心をQ(α)、三角形P3P5P6の重心をR(β)とおくとき、複素数αとβを求めよ。

三角形P0QRの面積を求めよ。

という問題があるのですが、全くわからないので解法を教えていただけないでしょうか?
よろしくお願いします。
>>192
区分求積法
A_n/n^3=(1/n)Σ(4-(1/n^2))
195132人目の素数さん:04/07/26 19:09
>>192
A_nぐらいもとめれなくちゃいけないよ
>>192
どうせnが2次以下の項は極限取れば消えるから注目すべきは
Σ4n^2=4n(n+1)(2n+1)/6
の三次の係数
4*2/6=4/3
197192:04/07/26 19:23
ぶっちゃけ自分大学なんですけど、高校で数Vやってないからさっぱりわかりません…。
194さんありがとうございます。
しかしΣの意味さえわかりません…。ちょっと調べてきます。
高校まちがえたなぁ…
ΣってのはΣ[k=1...n](4-(1/n^2)) ってこと。
分かりにくかったらスマソ
199192:04/07/26 19:31
198さんありがとうございます。
あとひとつ
lim = (x^3+8)/(x+2)の極限値がわかりません。
x -2

どうかおねがいします。
200192:04/07/26 19:33
訂正を

あとひとつ
lim = (x^3+8)/(x+2)の極限値がわかりません。
x→ -2

どうかおねがいします。
x^3+8の因数分解
202132人目の素数さん:04/07/26 20:08
互いに素ってどういう意味なんでしょう?

「互いに素であるから3(m+1)=5(l+1)はm+1=5n、l+1=3nとおける」と書いてあるんですが。

∫logxdx 

おながいします。
204202:04/07/26 20:12
あと、既約分数の意味も教えて下さい。
205132人目の素数さん:04/07/26 20:19
言葉の意味くらいは教科書読め! と言っていいですか
206202:04/07/26 20:31
すいません。意味は調べましたが、202の置き換えはまだよく分からないので誰か詳しく説明お願いします。
207132人目の素数さん:04/07/26 20:36
m(Y/n)=1/n(mY) を示せ という問題です。

もうお手上げです
よければどなたか解いていただけませんか?(切
208132人目の素数さん:04/07/26 20:38
>>206
じゃあ、3と5が互いに素というのはいいのかな?

この問題の場合は(おそらく m, l が整数という条件があるとして)
3(m+1)は5を約数に持つはずだが、3と5が互いに素なので、m+1の方が5を約数に持つことになる。

ただ、互いに素とまで言わなくても、
3は5の倍数ではないのでm+1が5の倍数になる、
でも出来るけどね
209132人目の素数さん:04/07/26 20:39
して礼は?
210132人目の素数さん:04/07/26 20:40
>>207
両辺とも (mY)/n でいいだろ
それとも ( ) に特別な意味があるのか?
211132人目の素数さん:04/07/26 20:58
もしよろしければ、>>203もおねがいします。
212132人目の素数さん:04/07/26 20:59
>>211
部分積分。

log(x) = (x)' log(x)
213132人目の素数さん:04/07/26 21:03
∫logxdx
=∫1・logxdx
=xlogx-∫(x・1/x)dx
=xlogx-1+C
214132人目の素数さん:04/07/26 21:07
>>208
ふむふむ(゜o゜)
僕の卓越した頭脳で容易に理解してしまいました。いくら僕の頭脳が常人を遥かに超越しているとはいえ貴方の力添えが多少あったのも確かです。僕はこの事実を真摯に受け止め更なる高みを目指し精進したいと思います。
そう、あの聖なる頂東京大学理科一類を目指して…。どうもありがとうございましたm(__)m
215132人目の素数さん:04/07/26 21:16
>>210
即レスありがとうございます^^
それがですね、
ものっすごくあまのじゃく(ひねくれた?)な先生が出した問題なので、
なんか特別な解き方じゃないといけないのかと・・・><

分数をとらえろ!とかいって叫んでました;
216132人目の素数さん:04/07/26 21:29
座標空間に3点A(2,-1,2)、B(8,-4,4)、C(4,1,1)がある。
点E(p,0,0)が平面ABC上にあるとき、pを求めよ。

分からないので、解いてください!!お願いします。
217132人目の素数さん:04/07/26 21:30
集合の問題です。

A∨(B∧C)=(A∨B)∧(A∨C) を証明せよ。

たのんます!
218193:04/07/26 21:31
お願いですからスルーしないでください・・・。
せめてヒントだけでも教えていただけないでしょうか?
さっきから考えても本当にわかりません・・・。
どうか・・・お願いします!
>>216
とりあえず、平面の式を出してみたら。
>>193
z=cos360°/7+isin360°/7 が意味不明です
分数の中に分数?
>>193
α+β と (α+β)^2 を計算してみればあ?
223222:04/07/26 21:37
>>193
× α+β と (α+β)^2 を計算してみればあ?
○ α+β と (α-β)^2 を計算してみればあ?
224132人目の素数さん:04/07/26 21:40
>>219
それでも分かりませんでした。。助けて!!
225193:04/07/26 21:57
>>221
分数の中に分数ってどういうことですか?
極形式でコサイン7分の360°+iサイン7分の360°がなぜ分数の中に分数になるんですか?

煽りとかじゃなくて本気でわかんないです。
226132人目の素数さん:04/07/26 22:10
>>224
平面の式の求め方くらい教科書にあるだろ?
227132人目の素数さん:04/07/26 22:14
>>213
プププ
228132人目の素数さん:04/07/26 22:19
>>218

要は、$\alpha $ が求まれば、万事解決やろ? だって、$\alpha $ と
$\beta $ は共役やから。問題は、$\alpha $ をちゃんとした形($x + yi$)
で求められるかやな、、、。さて、

単位円と $1 , z , ... , z^6 $ のなす正六角形の対称性より、
$\alpha $ と $\beta $ は共役。そこで、$\alpha + \beta $ と
$\alpha \beta $ を求める。$1 + z + ... + z^6 = 0$ より \\
$\alpha + \beta = (z^1 + ... + z^6)/3 = - 1/3$ ,
$\alpha \beta = (z^1 + z^2 + z^4)(z^3 + z^5 + z^6) /9 = 2/9$
以上より、$\alpha , \beta $ を求めると、
$\alpha = -1/6 + 7/3 i , \beta = -1/6 - 7/3 i$ なので、
求める三角形の面積は、$49/18$。

計算間違ってたらごめんな。$1 + z + ... + z^6 = 0$ になる理由は、図
書いても求められるし、因数分解で、
$z^7 - 1 = (z - 1)(z^6 + ... + z + 1) = 0$ として導いても良し。
大学入試か何かか? よう分からんけど、頑張りや。


229132人目の素数さん:04/07/26 22:20
↑ごめん。「正六角形」→「正七角形」。
230132人目の素数さん:04/07/26 22:21
↑計算もだいぶ間違ってるな。でも、解法は確かやと思う。
231228:04/07/26 22:25
正解は多分、$7 \sqrt{7} /36$ やと思う。ちょっと急ぎすぎた。X-(
$\sqrt{7} $ は √7 のこと。
232228:04/07/26 22:31
このスレのタイトル見たら、高校生向けか。なら、TeX 知らんか。。。

\alpha , \beta はそれぞれ、α、βのこと。後は大体分かるやろ。
x^2+14x+48<0を満たすようなすべてのxが、x^2-ax-2a^2>0をみたすとき
aのとりうる範囲を求めよ。 なんですが、これって、まず左の式
でxの範囲を求めて、その範囲内で右の式の最小値が正になるということを
場合分けで求めるんですか?
234132人目の素数さん:04/07/26 22:45
>>213
xろぐx-x+C
だな

糞煽りが
235132人目の素数さん:04/07/26 22:46
>>227
死ね
236132人目の素数さん:04/07/26 22:47
>>233
OK
>>236 ありがとうございます
238190:04/07/26 23:47
>>190ですがよろしくお願いします
239193:04/07/27 00:29
>>288
助かりました!とってもわかりやすく解説していただいてありがとうございました!
20円以上の切手は5円切手と6円切手の組み合わせとして買える事を示せ、という問題で、解答は数学的帰納法を用いているのですが、

【解答】(求める切手の値段をNとして)
1.N=20のとき、5円切手4枚で買える。よって成り立つ
2.N=k(kは20以上)のとき題意は満たされると仮定して、5円切手の枚数をx、6円切手の枚数をyとすると、
このとき、i=1 2 3 4とすると、k+iは(5円:x-i枚、6円:y+i枚)のもとで必ず成立する

以上が解答なのですが、どうも府に落ちません。数学的帰納法は「始まり」と「kとk+1、k+2、…」を確約することでドミノのように証明するんですよね?
でも、これじゃ20 21 22 23 24は証明できても、24と25のつながりを示せてないから、それ以降は証明出来ないような気がするのですが…そうではなく、自分が証明の2番の文章を完璧には理解できてないだけなのでしょうか。
何か見落としているのでしょうか?アドバイスお願いします。
>>240
「n円の切手は…」ではなくて
「n円以下の全ての切手は…」を証明するつもりで考えてみよう。
参考
http://ja.wikipedia.org/wiki/%E6%95%B0%E5%AD%A6%E7%9A%84%E5%B8%B0%E7%B4%8D%E6%B3%95
242241:04/07/27 08:47
よく見たら、元の回答も不十分だね。
1.N=20〜24のとき成り立つ
2.N=kのとき5円x枚、6円y枚で買えるならば、
  N=k+5のとき5円x+1枚、6円y枚で買える。
ならば筋が通っているけれど。
高校生レベルならばここから5本に分岐したドミノ倒しを考えれば十分かな。

で、厳密にやるならば
1.N=20〜24のとき成り立つ
2.N=k(k≧24)のときk円以下の切手が全て5円と6円の組み合わせで買えるとする。
  特にk-4円の切手が5円x枚、6円y枚で買えるとすると
  k+1円の切手は5円x+1枚、6円y枚で買える。
これなら普通の数学的帰納法
243240:04/07/27 08:59
>>241
ありがとうございます!
244 :04/07/27 12:45
三角形ABCにおいてBC=13 外接円の半径R=13/√3
∠Aは鈍角とする。

AB+AC=15 AB≧ACのとき
AB,ACの長さを求めてください
245132人目の素数さん:04/07/27 13:59
>>244
AB=c AC=b BC=a=13とおく

正弦定理より
2R=13/sinA
∴sinA=√3/2

またAは鈍角よりcosA=-1/2
ここで余弦定理より
cosA=(b^2+c^2-a^2)/2bc
cosA={(b+c)^2-2bc-a^2}/2bc
代入して
-1/2=(225-169-2bc)/2bc
-1/2=(56/2bc)-1
1/2=56/2bc
∴bc=56

b+c=15
bc=56
よりb,cは
実数tについての二次方程式
t^2-15t+56=0 の解であるから
t=7,8

b≦cより
b=7
c=8
補足すると
二つの数の和と積と大小関係が分かれば
二つの数は二次方程式から求まる

問題で和と大小関係が分かってるわけだから
いかに積を導き出すかがポイント
248 :04/07/27 15:35
>>246-247
わかりやすいですわ〜
ありがとうございます!!

>>246
わかりやすいイイ回答ですね
250132人目の素数さん:04/07/27 18:31
すいません、質問させて下さい。

(一般項)=1/(√(n+2)+√n)

数列が0に収束して、級数が発散することは
式的には理解できてるつもりですが、直感的に分かりません。
別に直感的に分からなくてもよさそうなんですが、
どうにもこうにも引っかかるんで、どなたか教えて下さい。
251132人目の素数さん:04/07/27 18:37
有理化
>>250
Σ[n=1〜∞]1/nが発散するのは納得できるか?

1/(√(n+2)+√n)≒(1/2)(1/√n)で
1/nよりも更に収束が遅い(というか収束しないが)級数なのだが。
253高1:04/07/27 19:18
3+nC2・3^2+nC3・3^3+・・・
+nCn-1・3^n-1
を計算したら、(4^n)−(3^n)−3n+2
になりますか??
254132人目の素数さん:04/07/27 19:31
>>253

問題がよう分からんけど、
$3 + 3^2 {}_n C_2 + ... + 3^{n - 1} {}_n C_{n - 1}$ ということ?

それなら、二項定理から
$( \textrm{与式} ) = (1 + 3)^n - 1 - 3^n - 3n + 3
= 4^n - 3^n - 3n + 2$ になるから、正しいな。
α=cosθ+isinθに対して0,1,i,αi,α^2iをそれぞれO,A,B,C,Dとする。
更にOCとBDの交点をPとする。0°≦θ≦180°のときAP^2+PB^2の最大値、最小値を求めよ。
またこの時のαを求めよ。

どこから手をつけてよいものかさっぱりわかりません。どなたか解法を教えてください!
256132人目の素数さん:04/07/27 20:10
>>255
複素数平面で考える
257132人目の素数さん:04/07/27 20:55
>>255

$\vec{OP} = k \vec{OC} (0 \leqq k \leqq 1)$ とすると、
点 P は線分 BD 上にあるから、
$\vec{OP} = (1 - t) \vec{OB} + t \vec{OD} (0 \leqq t \leqq 1)$

$\vec{OB} = i, \vec{OC} = i \alpha , \vec{OD} = i \alpha^2 $
を代入し、整理すると、
$1 - t + \alpha^2 t = k \alpha $
$\alpha = x + yi$ を代入し、計算すると、何か式が出てくるから、
$x, y, t, k$ は実数のため、二つの方程式が出てくると思う。
それを $x^2 + y^2 = 1$ の条件下で計算し、$k , t$ をそれぞれ
$x , y$ で表す。あとは、$\vec{OP} $ が $x, y$ で表せるから、
計算で、$\vec{AP}, \vec{BP}$ を求めて、$AP^2 + BP^2$ を $x , y$
で表し、$x = \cos \theta , y = \sin \theta $ を代入すれば、
三角関数の問題になる。図も書いた方が良いと思う。
258高1:04/07/27 21:09
>>254
ありがとうございます!!
259132人目の素数さん:04/07/27 21:15
(0.99)^(99)と(1.01)^(-101)の大小関係を比較せよという問題で、
次のようにやったんですが、途中で引っかかってしまいました。
対数をとってから差をとって大小を比較します。
99log(0.99)-(-101log(1.01))=100*(0.99log(0.99)+1.01log(1.01))
上の式はベクトルA=(0.99,1.01)とB=(log(0.99),log(1.01))の内積なので
0.99log(0.99)+1.01log(1.01)=|A||B|cos(x)
だからあとは、xがどうなのかわかればいいんですけどこれって分かりますか?
それとも、これは見当違いの解答ですか?
教えてください、お願いします。
凸不等式使え
>>259
せっかく対数とったんだから微分を使うのはどうだろうか?
三角関数持ち出しても解決するのはムズかしいと思うよ。

aloga+(2−a)log(2−a)の増減を調べたり,
(x−1)^99*(x+1)^101の増減調べたりでできそう。
>>261
ああ、微分の発想がありませんでした。
やってみます。
ありがとうございました。
263255:04/07/27 22:10
>>257
ありがとうございました!
めちゃわかりやすくて助かりました!
264132人目の素数さん:04/07/27 22:10
d=1/100, a=(1-d)^99, b=(1+d)^{-101}とおくと,
a/b=[(1-d)^99]*[(1+d)^101]
=[(1-d)^100 (1+d)^100]*[(1+d)/(1-d)]

ここで,x>0,n>0に対して
(1-x)^n > 1-nx
だから(微分法ですぐわかる),
(1-d)^100 (1+d)^100 = (1-d^2)^100
> 1-100*d^2
= 99/100
また,
(1+d)/(1-d)=101/99
したがって
a/b > [99/100]*[101/99]
= 101/100>1

a>0,b>0に注意してa>bを得る.
265132人目の素数さん:04/07/27 22:32
円に内接する四角形ABCDにおいて、
AB=3、BC=9、CD=6、DA=6として。
対角線BDの長さを教えてください。
>>264
なるほど!
対数をとって差を考えるよりも、そのままで比を考えるということか。
確かに同じことですね。
しかもd=1/100である必要はないということですね。
もしかしてこうやって問題は作られたのかな?
とにかく分かりました!とても勉強になりました!
267264:04/07/27 22:43
>>266
対数とって微分する方が見易いとは思う
>>264は別解のつもりで書いた.
座標平面上に円x^2+y^2+2ax+(4a+4)y+2a-5=0……@がある。但し、aは定数である。

(1) 円@は、aの値に関係なく2定点A(ア,イウ)、B(エオ/カ,キ/ク)を通る。

…で、2定点を求める問題なんですが。(図形と式)

円の方程式x^2+y^2+lx+my+nに当てはめて、変形して

(x+2ax+a^2)+(y^2+(4a+4)y+(2a+2)^2)=a^2+4a^2+8a+4-2a+5
(x+a)^2+(y+(2a+2))^2=5a^2+6a+9

とまでやってみたのは良いのですが、「aの値に関係なく」ですから、aが絡まない部分を見つけなくてはなりませんよね?
このままやってもaが絡まない部分って無いように思えて、分からなくなってしまったので質問に来ました。
ここまでの経過、やり方が間違ってるのでしょうか?ご指導の程、よろしくお願いします。

出来れば、明日の早朝までにお願いします。
>>268
与式を a について整理する
(2x+4y+2)a+x^2+y^2+4y-5=0
ここで
2x+4y+2=0
x^2+y^2+4y-5=0
の二本の式を満たすような点(x,y)は a の値にかかわらず必ず最初の方程式を満たす。
>>265
余弦定理より

BD^2=CD^2+BC^2-2CD*BC*cosC
=36+81-108cosC
=117-108cosC―@

BD^2=AD^2+AB^2-2AD*AB*cosA
=36+9-36cosA
=45-36cosA
ここで円に内接するからA+C=180°より
cosA=cos(180°-C)=-cosC を代入して
BD^2=45+36cosC―A

@−Aより
0=72-144cosC
∴cosC=1/2

@に代入して
BD^2=117-54=63
∴BD=3√7 ∵BD>0

ちなみにトレミーの定理を知ってると便利 かもしれない
AC*BD=AB*AD+CB*CD
271255:04/07/27 23:42
>>257
わかったと書いておきながら度々すいません。

>$1 - t + \alpha^2 t = k \alpha $
$\alpha = x + yi$ を代入し、計算すると、何か式が出てくるから、
$x, y, t, k$ は実数のため、二つの方程式が出てくると思う。

式をどう変形すれば、2つの方程式になるのでしょうか?
代入、展開すると、x^2+2txyi-ty^2=kx+kyi
となったんですが、これ以降どう変形すれば、tとkがxとyで表せるのかわかりません。

申し訳ありませんがもう一度教えていただけないでしょうか?
272132人目の素数さん:04/07/27 23:45
>>270
詳しい回答ありがとうございました。
273132人目の素数さん:04/07/27 23:47
正数ってなんですか?
@正の整数
A0より大きい実数

のどっちでしょう?
274132人目の素数さん:04/07/27 23:54
うそじゃない数かも
正のマークで表された自然数のことです。
5=正
7=正T
276132人目の素数さん:04/07/28 00:02
男5人、女6人が一列に並ぶとき、特定の男女が隣り合い、男同士は隣合わない
確立を求めよ。
と言う問題です。

自分では、特定の男女一組と、残りの女5人の並び方は6!*2通りあり、このとき
どの並び方を見ても、他の男同士が隣り合わない並び方は6C4*4!通りある。
よって求める確立は 6!*6*5*4*3*2/11! で、1/77となるのですが、答えでは、
2!*5*5!*6*5*4*3/11!=5/462となっています。よかったら詳しい回答をお願いします。
>>273
(2)。(1)なら普通は正整数と書く。
278132人目の素数さん:04/07/28 00:12
-(2x-3)y^2+(1-x^2)y+5x+(x^2-1)y
この問を工夫して解くことが出来ると思うのですがわかりません。
>>278
寝言は寝てから言え。
問いにもなってないものを、工夫しようが何しようが解くことは出来ん。
281132人目の素数さん:04/07/28 00:16
>>278
xについて降べきの順に整理せよ
282132人目の素数さん:04/07/28 00:24
↑こういう問です。
>>278
工夫も何も展開するしかなかろうて
284132人目の素数さん:04/07/28 00:46
>278
(1-x^2)y+(x^2-1)y = 0 ってことくらいか
285132人目の素数さん:04/07/28 01:00
>>283-284
ありがとうございます。
>>276
問題文はそのままか?
他に条件がない限り藻舞のが正解だと思うんだが。

ちなみに「確率」な。
287286:04/07/28 01:05
ちなみに、2!*5*5!*6*5*4*3/11!=5/462となってる、つーことは
もしかして「特定の男女一組」が両端に来ないとか?

あと、6C4*4!=6P4ってのは習ってないのかな。
288132人目の素数さん:04/07/28 01:08
必要条件、十分条件という概念がいまいちよく分かりません。高校ではそこ
に限っては、暗記するよう言われますた。機械的には分かるのですが、問題
によっては、一般性を調べてから(必要条件)、十分性の確認を行ったりしま
すけど、そのときしっかりと理解しておいた方がいいような気がして・・。
必要条件や十分条件の意味まで理解するのは高校の範囲を超えるのでしょうか?
>>288
「A(であること)はB(であること)の必要条件」→「Bであるためには、まずAである必要がある」
例えば、正三角形であるためには、まず二等辺三角形でなければならない、とか。

…あー、ベン図でも書けばわかりやすいかも知れんが
文章だけで伝えるのは難しいのう。
290132人目の素数さん:04/07/28 01:31
>>288
> 必要条件や十分条件の意味まで理解するのは高校の範囲を超えるのでしょうか?

必要条件・十分条件は、高校数学の必要条件か?ってことだな。
まあ、十分条件じゃないことは想像できるが

マジレスするなら、悩む時間があるなら勉強しろってことだ
291132人目の素数さん:04/07/28 03:09
2点A(4,3)、B(-2,1)を直径の両端とする円
の方程式をだれかぁぁぁ・・・・!!
お願いします!頭悪すぎてごめんなさい!
ちなみに私は

(x+1)2乗+(y−3)2乗=26

になったんですが、なんかおかしい気がするんです!
てか絶対間違ってます!だれか助けてください!
292132人目の素数さん:04/07/28 03:53
>>291
中心の座標: A(4,3)、B(-2,1)
半径: ABの距離の半分

中心の座標でまず間違ってるね
293132人目の素数さん:04/07/28 03:54
>>292
おっと・・・
中心の座標: A(4,3)、B(-2,1) の中点
です
294132人目の素数さん:04/07/28 08:02
>>286
問題は自分もそうじゃないかと思って何回も読み返してます。
きっと回答者が男同士が隣合わないのと、男の両隣には女がいる、って
言うのを間違えたんですね。 ありがとうございました。
>>288
A→B(AならばB)と言う場合、
Aは、Bであるための十分条件。
Bは、Aであるための必要条件。
A←→Bと言う場合は、AとBは互いに必要十分条件

問題を解く=命題を変形していくとき、
同値変形(←→)だけで進めていけば必要十分条件だけど、
→しか成り立たない理屈を使ったら、その結論は必要条件でしかないから、
十分条件である確認をしないと最初の命題が成り立つ保証はない。
>>288
「A は B の必要条件である」の主語は A で述語が「必要条件である」。B であるためには A であることが必ず必要である。
つまり「A でないのに B である」なんてことは起きない。B が成り立てばかならず A も成り立ってないといけない。
式で書くと B⇒A が成り立つ。このとき A は B の必要条件って言う。

「A は B の十分条件である」主語は A で述語が「十分条件である」。B であるためには少なくとも A という条件さえ満たしていれば十分である。
つまり、A さえ成り立っていれば B も成り立つ。
式で書くと A⇒B が成り立つ。このとき A は B の十分条件って言う。
n,a,bを0以上の整数とする。a,bを未知数とする方程式
a^2+b^2=2^n…(*)
を考える。

(1)nは2以上とする。a,bが方程式(*)を満たすならば、a,bはともに偶数であることを証明せよ。(ただし、0は偶数に含める)
(2)0以上の整数nに対して、方程式(*)を満たす0以上の整数の組(a,b)をすべて求めよ。

どなたかこの問題の模範解答を仕上げて頂けないでしょうか。
予備校のテキストの問題で、授業では「無限降下法」なる胡散臭い手法を用いていたのですが、どうにも納得できないのです。
>>297
無限降下法が納得できないのか?
無限降下法は背理法と数学的帰納法の組み合わせ。

P(n)が真ならばあるk<nが存在してP(k)
←(対偶)→任意のk<nについてP(k)が偽ならばP(n)は偽

こう読み替えて、後者について数学的帰納法を使えば
任意の自然数nについてP(n)が偽と言える。
2点A(2,2,3),B(0,1,1)とxy平面上の動点Pがある。
AP+BPの最小値を求めよ。
答えは√21になるそうなんですが、そこまでの計算方法がわかりません。
どなたか教えてください!
>>299
B'(0,1,-1)という新しい点(実はxy平面についてBと面対称)を考えると、
BP=B'P ∴AP+BP=AP+B'P
AとB'はxy平面を挟んで反対側にあるから
折れ線APB'がもっとも短くなるのは一直線になったとき。
あとは自分で考えよう。
301268:04/07/28 12:08
>>269
お返事遅れますた。
ありがとうございます。おかげさまで分かりました。
302299:04/07/28 12:47
>>300
やっと解けました・・・
わかりやすい解説ありがとうございます。
303297:04/07/28 13:14
>>298
無限降下法とはそのような手法なのですか…授業のときはなにも説明がなかったもんで…
この手法を用いないと解答できないのでしょうか?
304132人目の素数さん:04/07/28 14:32
sin3θ=3sinθ-4sin^3θ
を証明する問題が分かりません。お願いします。
>>304
左辺=sin(2θ+θ)
で加法定理
>>305
すみません、そこから式がどうなるか分かりません。
>>303無限降下法知らなくても解ける。
(1)a=2c+1,b=2d+1として計算すると、4(c^2+4d^2+c+d)+2=4*2^(n-2)となり、
4の倍数の右辺に対して左辺が2の倍数となり成り立たない。
a=2c,b=2d+1として計算しても倍数で合わないため成り立たない。
よって、a,bはともに偶数。(ただし逆は必ずしも成り立たない)

(2)
補題:自然数mに対してn≧2mのとき、(*)を満たすならばa,bはともに2^mの倍数である。

証明:
m=1のときは(1)で証明済みである。
n≧2kのときa=c*2^k,b=d*2^kであるとする。n≧2(k+1)のときを考えると方程式は、
両辺2^2kで割って、
c^2+d^2=2^(n-2k)
となり、n-2k=n'とすると、n'≧2のとき上式を満たすc,dの条件に帰結する。
これを満たすc,dは(1)で求めたa,bの条件と同じであるので、
c、dともに偶数となり、つまりm=k+1のときa,bともに2^(k+1)の倍数となる。

この補題を用いると、a,bとc,dの関係を引き継いで、
(i)n=0のとき、a^2+b^2=1を満たすa,bはa or b=0 or 1である。
(ii)n=1のとき、a^2+b^2=2より、a,bはa,b=1,1である。
(iii)n=2kのとき、c^2+d^2=1より、c or d=0 or 1から、a or b=0 or 2^k
(iv)n=2k+1のとき、c^2+d^2=2より、c,d=1,1から、a,b=2^k,2^k
となる。
>>306
加法定理
sin(α+β)=sinαcosβ+cosαsinβ
で今、αが2θでβがθだ

二倍角の公式
sin2θ=2sinθcosθ
cos2θ=cos^2θ-sin^2θ=1-2sin^2θ=2cos^2θ-1
を代入すれば導ける

こいつは自分で導いて三倍角の公式ぐらい覚えておいた方がいい
309257:04/07/28 15:20
>>271

$x, y, k, t$ は実数なわけやから、
$1 - t + tx^2 - ty^2 + 2txy i = kx + ky i$ で、実部と虚部を比較すれ
ばよろしい。そしたら、
$(2tx - k)y = 0 , 1 - t + tx^2 - ty^2 = kx$
が出てくるから、第一式より、$y = 0 $ または $k = 2tx$ となるから、
$y = 0$ か $y ¥not= 0$ かで場合分けすると良い。
原点Oと点A(3,0,0),B(0,6,0),C(1,2,8)を頂点とする四面体OABCがある。
線分AB上の点をPとするとき、CPの最小値を求めよ。
CP⊥ABの時に最小になるのだろうと思いますが、
そこからの求め方がわかりません・・・
311132人目の素数さん:04/07/28 15:47
>>310

なぜ、「四面体OABC」になるのか意味が分からん。△ABC で考えればす
むことやし。まぁええわ。

△ABCの図を書いてごらん。点 C を中心とし、直線 AB に接する円を描
いたら、その接点を P としたら、そのときの CP が最小になる。なぜ
なら、それ以外の直線 AB 上の点 Q では、円 C の外になるので、
CP < CQ 。よって、CP が AB に垂直のとき、CP は最小となる。

あとは、計算で、ベクトルで、$¥vec{CP} ¥cdot ¥vec{AB} = 0$ とな
ることを利用して、計算すればよろしい。$¥vec{AP} = t¥vec{AB}$
より、$t = 4/9$ 。
>>308
何回計算しても導けません。申し訳ありませんが全部教えてください。
>>312
と思ったらできました。ごめんなさい。
ありがとうございました。
314310:04/07/28 16:36
>>311
おかげさまで解けました。ありがとうございます。
物理の授業に微分方程式出てきたんですけど
たとえば(1/x)dx=duにそのままインテグラル記号を付けて
∫(1/x)dx=∫du に出来ちゃったりするのが意味不明です、
どうしてこうなるのか教えてください。
変な説明ですいません。
dx/du=x
1/x(dx/du)=1
積分して
∫(1/x)(dx/du)du=∫du
u→x置換積分をすると、(dx/du)du=dx
∫(1/x)dx=∫du

の途中式の∫を省いただけ。
>>316
分かりやすい説明ありがとうございました。
大体理解できました。
318132人目の素数さん:04/07/28 18:07
数学の偏差値75にしたいのですが 何やればいいでしょうか
319132人目の素数さん:04/07/28 18:17
xy平面上に3点A(1,1),B(7,3),C(4,6)がある。
(3)点Pが直線AB上を動くとき,三角形PBCの重心Gは直線x-@y+@=0上にある。

どうやって解けばいいでしょうか。
>>318
平均との差が標準偏差の2.5倍になるような数をとればいいですよ。
321妃 ◆fvsh1Vqszw :04/07/28 18:22
まず2ch止めることかな。
>>307
ありがとうございました!
>>319
(1)媒介変数tを使って、点Pの座標を表す。
(2)△PBCの重心Gの座標を計算する
(3)媒介変数tを消去する
という方針でできると思うが、どこか引っかかるところは有るか?
324132人目の素数さん:04/07/28 18:37
媒介変数tって(a,b)に置き換えればいいんですよね。
そうするとどうやって点Pの座標を表せばいいんですか?
>>324
AとBの座標が出てるからそこから直線ABの式をだして点Pの座標をだす
326132人目の素数さん:04/07/28 19:21
スタート □□□□□★□□■ ゴール

双六をしていて、現在★の所に居ます。ゴールが■で
さいころを振ってぴったりに上がれなければその分だけ戻ります。
ちょうど2回で上がれる確率を求めなさい。

-----

って問題なのですが2回で上がるためのさいころの出る目は5通りだというのはわかります。
(1,2)(2,1)(4,1)(5,2)(6,3)ですよね。

ただ、答えに5/36とあるのですが、どうして分母のほうが36になるのかがわかりません。

1回目に3でゴールをしてしまったならば、2回目でさいころを振ることはないのだから
36通りってのはおかしいのではないでしょうか?
327132人目の素数さん:04/07/28 20:14
>>326
問題がおかしい
>>326
1回目に3が出たならば実際には2回目は振らないが、
仮に振ったとしたら2回目は何が出てもよい。
つまり、2回振ることを前提とするならば、1回目で上がってしまう場合は6通り。
これを1通りと考えると、各可能性の確率が均等にならない。
329132人目の素数さん:04/07/28 21:01
以下,抜粋のような形で申し訳ありませんが,質問させてください.

a,bは実数(b≠0),n,kは整数,|n|≧1 で,
0<|a|<|n|, (a-(n/2))^2+b^2 > n^2/4 …@
という条件をもとに,
f(k,n)=(a+k-(1/2))^2+b^2-(1/4) > 0 …A
を証明したいと考えます.

ab平面で考えると,@とAはともに円をあらわしていますが,これ
を踏まえると,証明はどのようになりますか.
330255:04/07/28 21:02
昨日と同じ問題ですいません。

α=cosθ+isinθに対して0,1,i,αi,α^2iをそれぞれO,A,B,C,Dとする。
更にOCとBDの交点をPとする。0°≦θ≦180°のときAP^2+PB^2の最大値、最小値を求めよ。
またこの時のαを求めよ。

このとき交点Pが -sin2θ/2+i(1-cos2θ)/2
となるらしいのですが、どういう計算でこうなるのか、よろしければどなたか教えてください。
331132人目の素数さん:04/07/28 21:04
訂正です.

「円をあらわしています」

「円の外側の領域(境界含まず)をあらわしています」

失礼しました.
332132人目の素数さん:04/07/28 21:08
{(1,2)(2,1)(4,1)(5,2)(6,3)}/{(x,y)}=5/36
333297:04/07/28 21:16
すいません…>>307さんかどなたか、やっぱり>>297の(2)を無限降下法をつかって解いて頂けませんか…?
一回納得したくせにすいません…
334255:04/07/28 21:40
>>330

× i(1-cos2θ)/2

○ i(1+cos2θ)/2

です。失礼しました。
>>329
2 番目の円が 1 番目の円の外部にあるか、2 番目の円が
1 番目の円の内部にあることを示せばよい。

2 つの円の中心間の距離は |k-(n-1)/2|,
1 番目の円の半径は |n/2|, 2 番目の円の半径は 1/2 であるから、
2 番目の円が 1 番目の円の外部にあるための条件は
|k-(n-1)/2| ≧ |n/2| + 1/2
2 番目の円が 1 番目の円の内部にあるための条件は
|k-(n-1)/2| + 1/2 ≦ |n/2|.
k, n が整数という条件より、これらはそれぞれ次と同値である。
|k-(n-1)/2| > |n/2|, |k-(n-1)/2| < |n/2|.

したがって、|k-(n-1)/2| = |n/2| となる場合だけを調べればよいが、
絶対値の中身の正負で場合分けをして整理すると、2k+1=2n または 2k+1=0 となる。
k, n は整数なので、左辺は奇数、右辺は偶数となりこの場合は生じない。
336132人目の素数さん:04/07/28 21:52
x>0,nは自然数、f(x)=1+x/(1!)+(x^2)/(2!)+...+(x^n)/(n!)のとき、
e^x>f(x)を証明せよという問題の解答を、g(x)=(e^x)/f(x)とするとg(0)=1,g'(x)>0なので、
g(x)>1として証明したんですけど、微分を使わないで証明する方法はありますか?
もしくは、e^x>1+x+(x^2)/2の証明の微分を使わないやり方でもいいです。
お願いします。
あと、いくつも申し訳ないですけどその不等式を使ってlim(x→∞)(x^k)e^(-x)=0
の示し方の概要も教えてもらえるとうれしいです。
337132人目の素数さん:04/07/28 21:56
不定形にはロピタルの定理をつかえ
338132人目の素数さん:04/07/28 22:04
a>0 , b>0 , a+b=1 , A=ax+by , B=bx+ay  のとき
x^2+y^2 と A^2+B^2 の大小関係を調べなさい。

これがどう足掻いてもできませんでした。
ヒントをください。

339微積できん:04/07/28 22:13
突然ですが、
曲線C:y=-x2(xの2乗)+5x-3について、
(1)Cと曲線y=2x2(xの2乗)-7x+6とで囲まれた面積S1は?
(2)Cと直線y=mxとが2点で交わるとき、それらで囲まれた図形の面積S2を求めよ。
また、S2=S1/3を満たす定数mは?

という問題が分かりません。
申し訳ないんですが誰か解き方と簡単な解説をお願いします(泣)
340FeaturesOfTheGod ◆UdoWOLrsDM :04/07/28 22:16
Re:>339
(1)はグラフの交点を求めて、二交点間で、(-x^2+5x-3)-(2x^2-7x+6)を積分しよう。
(2)もグラフの交点を求めて、二交点間で、-x^2+5x-3-mxを積分しよう。
>>336
e^x をどのように定義するかによって問題の難易が変ってくる。
342329:04/07/28 22:30
>>335 さん,お返事ありがとうございます.

>2 番目の円が 1 番目の円の外部にあるか、2 番目の円が
>1 番目の円の内部にあることを示せばよい。

の部分だけがどうしてもわかりません.恥ずかしながら.
簡単な解説をいただけたら助かります.すいません.
343微積できん:04/07/28 22:33
Re:>340
有難うございます。
ただ、根本からわかってないのようなやつなんで、もう少し詳しくお願いします_| ̄|○
344330:04/07/28 22:52
複素数平面で、
O(0.0) A(1,0) B(0,1) C(-sinθ ,cosθ ) D(-sin2θ ,cos2θ )
となると思うんですが、これを使ってOCとBDとその交点Pはどのように表せばいいんでしょうか?

2日も同じ質問をして申し訳ないと思っていますが、
早朝までにでいいんでどなたか本当に教えてください!ヒントだけでもお願いします!
>>342
n=2 で k=1 のとき、(-1/2,1/2) は 1 番目の円の外部にありますが、
2 番目の円の周上にあって、そもそも証明すべきことが成立しないようです。

>>335 の解答では、a と n が同符号という問題にない仮定をしてしまっていました。
>>339
(1)-x^2+5x-3=2x^2-7x+6⇔x^2-4x+3=0⇔x=1,3
よってS1=∫[1,3]{(-x^2+5x-3)-(2x^2-7x+6)}dx=3∫[1,3](x-1)(3-x)dx
=3(1/6)(3-1)^3=4
(2)-x^2+5x-3=mxの2解をα,β(α<β)とする。このとき、解と係数の関係
によりα+β=5-m、αβ=3。
よってS2=∫[α,β]{(-x^2+5x-3)-mx}dx=∫[α,β](x-α)(β-x)dx
=(1/6)(β-α)^3=(1/6){(α+β)^2-4αβ}^(3/2)=(1/6){(5-m)^2-12}^(3/2)
=(1/6){m^2-10m+13}^(3/2)
また、S2=S1/3⇔(1/6){m^2-10m+13}^(3/2)=(1/6)2^3⇔m^2-10m+13=2^2
⇔m^2-10m+9=0⇔m=1,9

準公式∫[α,β](x-α)(β-x)dx=(1/6)(β-α)^3が分かっていれば
積分は必要ない問題。
347336:04/07/28 23:08
>>337
早速の返事ありがとうございます。
e^x>f(x)を使って証明はできませんか?問題にそう書いてあるので。
>>341
eの定義にまで戻らないと無理ということですか。いや、なんか微分を使うのは
最終手段のような気がして他に方法がないかなと思ったのです。
どうもありがとうございました。
xy平面円に円 C:x^2+y^2=25がある。
この円周上の点のA(3,4)における接線をlとし、円Cの弦のうち、
lが長さが6のものを弦PQとする。
ただし点Pのx座標より点Qのx座標は大きく点P,Qのy座標は正とする。

直線PQの方程式と点P,Qの座標をそれぞれ求めなさい。

どなたかお願い致します。
349132人目の素数さん:04/07/28 23:19
a>0 , b>0 , a+b=1 , A=ax+by , B=bx+ay  のとき
x^2+y^2
(A+B)=x+y
A^2+B^2=(x+y)^2-2AB=x^2+y^2+2(xy-AB)
=(1-2ab)(x^2+y^2)+2(-a^2-b^2+1)xy
A^2+B^2-(x^2+y^2)=-2ab(x^2+y^2)+4abxy=2ab(2xy-x^2-y^2)
=-2ab(x-y)^2<0
AB=ab(x^2+y^2)+(a^2+b^2)xy
>>348
1.接線lと弦PQの関係が分らない。
2.>lが長さが6のものを弦PQとする。 ←ここがまた意味不明。接線l?弦PQ?

lと平行な弦でも張ってあるの?
351348:04/07/28 23:53
すみません。
lが長さが6のものを弦PQとする。

lに平行で長さが6のものを弦PQとする。
肝心なところが抜けてました・・・
>>349
助かりました。ありがとうございます。
>>348
図を描いて、Aを取り、直角三角形を書きましょう。
原点からx軸プラス方向に3、さらにそこからy軸のプラス方向に4、そして斜辺5
(ついでに、斜辺を対角線とする、長方形もイメージしておく)
斜辺の傾きは、4/3。
これに直交するので、傾きは-3/4。
弦の長さ6だから、その半分の3の位置で交わるので、
さっきイメージした長方形を時計回りに回転して、
長方形の長さ3の一辺を、弦に重ねると(重なるんだなこれが)、
原点から交点までの距離が4(長方形の一辺)とわかる。
あとはとことん計算。
>>351
ま、普通はl〃PQよりPQの傾きわかって
円の方程式と連立させるんだが。

その先、少々楽な方法としては
点と直線の距離を利用する、つーのがあるな。

図を書くと明白になるが
求める直線は原点との距離4。
>>353
普通、原点中心の円に対する接線は
公式でやる方が早かろ。
接点が与えられてるわけだし。
>>348
マルチ発見。
以降放置で。
357348:04/07/29 10:51
すみませんでした。
353さん、354さん、355さんよくわかりました。
丁寧にありがとうございました。
358132人目の個数さん:04/07/29 12:42
高校数学
男子20人女子10人から少なくとも女子を一人含む三人を選ぶ組み合わせはいくつあるか?

【模範解答】
全体 30C3=4060
余事象:すべて男子 20C3=1140
答え:4060-1140=2920

【自分の解答】
女子を少なくとも1人選ぶから 10C1=10
残りの29人から2人選ぶから 29C2=4060
答え:10×4060=4060

【模範解答】が正しいのは理解できますが
【自分の解答】のどこが誤っているのか理解できません。教えてください。
359FeaturesOfTheGod ◆UdoWOLrsDM :04/07/29 12:47
Re:>358
例えば男子1人、女子2人から少なくとも女子1人を含む2人を選ぶ方法が何通りあるかという問題では、
当然三通りになるが、貴方の方法だと重複して数えるから四通りになってしまう。
>358
女子を1人選び 10C1=10 、 男子を2人選ぶ 20C2 = 190  190×10 =1900
女子を2人選び 10C1=45 、 男子を1人選ぶ 20C1 = 20    45×20 = 900
女子を3人選び 10C1=120 、 男子を0人選ぶ 20C0 = 1    120 × 1 = 120

答え:1900+900+120=2920
361132人目の素数さん:04/07/29 14:12
僊BCにおいて、次の等式が成り立つことを証明せよ。
cosA+cosB+cosC=1+4sinA/2sinB/2sinC/2

いろいろ考えてみたんですが分かりませんでした、どなたかお願いします。
362132人目の素数さん:04/07/29 14:27
>>361
分数、分子、分母、関数の引数、がどこからどこまでなのか確定するように
括弧を沢山使って表現してください。
363132人目の素数さん:04/07/29 14:39
>>362
uzai
>>361
sin α sin β=(-1/2){ cos (α + β) - cos (α - β) }
とか、
cos α sin β =(1/2){ sin (α + β) - sin (α - β) }
の公式が教科書に載ってると思うけど、
これを使って右辺の
4sin(A/2)sin(B/2)sin(C/2)
をゴリゴリと変形する。

そうすると、sinが4つ出てくる。
このとき、A+B+C=180度の関係を使うと、
それぞれがcosA,cosB,cosC,1に化けるはず。まあ、やってみれ。
365質問します。:04/07/29 16:03
(a-b-c+d)(a-b+c-d)をa-bでくくって計算すると、どーなりますか??
366132人目の素数さん:04/07/29 16:10
>>365
計算するとは?
367365:04/07/29 16:16
自分の解答。↓
a-b=Xとおく。
(X-c+d)(X+c-d)
=-(-X+c-d)(X+c-d)
c-d=Yとおく。
-(-X+Y)(X+Y)
=(X-Y)(X+Y)
…というような手順でOKですか??
368365:04/07/29 16:17
>>366
展開です!!
369132人目の素数さん:04/07/29 16:21
はじめまして。高校1年生です。
□□の部分を穴埋めする問題なのですが、よろしくお願いします。

放物線 C1:y=2(x2乗)をx軸方向に□□、y軸方向に□□だけ平行移動すると
放物線 C2:y=2(x2乗)−4x+□□となり、次にこれを直線 y=2に関して
対称移動すると、C3:y=−2(x2乗)+□□x−1となる。
370132人目の素数さん:04/07/29 16:27
>>367
> …というような手順でOKですか??

OK だけど、出来れば
(X-c+d)= (X-(c-d))
と考えられるようにしたほうがいいね
371365:04/07/29 16:30
>>370
なるほど!!ていねいにどうもありがとうございました!
372132人目の素数さん:04/07/29 17:43
(x^3+3x^2+2x+7)(x^3+2x^2-x+1)
工夫して展開できそうですができますか。普通にやるしかないですか。
>>369
まずな
C1: y=2x^2
をx軸方向にa,y軸方向にbだけ平行移動すると
C2: y=2x^2-4x+c になり
これを直線y=2に関して対称移動すると
C3:-2x^2+dx-1 になる 
と書け

でどこまでといたの?
まずC2を平方完成するとかいろいろあるだろ

>>372
ここで訊くぐらいならさっさと展開汁。
xyz空間内に半径と高さがともに1である直円柱があり、この直円柱の下底はxy平面上にあって、その中心は原点と一致している。
点P、点Qは点A(1,0,1)、点B(0,1,0)を出発し、それぞれ上底、下底の周上を同じ方向に線分PQの長さを変えないで1回転するものとする。
このとき線分PQが通過してできる曲面と、上底、下底で囲まれる立体の体積を求めよ。

まあ回転双曲面の問題なのですが、体積を求める際、立体の切口が円であることを証明する必要はあるのでしょうか?
あるのなら、どうすればいいのでしょうか。
375132人目の素数さん:04/07/29 18:38
>>374
問題書く暇あったら自分でとけ
376132人目の素数さん:04/07/29 18:43
漸化式が分かりません。例えば

a1=1、an+1=1+2anによって定義される数列の一般項を求めよ


という問題で


an+1=2an+1…@

に対してan+1とanをともにαで置き換えた方程式

α=2α+1…A

で辺々引くと

an+1−α=2(an−α)…B

と書いてあるんですが、なんでA式で引くんでしょうか?A式はどこから出て来たんでしょうか?参考書の解説が不丁寧でイマイチ釈然としないので誰か詳しく教えて下さいm(._.)m
377369:04/07/29 18:44
>>373さん
ありがとうございます!!

「x軸方向にa」までしか解けませんでした・・・。
C2を平方完成して、C2の頂点 (1,Q-2)は求められたんですが、
Qが邪魔で解り辛くて先に進められませんでした。
対称移動する前のグラフの頂点のY座標をQとして、対称移動した後の
Y座標をどう表したらいいのか解りません。

よろしくお願いします。
378132人目の素数さん:04/07/29 18:46
>>375
お前は解けないんだろ?
イ ラ ネ (・△・)
379375:04/07/29 18:49
>>374
ないんじゃない?体積は1・1・π×1=πかな?ちらっと読んだだけだから間違ってるかもしれないけど。
380376:04/07/29 18:51
>>379ですが名前間違えました。
>>376
漸化式というものはそのままでは分からんから
変形する必要がある
そしてどのように変形するかだが
Aを方程式を漸化式の特化方程式という
これで具具るなりしてみろ
これは基礎だから
382372:04/07/29 18:52
>>372
は、x^3でくくればいいですか?そうするとどうなりますか。
スマン
何を思ったから
特化→特性
384376:04/07/29 19:01
>>381
ありがとうございます。ですが携帯からなので出来たら変形まで教えて頂けるとありがたいのですが。
>>376
変形もくそもない。
たまたまα=2α+1って式を作って引き算したらきれいな形になったってだけの話。
>>377
この問いは全て頂点で考えるからどの式も平方完成させておく

C2:y=2(x-1)^2-2+c より頂点(1,c-2)
C3:y=-2{x-(d/4)}^2+(d^2/8)-1 より頂点(d/4,(d^2/8)-1)

まず分かるのは
a=1
b=c-2
次にC3はC2をy=2を対称軸に移動させたものだから
頂点のx座標は変わらない←これがポイント

すなわちd/4=1
∴d=4
ゆえにC3の頂点は(1,1)
これをy=2を対称軸に移動させると
頂点は(1,3)つまりC2の頂点になるから
c-2=3
∴c=5

またb=c-2
∴b=3
387132人目の素数さん:04/07/29 19:10
>>377
x方向にa、y方向にb移動したら、式C1は
(y-b) = 2 * (x-a) ^2
これを y= で書き直したのと、
C2を見比べると、なにか見えてこない?

y=2の直線は、y軸の2を通る横一直線。図を書くと分かるでしょう。
>>376
むしろ(an+1-x)=r*(an-x)の方が先にあると考えた方がいい。これを分解して
変形すると
an+1=r*an-r*x+x.
これがan+1=r*an+dに等しくするには-r*x+x=dとなるようなxを探せばよい。
389132人目の素数さん:04/07/29 19:22
>388
何度も質問してすいませんが*←これはどういう意味なんでしょうか?
390132人目の素数さん:04/07/29 19:23
>>389
アナルだろ?
>>390
              ∩
             | |
             | |
             | |
             | |
       ∧_∧  | |    / ̄ ̄ ̄ ̄ ̄ ̄ ̄ ̄ ̄ ̄ ̄
      ( ´Д`)//  < エッチなのはいけないと思います!!
      /     /    \___________
     / /|    /
  __| | .|    | __
  \   ̄ ̄ ̄ ̄ ̄   \
  ||\            \
  ||\|| ̄ ̄ ̄ ̄ ̄ ̄ ̄|| ̄
  ||  || ̄ ̄ ̄ ̄ ̄ ̄ ̄||
     .||              ||
>>389
なにー!そうか、すまん。
r*xは「rかけるx」の意味
393132人目の素数さん:04/07/29 19:33
f(x)=(2x+1 (-1≦x≦0)
(-2x+1 (0≦x≦1) のように定義された関数f(x)についてY=(fоf)(x)・・・合成関数のグラフを書け

  よろしくお願いします。
394369=377:04/07/29 19:37
>>386-387
ありがとうございます!
教えていただいた通りにやってみたらできました!!
「頂点のx座標は変わらない」っていうことを見落としていました。
それに式をうまく利用できずにいました。
本当にありがとうございました!
解けたら気がラクになって、また頑張る気力が湧いてきました。

これから度々お世話になるかもしれませんが
その時はどうぞよろしくお願いします。
395132人目の素数さん:04/07/29 19:40
378 132人目の素数さん New! 04/07/29 18:46
>>375
お前は解けないんだろ?
イ ラ ネ (・△・)
396132人目の素数さん:04/07/29 20:03
>>388
an+1−x=r(an−x)が先にあると考えるんですか?しかしそれだと最初にその式がどうやって出て来たんだってことになって>>376でAの式がどこから出て来たか分からないのと言ったのとたいして変わらないような…。

すいません、わがまま言って。結局のところ式を整えるためにA式を引いたって考えとけばいいんでしょうかね?まぁ分からなければ予想→証明の方法か階差数列使っても解けるわけですし。どうもみなさんありがとうございました。
397132人目の素数さん:04/07/29 20:05
>>374

結局、平面 $z = k (0 < k < 1)$ で切断したときの切り口の面積を求めると
きに、円であることは言うしかないが、証明をするほどのことはない。あと
は、定積分の問題。
>>396
漸化式an+1=2an+1みたいな奴は等差数列を表すわけでもないし等比数列
を表すわけでもない。でも、高校で一般項を求める公式があるのは
等差数列An+1=An+dと等比数列Bn+1=r*Bnだけ。
だからan+1=r*an+dみたいな奴もどうにか変形して等差数列か等比数列を
表す漸化式に出来ないかと考える。まあ、等比数列に変形する方が
簡単そうだしそのように変形するには(an+1-x)=r*(an-x)と置いてみる。
つまりan+1=r*an-r*x+x。
これをan+1=r*an+dと等しくするには-r*x+x=dであればよい。
399132人目の素数さん:04/07/29 21:20
>>393 お願いします。
>>393
-1≦2x+1≦0 ⇔ -1≦x≦-1/2 とき
(fof)(x)=2(2x+1)+1
0≦2x+1≦1 ⇔ -1/2≦x≦0 とき
(fof)(x)=-2(2x+1)+1
-1≦-2x+1≦0 ⇔ 1/2≦x≦1 とき
(fof)(x)=2(-2x+1)+1
0≦-2x+1≦1 ⇔ 0≦x≦1/2 とき
(fof)(x)=-2(-2x+1)+1
401336:04/07/29 21:59
すいません、>>336ですけどe^x>1+x/(1!)+(x^2)/(2!)+...+(x^n)/(n!)から
lim(x→∞)(x^k)e^(-x)=0を証明する何かいい案はありませんか
402DQNにも数学オシエテ:04/07/29 22:20
他スレでも質問したのですがやっぱりわかりません…
x,yについての二次方程式
x^2+2xy-3y^2+8x+a=0
のグラフが二直線を表すような定数aの値を求めよ。
できれば答えまでの式をかいていただけないでしょうか?
403132人目の素数さん:04/07/29 22:21
>>401
あるません
>>402
自らマルチポストを晒す必要もなかろうて ブブブ
>401
x^k e^(-x) = x^k / e^x
≦ x^k /{ 1+x+ … +(1/m! ) x^m } = 1 / { 1/ x^k+x/ x^k+ … +(1/m! ) x^(m-k) }
≦ 1/(1/m! ) x^(m-k) ---> 0

分母の正の項を一つを除いてより小さい 0 で置き換えて得られた不等式。
406132人目の素数さん:04/07/29 22:27
>>405
随分まんどくさい事しるな。
407336:04/07/29 22:32
>>405
そうか!それに気づかなかった!
はさみうちの原理とか使うものだと決め付けてました。
どうもありがとうございました!
入試会場で解けるようがんばります。
>>406
他にも方法があるんですか??
408DQNにも数学オシエテ:04/07/29 22:35
本当に困ってます
どうかお願いします。
>>408
氏ね。
410132人目の素数さん:04/07/29 22:37
>>400 理由を教えてください・・・・
411336:04/07/29 22:37
すいません。>>405もはさみうちの原理でしたね。
>>410
理由も糞も無かろう。
413132人目の素数さん:04/07/29 22:40
>>406
e^x > x^(k+1)/(k+1)! で十分。
414336:04/07/29 22:43
>>413
ああ、そうだった!
1+x/(1!)+(x^2)/(2!)+...+(x^n)/(n!)でnはなんでもいいんだった!
おれは何をやってたんだ!
どうも、ありがとうございました!
415数学の大森 ◆Ho3ICS2EdA :04/07/29 23:11
1,2,3,…,nの順列a1,a2,a3,…,anのうちai≦i+1(i=1,2,3,…,n)を満たすものの個数を求めよ。

まったく、分かりません。お助けあ〜れ。。
a_1から順に決めることにすると、毎回2つずつの選択があるわけだな。
>>410
-1≦x≦-1/2 とき
f(x)=2x+1
-1≦f(x)≦0 だから
f(f(x))=2f(x)+1

-1/2≦x≦0 とき
f(x)=2x+1
0≦f(x)≦1 だから
f(f(x))=-2f(x)+1

以下同様
aiの候補i+1個の内、
1~iの中のi-1個はa(i-1)までに選択済と考える
419数学の大森 ◆Ho3ICS2EdA :04/07/30 01:00
>>416>>418
ありがとうございます、もう少しヒントください。。。
まず、1,2,3,…,nの順列a1,a2,a3,…,anから意味が分かりませんヌ。


>>419
馬鹿、アホ、マヌケの順列 アホ、馬鹿、マヌケ
5^(1-2x)=2^(2x+1) の解が分かりません。

両辺に常用対数を取ってみましたが上手くいきません。
簡単な計算問題かとは思いますが、解法を教えてください。
422132人目の素数さん:04/07/30 01:14
(x−y)^2+yz-xz
因数分解してください。単純にzでくくって終了でokなのかな?
>>415 (数学の大森 ◆Ho3ICS2EdA)
例えば1,2,3,4の順列とは次のものである。
{1,2,3,4},{1,2,4,3},{1,3,2,4},{1,3,4,2},{1,4,2,3},{1,4,3,2},{2,1,3,4},{2,1,4,3},
{2,3,1,4},{2,3,4,1},{2,4,1,3},{2,4,3,1},{3,1,2,4},{3,1,4,2},{3,2,1,4},{3,2,4,1},
{3,4,1,2},{3,4,2,1},{4,1,2,3},{4,1,3,2},{4,2,1,3},{4,2,3,1},{4,3,1,2},{4,3,2,1}

>>421
5^(1-2x)=2^(2x+1)⇔(1-2x)log(5)=(2x+1)log(2)⇔2{log(5)+log(2)}x={log(5)-log(2)}
⇔x={log(5)-log(2)}/2{log(5)+log(2)}
対数の底はご自由に
>>421
「解」つーのはxの値のことだぞ。
だったら、どうすれば良いかすぐわかるべ?
>>422
zでくくったら新しい共通因数が見えてこないか?
>>422
そう思うならそうやれば?

>>421
2^(2x+1)=4*2^(2x-1)として再度挑戦。
>>422
それは因数分解とは言わない。その式は因数分解できない。
因数分解しろという問題ならば、問題を写し間違えてないか確認せよ。
>>427
できるよ。
>>457
ハァ?バカジャネェ?
(x-y)(x-y-z)
430429:04/07/30 01:21
おっと>>427
さあ、457でどういうレスをつけようかな。
432132人目の素数さん:04/07/30 01:23
(x-y)^2-z(x-y)
共通因数はあるにも分からない…
433425:04/07/30 01:25
>>432
そこまでやったら終わったも同然。
x-y=Aとおいたりするのは煩雑なんで嫌いだが。
434132人目の素数さん:04/07/30 01:31
>>433
ありがとうございました!!なんで>429になるのかわかりました。x-y=Aの発想がないと解けませんでした。
435421:04/07/30 01:45
指数を2x-1に統一することはできましたがそれから先がさっぱりです。
再び両辺対数とってみたりしたけどてんで駄目でした
>>435
>>423は読んだかい?
437421:04/07/30 01:57
携帯でこのスレ見てるので見逃してしまいました。。。

どうやら自分が解けなかった原因はlogを外そうと躍起になってたことのようです。
演習不足ですね(´Д`;)
438132人目の素数さん:04/07/30 02:31
>>417 あんがと!
2次以上の整式f(x)を(x-a)^2で割った時の余りを
f(a)とf'(a)で表せ。

f(x)=(x-a)^2*Q(x)+ax+b とおくと微分して
f'(x)=2(x-a)*Q(x)+Q'(x)*(x-a)^2+a

f(a)=a^2+b
f'(a)=a

この二式からb=f(a)-a^2=f(a)-{f'(a)}^2を代入して
ax+b=f'(a)*x+f(a)-{f'(a)}^2
=f'(a){x-f'(a)}+f(a)

でよかですか?
よかばい
>>440
dクス
442132人目の素数さん:04/07/30 19:42
次の和を求めよ。
(1) 1+3*3+5*3^2+..............+(2n-1)*3^(n-1)
(2) (1/2)+(4/4)+(7/8).............(3n-2)/2^n

手も足も出ません。
お願いします。
443132人目の素数さん:04/07/30 20:20
>>442
(1)S(n)=1+3*3+5*3^2+..............+(2n-1)*3^(n-1)

としたとき、まず1, 3, 5, ・・・と増加する部分を何とかする。

S(n) - 3*S(n-1) = 1+3*3+5*3^2+..............+(2n-1)*3^(n-1)
             -1*3+3*3^2+..............+(2n-3)*3^(n-1)
          = 1+2*3+2*3^2+..............+2*3^(n-1)
          = 2*(1+3+3^2+・・・・・・+3^(n-1))-1
          = 2*(3^n-1)/(3-1) - 1
          = 3^n-2 ■

(2)S(n)=(1/2)+(4/4)+(7/8).............(3n-2)/2^n

としたとき、同じように

S(n) - (1/2)*S(n-1) = (1/2)+(4/4)+(7/8).............(3n-2)/2^n
                  -(1/4)+(4/8).............(3n-5)/2^n
             = (1/2)+(3/4)+(3/8).............3/2^n
             = 3 * ((1/2)+(1/4)+(1/8)+・・・+(1/2^n)) - 1
             = 3 * (1/2) * (1-*(1/2^n)) / (1-1/2) - 1
             = 2 - 3/2^n ■
444132人目の素数さん:04/07/30 20:22
箱に二枚のコインが入っています。一枚のコインは両面が表模様で、
もう一枚は片面が表模様でもう片面が裏模様です。箱からランダムに
コインを一枚選んで片面だけを見るとします。
もしその片面が表模様の場合、その裏面も表模様である確率は?

これお願いします。
445132人目の素数さん:04/07/30 20:28
>444
2/3
>>445
ありがとうございました。
447ひよこ名無しさん:04/07/30 20:39
[(-1)^n-1+1]/2 − [(-1)^n-2+1]/2

=(-1)^n-1

ってどういうことでしょうか?
分かりやすく教えて下さい。

[ (-1)^n-1 - (-1)~n-2 ] /2

ってなるとこまでは分かるんですが。
数学科にいきたいのですが、
東京大学にするか京都大学にするかで迷ってます。
数学大好きです。

どちらがオススメですか?
東大生の方、京大生の方よろしくお願いします
450132人目の素数さん:04/07/30 23:32
>>339の問題で、(2)のまた、定数mは?って言うところの回答の導き方がよく分からないんですけど、
誰か詳しく説明お願いします。
>>450
こっちは見た?>>346
452132人目の素数さん:04/07/30 23:47
見たんですけど、「・・・また」ってところから、よく分からなかったんで。
もう少し説明を加えてほしいんですが。
453346 (=451):04/07/30 23:57
>>452
S1=(3/6)(3-1)^3
S2=(1/6){m^2-10m+13}^(3/2)だから
S2=S1/3
⇔(1/6){m^2-10m+13}^(3/2)=(1/6)(3-1)^3 (代入)
⇔[{m^2-10m+13}^(3/2)]^2={(3-1)^3}^2 (両辺2乗)
⇔{m^2-10m+13}^3=2^6
⇔[{m^2-10m+13}^3]^(1/3)=(2^6)^(1/3) (両辺1/3乗)
⇔m^2-10m+13=2^2⇔m^2-10m+9=0⇔(m-1)(m-9)=0
⇔m=1,9 これで分かるか?
454132人目の素数さん:04/07/31 00:05
『同様に確からしい』っていう意味を教えてみろ、うんこマン。
455132人目の素数さん:04/07/31 00:06
>>453
大変分かりやすかったです。
本当に有難うございました。
>>449
灯台に行って、兄弟に聞け。
>>443
遅くなってすいませんでした!
ありがとうございました。
458132人目の素数さん:04/07/31 05:33
>>449
東大の方が受かりやすい
>>449
専用スレ池や
>>439
ほっとこうかとも思ったが、一応書いとこう。もう見てないかもしれないが・・・。

>2次以上の整式f(x)を(x-a)^2で割った時の余りを
>f(a)とf'(a)で表せ。
>
>f(x)=(x-a)^2*Q(x)+ax+b とおくと微分して・・・・

余りはax+bじゃなく、bx+cとでも置いてとき直すことを
お勧めする(www
>>460
いやいやdクス

代入するのがaなのに
ax+bって置いたらダメだなw
462132人目の素数さん:04/07/31 23:18
>>449

将来目指す分野にもよると思うが、候補が東大と京大でしかない理由は?
円x^2+y^2-8x-6y+20=0に点A(0,-2)から引いた接線と円との接点をBとするとき、
線分ABの長さを求めよ。
どなたか教えてください。
464132人目の素数さん:04/07/31 23:48
CA=CB+BA
BA=CA-CB=(CA^2-r^2)^.5
>>463
円の半径求めて
中心とAとの距離求めて
三平方使ってウマー。

間違っても接線の方程式出そうと思わないこと。
466463:04/08/01 00:19
解けました。ずっと接線の方程式で解こうとしてました・・・
解説ありがとうございます。
467393:04/08/01 01:25
>>400 >>417 やっぱ分かりません・・・合成関数における値域と定義域が分かってないような気がするのですが・・・
 詳しく教えてください。
>>467
Y=(fоf)(x)において
f(x)=tと置いた場合
Y=f(t)と表される、つーのはいいんか?

そこをきっちり押えとけば
xの範囲(定義域その1)からtの範囲(値域その1)が定まって
値域その1がYにおける定義域として扱える、と。
>>467
関数 f(x) と g(x) の合成 (gof)(x) が定義できるためにはf(x) の値域が g(x) の定義域に含まれていないといけません。
そして (gof)(x) の定義域は f(x) の定義域になり、(gof)(x) の値域は g(x) の定義域を f(x) の値域に限定したときの g(x) の値域となります。
両方が f(x) の場合は f(x) の値域が f(x) の定義域に含まれていないといけません。
この問題の場合は f(x) の定義域と値域は両方とも -1 以上 1 以下 という範囲で一致しているので(fof)(x) の定義域と値域はともに -1 以上 1 以下の範囲となります。
そしてこのことはこの問題を解く際にはあまり関係はありません。

-1≦x≦1 であるような任意の x に対して
 -1≦x≦0 ならば f(x)=2x+1 ,
 0≦x≦1 ならば f(x)=-2x+1
である、というのが問題文の条件です。したがって
-1≦f(x)≦1 であるような f(x) について
 -1≦f(x)≦0 ならば f(f(x))=2f(x)+1
 0≦f(x)≦1 ならば f(f(x))=-2f(x)+1
が成り立ちます。

一体どの部分がわからないのでしょうか?
470132人目の素数さん:04/08/01 02:27
>>469 詳しくありがとうございます・・・
    >>-1≦f(x)≦0 ならば f(f(x))=2f(x)+1
       0≦f(x)≦1 ならば f(f(x))=-2f(x)+1  ここが全然分からないんですが涙・・・
471132人目の素数さん:04/08/01 06:56
∫[0→π]log(a^2-2acosx+1)dxを求めよ
教えてください
朝っぱらからすみません、
「48と52の最小公倍数の約数は全部で何個あるか」
という問題があり、なぜ20になるのか理解できません。
説明が端折ってあって分からないので、もし宜しかったら、
ご説明お願いします。
>472 朝っぱらから勉強ご苦労様(w

48=2*2*2*2*3
52=2*2*13
==>最小公倍数は2^4*3*13

これの約数の個数は、
 2^4からの5通り、
 3からの2通り、
 13からの2通り
を掛け合わせて,20(=5*2*2)。

474472:04/08/01 08:52
>>473さん
うはっ本当だ!!
とても分かりやすく、理解できました。
ありがとうございます(´▽`)
>>470
「-1≦x≦0 を満たすような任意の数 x に対して f(x)=2x+1 が成り立つ」というのが仮定の前半です。
「-1≦y≦0 を満たすような任意の数 y に対して f(y)=2y+1 が成り立つ」と言っても全く同じ内容の命題です。
「-1≦t≦0 を満たすような任意の数 t に対して f(t)=2t+1 が成り立つ」でも同じです。
命題の中に含まれている文字が変わっても命題の表す内容が全く同じであることを実感してください。
つまり、「-1 以上で 0 以下であるような数を任意に持ってくれば、その数は後半の式を満たす」という内容の命題です。
ただ、その任意の数をなにかの文字で表しておかないと命題の後半が表現しにくいので x や y や t という文字で表してるだけ。
で、f(x) が -1≦f(x)≦0 という条件を満たしていればこの仮定より f(x) は後半の式を満たすわけです。
すなわち f(f(x))=2f(x)+1
476132人目の素数さん:04/08/01 20:39
1/tan(-a/2)  →  tan(-(π-a)/2)  
の変形の仕方が分かりません。教えてください。
お願いします。
477ポラリスを見失った:04/08/01 21:02
an=3n-2,bn=4n+1,cn=7n (n=1,2,…)で定義される3つの数列{an},{bn},{cn}のどれにも現れる値の内、
1000以下となるものの個数を求めよ。
お願いします、ヨン様。
478132人目の素数さん:04/08/01 21:20
>477
最初に{bn}と{cn}を比べる(公差が大きいから)
すると最初に両方に現れる値は21。これは{an}にはあらわれない。
{bn}と{cn}の公差の最小公倍数は28だから、21に28を加えると49。
これは{an}にも現れる。{an}{bn}{cn}の公差の最小公倍数は84。
だから、3つの数列{an},{bn},{cn}のどれにも現れる値 dn=84m-35 (m=1,2,…) とあらわせられる。
1000以下でもっとも1000に近いのは m=12 の時。よって12個。
479訂正:04/08/01 21:22
>477
最初に{bn}と{cn}を比べる(公差が大きいから)
すると最初に両方に現れる値は21。これは{an}にはあらわれない。
{bn}と{cn}の公差の最小公倍数は28だから、21に28を加えて49。
これは{an}にも現れる。{an}{bn}{cn}の公差の最小公倍数は84。
だから、3つの数列{an},{bn},{cn}のどれにも現れる値 dm=84m-35 (m=1,2,…) とあらわせられる。
1000以下でもっとも1000に近いのは m=12 の時。よって12個。
cnから、an,bnとcnの値が共通するのは、an,bnがそれぞれ7の倍数の時だから、
a∩c_1=7=a_3、a∩c_n=21n-14となり、
b∩c_1=21=b_5、b∩c_n=28n-7となる。
a∩c_n=21(n-3)+49,b∩c_n=28(n-2)+49とすると、
一致するのは21(m-3)=28(n-2)を満たすm,nの組のときと分かる。
3(m-3)=4(n-2)であるので、m=3+4k,n=2+3kであり、
a∩b∩c_n=84(n-1)+49=84n-35
これが1000以下にある個数であるから、
84n-35≦1000をとけばよい。
84n≦1035
n≦12.・・・
これを満たすnの数は12となる。
481132人目の素数さん:04/08/01 22:02
>>475 やっと分かりました・・・ありがとうございました。感謝感謝。 >>468さんもありがとう!
482477:04/08/01 23:04
>>478 >>479 >>480
ありがとう!ありがとうございます!!
完璧に理解できました!夏休みなんで学校に質問しに行けないから、助かりました!!
483また、ポラリスを見失った:04/08/01 23:06
pは素数,m,nは正の整数でm<nとする。
mとnの間にあって、pを分母とする既約分数の総和Sを求めよ。
お願いします。。
tが任意の実数値をとって変化するとき、
円x^2+y^2-2tx+(t+1)y=0の中心の軌跡を求めよ。
どなたかお願いします。
>>483
pは素数 というのがポイントかも。
mからnまでにある既約分数は、
(mp+1)/p〜np/p-(m+1)-(m+2)-・・・-nだから、
Σ[k=mp+1〜np]k/p - Σ[k=m+1〜n]k
=(np+1)n/2-m(mp+1)/2 - (n+1)n/2 + m(m+1)/2
=(p-1)(n^2)/2-(p-1)(m^2)/2
=(m+n)(m-n)(p-1)/2
486132人目の素数さん:04/08/01 23:25
>>484
(x-a)^2+(y-b)^2=c の形にする。
すると円の中心の座標が求まりtを消去する。
答えは y=(x+1)/2 になるはず。
487132人目の素数さん:04/08/01 23:28
>>484
まず、(x-a)^2 + (y-b)^2 = r^2 の形にする。

左辺 = x^2-2tx+y^2+(t+1)y
    = (x-t)^2 - t^2 + (y+(t+1)/2)^2 - (t+1)^2/4

よって、円の式は

 (x-t)^2 + (y+(t+1)/2)^2 = t^2 + (t+1)^2/4

となる。右辺は使わないので整頓しない。
次に円の中心座標を (X, Y) として、X, Y を t で媒介変数表示する。

 X = t
 Y = (t+1)/2

よって、円の中心の奇跡は

 Y = (X+1)/2 ■
488132人目の素数さん:04/08/01 23:40
思ったんだけど、きちんと全部書く必要なくない?
やり方と答えくらいでいいような・・・
自分でやらせようze?
489484:04/08/01 23:42
>>486
>>487
やっとやり方がわかりました。どうもありがとうございます。
でもそれだとY=-((X+1)/2)になりませんか?
490486:04/08/01 23:45
>>489
そのとおり。
ミスりました_| ̄|○
たぶん487も俺につられたハズ・・・
>>489
間違えた。

 Y = -(t+1)/2

だから、

 Y = -((X+1)/2)

でした。失礼しました。
492484:04/08/01 23:49
>>490
>>491
そうでしたか。ありがとうございました。
>>488
回答者の側も、自分で解いてきちんと全部書くことで回答者自身の勉強を兼ねている場合もあるのです。
人に教えることが有効な勉強方法のひとつであるということもありますので。人に教えて自分の勉強にもなって一石二鳥。
本当に質問者のことを考えて、質問者が自分で解けるための力を付けることの手助けをしたいと考えている人は
きちんと全部書いたりはせずに自力ですべきところを残した形で適切なアドバイスだけをするでしょう。
現にそういった形で質問に答えている人も多いです。
494488:04/08/02 00:18
>>493
そうですね。
レスありがとうございました。
495 ◆nsUc5NkYCc :04/08/02 02:38
∫(x〜∞) {1/√(x^2+1)}dx

がどうやっても解けないです。。。
部分積分でも x=tanθの置換積分でもうまくいかず・・・
どなたかよろしくお願いします。
「x〜∞」というのがどういう意味か知らないが、1/√(x^2+1)の
原始関数はArcsinh(x)すなわちlog(x+(√x^2+1))と同値だが。
しかもx=tanθで解ける。
497 ◆nsUc5NkYCc :04/08/02 03:16
すいません、問題の勘違いでした。(ホントにすいませんっ!!)

・・・(x〜∞)は積分範囲のことです。っていうか逝ってきます・・・。
何が起きたのやら
499132人目の素数さん:04/08/02 03:40
〜だけでは足りないという意味で、
よく「〜は必要条件だが、十分条件ではない」って言い回しを聞きますが、
それを言うなら「〜は必要条件だが、必要十分条件ではない」ではないでしょうか?
>>499
どちらでも同じ意味です。
必要十分条件ってのは必要条件であり、かつ十分条件である ということです。
したがって「必要条件だが必要十分条件でない」というのは「必要条件であるが、(必要条件であり、かつ十分条件である)ではない」ということですが
必要条件という言葉を二回も使った冗長な言い回しで意味を掴み取りにくくしているだけなので普通は
「必要条件だが十分条件でない」という簡潔な言い回しが通常は好まれます。

「必要条件であり、かつ十分条件でもある」「必要条件ではあるが十分条件ではない」
「必要条件ではないが十分条件である」「必要条件でなく、かつ十分条件でもない」
これら4つのうち最初のものはよく用いるので「必要十分条件である」というふうに省略して言ってるってだけのことですよ。
501132人目の素数さん:04/08/02 07:24
微分の逆演算F(x)=∫f(x)dxを計算してf(x)の面積が求まる意味がわからないんですけど、
これって簡単に分かることですか?
教科書とか読んでもなんかしっくりこないのですが。
たとえば、f(x)>0のx=a〜bまでの面積を求めるときに、
求める面積=F(b)-F(a)
みたいに微分の逆の演算を計算する理由がわからないんです。
だって微分っていうのは接線の傾きを求めるってことで、
これがなんで面積と関係あるの?というわけです。
これをあえて一言で言うとしたらなんでなんでしょうか?
>>501
微分は速度を求めること。積分は面積を求めること。
その2つの関係は
http://village.infoweb.ne.jp/~fwgj3832/kmath240.html
>>502
微分と積分というのは計算方法(定義?)が独立にあって、
これらはたまたま逆の演算として関係してるってことですか。
それってすごいことだったんですね。
これでだいたい分かりました。ありがとうございました。
2次方程式 x^2+(m+1)x+2m-1=0 の2つの解が整数になるように整数mを求めよ。
という問題なんですが、チャートとかで調べてみたんですけどぜんぜんわかりませんでした。
おねがいします。
初カキコ

>>504
分かりにくかったらスマソ

解をa,bとする。(a,b : 整数)
(x-a)(x-b)=0
x^2-(a+b)x+ab=0
上式とx^2+(m+1)x+2m-1=0 を比較。

1 : -(a+b) = m+1 --> m= -a-b-1
2 : ab = 2m-1

2式に1式を代入。

ab = 2(-a-b-1)-1 = -2a-2b-3
左辺を右辺へ
ab+2a+2b+3 = 0
ab+2a+2b+4-1 = 0
(a+2)(b+2) = 1
a,b が共に整数 から かけて1になるのは 1x1 と (-1)x(-1) の2つ
1x1 のとき
a+2=1 b+2=1 => a=b=-1
1式から -(-1-1)=m+1 m = 1
-1 x -1 のとき 式省略 m = 5

間違ってたらすみません。
506504:04/08/02 19:09
>>505
ありがとうございます。
やっと分かりました、分かりやすいレスありがとうございました。
507132人目の素数さん:04/08/02 19:11
お願いします

∫(sin^3 x)cosxdx
>>507
キャー、痴漢よーw
509132人目の素数さん:04/08/02 19:17
>>507
sinx = t と置いてみれ。
510FeaturesOfTheGod ◆UdoWOLrsDM :04/08/02 19:21
cos(x)dx=d(sin(x))
などと書いたらビックリするかも。
511132人目の素数さん:04/08/02 19:35
すいませんわかりません
痴漢?
>>511は本当に痴漢らしいよ。
513(初):04/08/02 19:39
2でも3でも割り切れない正の整数の全体を小さいものから順に並べてa1,a2,a3,…anとする。
与えられた正の整数nに対してam≦6nとなる最大のmを求めよ。
よろしくおねがいします。。
514132人目の素数さん:04/08/02 19:43
さっきの問題の答えが
1/8 cos2x+C
になったんですがいいでしょうか?
>>514
何がどうなってそうなったって?
516132人目の素数さん:04/08/02 19:53
sin20°sin40°sin80° の値を求めてください。
(積)→(和)の公式を使え とのことです。
途中計算が知りたいです。お願いします。

答えは8/√3 です。
>>516
20+40=60, 40+80=120, 20-80=-60, ...
518516:04/08/02 20:18
>>517
レスありがとうございます。
sin20°sin40°=-1/2(cos60°−cos20°)
となってcos20°が邪魔なのですが、どうすればいいのでしょうか。

あと、答えは√3/8でした、すいません。
519132人目の素数さん:04/08/02 21:34
>>515
半角、倍角の公式つかってsin^3を分割しました。
>>518
sin20°sin40°sin80°
=-1/2(cos60°−cos20°) sin80°
=(-1/4+1/2*cos20°)sin80°
=-1/4sin80°+1/2*cos20°sin80°
=-1/4sin80°+1/4(sin100°-sin-60°)
=-1/4sin80°+1/4sin100°-1/4sin-60°
=-1/4sin80°+1/4sin100°+√3/8
=-1/4sin80°+1/4sin(180°-80°)+√3/8
=-1/4sin80°+1/4sin80°+√3/8
=√3/8
>>518
517さんじゃないけど、違う方針で。「°」は省略します。
sin20sin40sin80=cos(90-20)cos(90-40)cos(90-80)=cos70cos50cos10=(1/2)(cos120+cos20)cos10
後は、頑張って自分でやってみて。必ずできるから。
522521:04/08/02 21:39
>>521
あ、既に解答が・・・。でも違う方針だから、まあいいか。
>>522
( ´゚,_」゚)ヒッシダナ
524132人目の素数さん:04/08/02 21:41
僕は中3凄く数学が好きで塾の先生に「中学の数学より高校の数学のほうが
かなり面白いから楽しみにしてろ」って言われてちょっと教えてもらいました。
三角比を教えてもらったんだけど、かなり面白かっったです。
一応自主学習で中学の内容は全部終えたんですけど、高校の数学に興味を持った
ので夏休みを機会に高校の数学を勉強してみようと思ってます。
そこで、皆さんのお勧めの数学の参考書を教えてください!0からのスタートなので
なるべく分かりやすさ重視でお願いします。
高校生のための質問スレなのに中学生が質問してごめんなさい。ここのスレに来てる
人が詳しそうだったので・・・。
525516:04/08/02 21:44
わかりました!どうもありがとうございました!
526132人目の素数さん:04/08/02 21:47
>>524
なら教科書がいいんでねの?(特に数研出版)
527132人目の素数さん:04/08/02 21:56
>>526
本屋で売ってますか?
528132人目の素数さん:04/08/02 22:14
529132人目の素数さん:04/08/02 22:14
大きめなとこなら置いてあるよ
教科書ガイドとかでもいいし
>>527
学校に卸してる業者なら売ってる。
>>527
注文取り寄せ。ネットでも探せるだろうけど。
532132人目の素数さん:04/08/02 22:16
教科書ガイドですか。中学のを確か使ったことあるけど確かに分かりやすかった
ような気がします。では明日ちょっと遠出して探してみます。
533532:04/08/02 22:20
一応明日大きい本屋行ってみる予定なので教科書も探してみます。もしなかったら
ガイドのほうを買います。皆さんありがとうございました!
534132人目の素数さん:04/08/02 22:52
>>524
数学の勉強も良いが、国語の勉強も頑張ろう。
適度に句読点をけよう。
どっかの奴らみたいに専用スレ立てなかったのは良かった。
536法政大:04/08/03 14:58
赤玉と白玉が入っている袋がある。玉の数は合わせて16個で、赤玉白玉
とも2個以上である。また、袋から2個を取り出すとき、2個とも赤玉である
確率は4分の1である。今、袋から3個を取り出すとき、2色とも出る確率を
Pとする。
(1)袋の中の赤玉の個数をXとして、PをXで表せ。
(2)Pの最大値と最小値を求めよ。

お願いします。
537花丸木:04/08/03 15:34
高校生らむーん。
>>536
「また、袋から2個を取り出すとき、2個とも赤玉である確率は4分の1である。」
これって今の問題と関係あるのか?
自分で出した回答とかじゃないの?

この文抜くと答え出るのだが。
(M・3・Y)の3乗は?
540132人目の素数さん:04/08/03 16:06
おいこらコヨタンどこだ!
>>538
赤玉+白玉の数は問題に書いてあるが、
赤玉・白球単独の数は書いてない。
それぞれの数を求めるためにその条件が必要なはず。
542541:04/08/03 17:27
あ、俺が寝ぼけていた。
541はスルーして。
543132人目の素数さん:04/08/03 17:59
x^2 exp( -x^2 ) を x→∞ の局限ではどうなりますか?
544132人目の素数さん:04/08/03 18:02
ロピタルでも使え.


つーかまず正しい漢字仕え.
545132人目の素数さん:04/08/03 18:07
>>544
ありがとうございました
546FeaturesOfTheGod ◆UdoWOLrsDM :04/08/03 18:30
Re:>544 お煎もな。
↑放置で。
548132人目の素数さん:04/08/03 18:54
NHK人間講座 数学の愛しかた ピーター・フランクル
http://www.nhk.or.jp/ningenkoza/200408/tue.html

今日(8/3)、午後10:25から放送。毎週火曜日。
549132人目の素数さん:04/08/03 19:34
Σ[N,i<j]f(i,j)
の展開の仕方を教えてください。
>>549
は?
551132人目の素数さん:04/08/03 19:49
Σ[i<j,N]f(i,j)
でした。
552132人目の素数さん:04/08/03 20:23
>>551
f が分からない。
あと、どう展開すればいいのか説明キボン
Σを2回使うんだったら、

Σ[i=0,N-1]Σ[j=i+1,N]f(i, j)

但し、0≦i < j≦N の場合。
554132人目の素数さん:04/08/03 20:40
Σ[i=1,N]f(i)
の場合は
f(1)+f(2)+f(3)......f(N)
になるけど、
Σ[i<j,N]f(i,j)
はどういう形になるか解らないのですが、という事です。
555132人目の素数さん:04/08/03 21:29
>>553
わかりました。ありがとうございました。
f(x)=(1/3)x^3-ax+5 が1≦xを満たす全てのxで増加になるための
定数aの範囲を求めよ

よろしこ
>>556
関数が、増加になる という言葉の意味がわかりません。
単調増加の誤植の可能性もあるので確かめてください。
>>557
問題通りなんでつが…
関数の増加ってその範囲における
接線の傾きが正って定義してるんじゃないの?
559132人目の素数さん:04/08/03 23:21
>>556
f(x)=(1/3)x^3-ax+5

f'(x) = x^2 - a

f(x)が 1≦x で短調増加なので、f'(x) が 1≦x で正あるいは0。

f'(1) = 1-a ≧ 0

∴a≦1 ■
>>559
f'(x) に1を代入するだけだったのか

dクス
>>560
ちなみに、これは f(x) が x≧0 で単調増加という性質を利用した解法だから、
他の関数の場合は要注意。
>>561
補足サンクス

f'(x) = x^2 - a =0と置くと
x=±√a
すなわちx=-√aで極大,x=√aで極小をとる
1≦xにおいて増加となるためには
x≦1の範囲で極小をとっておかなければならない
よってa≦1

と同じことでつね
極値をとらない場合もあるだろうが。
>>563
そうか
a>0とa=0とa<0で場合分けが必要か

a>0の時
f'(x) = x^2 - a =0と置くと
x=±√a
すなわちx=-√aで極大,x=√aで極小をとる
1≦xにおいて増加となるためには
x≦1の範囲で極小をとっておかなければならない
よってa≦1

a=0の時
f'(x)=x^2≧0 よってf(x)は単調増加より
当然、1≦xにおいても増加

a<0の時 
はどうすればいいでつか?
xが虚数解に…
場合分けなんていらない。
>>564
a<0の時は
x^2-a>0だからxの範囲に関わらず…
567566:04/08/04 01:39
つか、y=f'(x)のグラフでも書いて
aを色々変化させる、程度のこともやってないんか。
568132人目の素数さん:04/08/04 03:27
x+y+z=20(x,y,zは自然数)となる自然数の組(x,y,z)は171通りあるこの組のそれぞれについて積xyzを考え、
次にそうしてできた171個の数をすべて加えるとどうなるか?
569132人目の素数さん:04/08/04 04:55
>>568
26334
で?
570132人目の素数さん:04/08/04 04:57
26334=171*154
571132人目の素数さん:04/08/04 04:59
26334=171*154
154=2*7*11
2+7+11=20
572132人目の素数さん:04/08/04 05:09
おはようございます。教えて下さい。

 面積がPの台形ABCDがありまして、この台形の
 線分ABと線分DCの長さではなく、傾きが各々ab と dcとして、
 定義されています。

 この時、この台形の上底(AD)の長さが最も短くなるときの
 長さと、台形の高さを求めよ。 っていう問題なんですが、
 教えて下さい、すいません。
 
>>572
日本語が不明瞭。
問題文を「正確に」再掲せよ。

こっちで勝手に解釈しても
マトモな回答にはなるまい?
面積一定の台形の上底の最小値は0でないか?
575132人目の素数さん:04/08/04 05:30
>>573,574
お返事ありがとうございます。

但し書きがありまして、

「ただし高さの上限はhとする」 って感じです。
すいません、よろしくおねがいします。
576573:04/08/04 05:55
>>575
>問題文を「正確に」再掲せよ。
は無視かい。
「って感じです」ってナメてるんかい。

そもそも、>>572の文じゃ
問題として成立してないんだから
解けるわけなかろう。

どうせ、このままじゃ重要な情報を
小出しにして来るだろうからもう少し。
*質問者の学年
*出題された単元
等も付記せよ。
577132人目の素数さん:04/08/04 06:43
>>576
むりむり。マズは国語から勉強しなおさないと。
ムリムリ カタツムリよ
         ./     ,. '                    '、   ノ
 か ム ぜ お |   /                      ',  / は こ
 た リ っ 兄 :l.  /    ,   i :i、  |、           ゙, ノ'  っ う
 つ ム た ち ヽ./     /! ./l  lヽ :i ',           i`)  き な
 む リ .い .ゃ ./!   /|:! | ./ |  ! ヽ l ヽ           ! l  り っ
 り ム ム ん / | /!//''|l‐=/、 ! l ,ゝ- ‐‐ヽ、        | |  言 た
 よ リ .リ に (, l, :l.|: /_ ァテゝ、ヽ !   ヽテ = 、ヽ       ! l  わ ら
  !!.ム よ 説 l . :!|:!:|, i` .}{ i゙!  `   ´ }.{; 'ィヾ,.  ∧    :} /.  せ
.   リ . 明 ,ゝ ! ! :i  '' "´,      ‐'='' ´ | i`:}    / /  て
      は (.   / ',   `          i ヒ/  /ソ ゝ.  も
         > /人 | 、  ヤ‐ヽ       ,イ  l /!/  _ ゝ  ら
\      _ /  '   ':, ヽ.  ' ‐ '     ,/ /,r, |i' |'   ` ).  う
  'レ'⌒´         ゝ,  ` 、    ,. '  _// ‐‐ 、     ム  わ
              //    >:t' ,. '´ /'   _>-‐- 、,_ヽ
            // /ノ,._'´ 丿 iヽ    , '´      `
          /   - ' /-) _' i'  ノ‐ ' ヽ、 ' 
580132人目の素数さん:04/08/04 09:11
sin25°=aのとき、次の値をaで表せ。

cos25°
581132人目の素数さん:04/08/04 09:26
>>568
C(20+2,3+2)=26334
むしろ>>571がどういう意味なのか教えてほしい。。
>>580
√(1-a^2)
583132人目の素数さん:04/08/04 09:48
この円柱の表面積は?

円の半径3p 円柱の高さ4p 円周率はπで。

解説つきで是非よろしく
584132人目の素数さん:04/08/04 10:01
底面積
3かける3かけるπは9π
底はふたつあるので、9πかける2は18π
側面積
展開図をかいて、長方形として考える。
たては4、よこは底面の円周と同じになるので、3かける2かけるπは6π
つまり側面積は4かける6πは24π

18πたす24πは42π  こたえ42π平方センチメートル
585132人目の素数さん:04/08/04 10:05
ああスマソι間違えました
三角すい でした さしつかえなければもう一度おねがいします
1辺の長さ1の四面体の体積を求めよ

ベクトルで解くんですが、底面積が√3/4 までは求めたんですが、高さはどう求めるんですか?
正四面体の底面に対して垂直に(真上の方向から)見下ろすと、頂点は底面の正3角形の重心に一致する。
よって高さをhとすると、1^2 = h^2 + (√3/3)^2 ⇔ h=√6/3
>>586
正四面体ABCDで辺CDの中点をMとして三角形ABMについて考えてみよう。
実質的に587と同じことだが。
>587,588
ありがとうございます
590C:04/08/04 14:13
>>586
正四面体の高さ、底面積、体積は公式があるでしょ
591C:04/08/04 14:19
正四面体の一辺の長さを a とすると

高さh=(√6*a)/3
底面積S=(√3*a^2)/4
体積V=(√2*a^3)/12
>>590
そんな公式は習わない…
むしろ、その公式を導き出すことこそが出題されるかと。

蛇足だが、588の点名を使って四面体ABCMの2倍として求めるとか、
立方体ABCD-EFGHを考えるとADFGが正四面体になることを利用する
などの解法もある。
593132人目の素数さん:04/08/04 16:46
>>592

そう? 中学校辺りで習ったような気がするけど。
>>593
公立ではまず習わないと思われ、受験用に私立で教えるくらいでない?
そもそもこんなもん覚えるもんでもないしな
595あつみ:04/08/04 21:55
初めまして、『高校生のための数学質問スレ』と言う事で
分からない問題があるので質問させて頂きます。

『全ての自然数nについてn(n+1)は2の倍数であることを証明せよ』

という問題なんですがどうしても分かりません。
どなたか教えてください。(ぺこり
596132人目の素数さん:04/08/04 22:00
>>595
小学校からやり直せ。
597132人目の素数さん:04/08/04 22:04
>>595
nが偶数の時と奇数の時で場合分け
>>595
(@)
nが偶数の時
n+1は奇数である
偶数と奇数の積は偶数すなわち2の倍数である。
(A)
nが奇数の時
n+1は偶数である
奇数と偶数の積は偶数すなわち2の倍数である。

@,Aよりnが偶数の時も奇数の時も
すなわちnが自然数ならばn(n+1)は2の倍数
599132人目の素数さん:04/08/04 22:47
つぎのような問題をみたことがある人いませんか?スマートな
解き方を知っている人,もしくはスマートに解ける人がいたら
解き方を教えてください。

ぼくは三角形の面積に注目して,がりがり計算しましたが,
ノート5ページ分くらいになりました。しかし簡単に解く方法が
あるそうです。

正解かどうかは分かりませんが,ぼくの答えは

x+y=2r(2/Sin2θ + 1/Cosθ)

となりました。

■問題文■
大きさが 2θの角XOYがあり,辺OX上に点A,辺OY上に点Bをとり,
三角形OABに一定の半径rの円が内接しているようにする。ただし
0<θ<π/2 とする。OA=x,OB=y とおく。

■問■
θを一定としたとき,x+y の最小値を求めよ。
600あつみ:04/08/04 22:47
598さん>有難う御座います。
本当に自分でも頭悪いなぁと思うんですが分からなくて。
ご丁寧に有難う御座いました。

すいません
もう一つ質問してもいいでしょうか?

『全ての自然数nについて1+3+5+7…+(2n-1)=n2(nの2乗)が成り立つ事を説明せよ』

何度もすいません。
601FeaturesOfTheGod ◆UdoWOLrsDM :04/08/04 22:49
Re:>600 数学的帰納法か。
>>600
数学的帰納法による

(@)
n=1の時
左辺=2*1-1=1
右辺=1^2=1
よって成立する
(A)
n=kの時成立すると仮定しn=k+1の時
左辺=1+3+5+…+(2k-1)+(2k+1)
=k^2+2k+1
右辺=(k+1)^2=k^2+2k+1
よって成立する

@,Aより全ての自然数nに対して成立する。
603132人目の素数さん:04/08/04 23:30
高一です
お願いします。

y=x^2-k|x|+k^2の最小値m(k)を求めろ
また、-5≦k≦5におけるm(k)の最大値、最小値を求めろ

よろしくお願いします。
>>603
何をお願いするって?
605603:04/08/04 23:32
>>603のところに書いてある問題です

お願いします
問題をお願いするってどういう意味?
607132人目の素数さん:04/08/04 23:34
>>603
無視してOKだよ。
>>603
絶対値があったらまず場合分け。
たいていはこれでオケ。
609603:04/08/04 23:38
判らないんで教えてください。
困っているんです
610132人目の素数さん:04/08/04 23:41
>>599
内接する円の中心をPとする。
∠OAP=α、∠OBP=βとすると、∠OAB=2α、∠OBA=2β なので、
2θ+2α+2β=π より、β=π/2 - θ - α

Pから線分OAに下ろした垂線をPHとすると、PH=r より、
 OP*sinθ= r
 OP = r / sinθ
 OH = r cosθ/sinθ
同様に、
 AH = r cosα/sinα

Pから線分OAに下ろした垂線をPIとすると、
 OI = r cosθ/sinθ
 BI = r cosβ/sinβ

 x = OH + AH = r cosθ/sinθ+ r cosα/sinα
 y = OI + BI = r cosθ/sinθ+ r cosβ/sinβ
  = r cosθ/sinθ+ r cos(π/2 - θ - α)/sin(π/2 - θ - α)
  = r cosθ/sinθ+ r sin(θ + α)/cos(θ + α)

x+y = f(α) とすると、f(α) = 2r cosθ/sinθ + r cosα/sinα + r sin(θ+α)/cos(θ+α)
f'(α) = -r/sin^2(α) + r/cos^2(θ+α)
f'(α) = 0 のとき、題意より 0<α<π/2 なので、
 sinα = cos(θ+α)
     = sin(π/2-θ-α)
∴α=π/4-θ/2
f(α)=2r*(2/sin2θ+cosθ) が x+y の最大値 ■
611603:04/08/04 23:41
バカで困っているんです
教えてください
>>611
だから場合分けしろって

y=x^2-k|x|+k^2
(@)
x≧0の時
y=x^2-kx+k^2
(A)
x<0の時
y=x^2+kx+k^2
613132人目の素数さん:04/08/04 23:48
>>603
f(x)=x^2-k|x|+k^2 とする
f(1)とf(-1)を求める・・・値は変わらない

よって
f(x)=x^2-kx+k^2
が成り立つ。

後は軸の位置で場合分け

(i)
k≦0の時
m(k)=f(0)=k^2

(ii)
k≧0のとき
m(k)=f(k/2)=(3/4)k^2

∴m(k)=k^2    (k≦0)
     (3/4)k^2 (k≧0)

あってますかね?ここまで来て不安になってきた
>>613
手の込んだ釣りだなw
615132人目の素数さん:04/08/04 23:51
613続き

上より
  m(k)=k^2    (k≦0)
     (3/4)k^2 (k≧0)

のグラフを書くと、m(k)は、k=0で最小値0
                 k=-5で最大値25
616132人目の素数さん:04/08/04 23:54
>>603
f(x)=x^2-k|x|+k^2 とする。
(1)x≧0 のとき、
y=x^2-kx+k^2
= (x-k/2)^2 + (3/4)k^2
k≧0 ならば、最小値 f(k/2) = (3/4)k^2
k<0 ならば、最小値 f(0) = k^2

(2)x<0 のとき、
y=x^2+kx+k^2
= (x+k/2)^2 + (3/4)k^2
k≧0 ならば、最小値 f(0) = k^2
k<0 ならば、最小値 f(-k/2) = (3/4)k^2

-5≦k≦5 のとき、(1)と(2)の場合についてグラフを書けば、
最大値25、最小値0 であることが分かる。■
617132人目の素数さん:04/08/04 23:58
夏休みの宿題です。。。おながいします


a↑とb↑のなす角、(a↑+b↑)と(a↑−b↑)のなす角はともに60°であり、
b↑が単位ベクトルのとき、
a↑の大きさを求めなさい。
618599:04/08/05 00:15
もし考えてくれている人がいたら,ぼくは今日徹夜で待ってます。
619132人目の素数さん:04/08/05 00:17
620あつみ:04/08/05 00:17
602さん>どうも有り難う御座いました!
数学的帰納法を用いて証明せよ という問題だったので
苦手な私はチンプンカンプンで…。
本当にご丁寧に教えて下さって有難う御座いました。
621132人目の素数さん:04/08/05 00:44
age
>>618
三角形OABに一定の半径 rの円が内接しているから、θを一定とした時 O の対辺=c は一定で
x+y の最小値は点 O が A または Bに重なった時、つまり x または y がゼロの時に実現し、
x+y の最小値= c である。
x が直径である場合を考えれば、 O の対辺 c = r sin 2θ、よって x+y の最小値 = r sin 2θ

■問題文■
大きさが 2θの角XOYがあり,辺OX上に点A,辺OY上に点Bをとり,
三角形OABに一定の半径rの円が内接しているようにする。ただし
0<θ<π/2 とする。OA=x,OB=y とおく。

■問■
θを一定としたとき,x+y の最小値を求めよ。
623132人目の素数さん:04/08/05 01:06
組立除法の問題が分かりません。
f(x)=5x^3+2x^2-3x+4 を 2x+1で割ったときの商と余り をお願いします。
>>623
組み立て除法じゃなくて普通に筆算で割れば? 俺は未だに組み立て除法は
使えないが、それでも数学科は卒業したぞ。
625599:04/08/05 01:18
>>618
丁寧な解答,ありがとうございます。
解答をすべてノートに写して,いま吟味しているところです。

もう少し時間がかかりそうです。

>>622
スマートな解答,どうもありがとうございます。

x または y がゼロになると,三角形OAB という条件に反する気がしたのですが
いかがでしょうか?
円が内接であって円に内接ではない。
>>620
数学的帰納法の意味を分かってるよな?w

n=1の時成立
で、n=kが成立すると仮定してn=k+1も成立する (k=1つまりn=2の時)

n=2の時
n=3が成立

n=3の時
n=4が成立
…以下同 だから全てのnで成立するんだぞ
>>625
そう言えばそうだ、「最小値はない」が正解となる。下限は有って、x+y> r sin 2θ
629622:04/08/05 01:26
>>626
間違えた。指摘サンキュウ。
630599:04/08/05 02:01
>>610
ノートに写した解答を最後まで吟味してみましたが,最後の1行だけ
何故そうなるのか分かりません。

f(α) = 2r cosθ/sinθ + r cosα/sinα + r sin(θ+α)/cos(θ+α)

α = π/4-θ/2 より

f(π/4-θ/2) = 2r cosθ/sinθ + r cos(π/4-θ/2)/sin(π/4-θ/2) +
r sin(θ-π/4+θ/2)/cos(θ-π/4+2θ)

ここからどのようにして「f(α)=2r*(2/sin2θ+cosθ)」を導かれたのか,
それをもっと考えてみます。
631天然水:04/08/05 02:12
>>617
b↑が単位ベクトル ⇔ |b↑|=1
a↑とb↑のなす角が60°より、
(a↑・b↑)=|a↑||b↑|cos60°=(1/2)|a↑| …@
また、(a↑+b↑)と(a↑−b↑)のなす角が60°より、
 (a↑+b↑)・(a↑−b↑)=|a↑+b↑||a↑-b↑|cos60°=(1/2)|a↑+b↑||a↑-b↑| …A
 (a↑+b↑)・(a↑−b↑)=|a↑|^2 -1 だから
A⇔|a↑|^2 -1=(1/2)|a↑+b↑||a↑-b↑|
 ⇔2|a↑|^2 -2=|a↑+b↑||a↑-b↑|
これの両辺を二乗する。
(右辺)^2=|a↑+b↑|^2|a↑-b↑|^2=(|a↑|^2+1+|a↑|)(|a↑|^2+1-|a↑|) ←@を使った
={(|a↑|^2+1)^2 -|a↑|^2}=|a↑|^4 + |a↑|^2 +1

∴ (右辺)^2=(左辺)^2
⇔4{|a↑|^2-1}^2=|a↑|^4 + |a↑|^2 +1
⇔3|a↑|^4 - 9|a↑|^2 +3=0
⇔|a↑|^4 - 3|a↑|^2 +1=0
∴解の公式より
|a↑|^2 =(3±√5)/2=(6±2√5)/4={(√5±1)/2}^2
|a↑|>0 より、
|a↑|={(√5±1)/2}^2(1/2)=(√5±1)/2. …(答)
632天然水:04/08/05 02:54
>>623
 5 +2 −3 + 4 「-1/2」

 0 -5/2 1/4 11/8 
────────────────────────────────────
 5 -1/2 -11/4 [43/8]←余り
5/2 -1/4 -11/8 ←商

∴商 (5/2)x^2 -(1/4)x -11/8
 余り 43/8
>>623
組み立て除法なんて応用範囲狭すぎだから忘れれ。
普通に割り算した方が無難
634623:04/08/05 03:07
>>632
解答ありがとうございました、
>>624.633
ちょっと機会があったので挑戦してみたんです。
確かに普通にやったほうがいいですね。
635622:04/08/05 03:10
>>618 、 読み違えて悪かった。
>>610 は俺には読み取りにくいので別にやってみた。>>599 と一致する。

円に外接するからOA.OB,AB との接点をそれぞれのR,S,T とおき、OR=OB=a , AT=b , BT=c
とおくと、x=a+b , y=a+c , AB=b+c 、と書ける。
余弦定理で結びつけて、整理すると a { a + (b+c) }(1- cos2θ)= bc(1+cos2θ) 、となる。
bc - ma(b+c) = ma^2 ; m=(1- cos2θ)/(1+ cos2θ) , (b-ma)(c-ma)=2ma^2 ; 一定
b+c=k とおいて b c を座標とするグラフ考えると直線であり、これが
b=c >0 となる所でグラフ (b-ma)(c-ma)= 2ma^2 に接する時 k は最小となる。
よってまた x=y の時 x+y は最小で、円の中心を P とおくと、OP=r/sinθ 、OT=OP+r
OA=OB=OT/cosθ 、よって 2r(1+ 1/sinθ)/cosθ
>>599
もう寝ちゃったかな?610氏じゃないけど、>>610の訂正。
×Pから線分OAに下ろした垂線をPIとすると、
○Pから線分OBに下ろした垂線をPIとすると、
×x+y の最大値 ■
○x+y の最小値 ■

で、最後の計算だけど、俺がやったら、f(π/4-θ/2)=2r/{tanθ*(1-sinθ)}になった。
つまり、622氏のいう2r(1+ 1/sinθ)/cosθであって、
610氏の
×f(α)=2r*(2/sin2θ+cosθ)
○f(α)=2r*(2/sin2θ+1/cosθ)
って事だと思う。

♯ 610さん、勝手にいじくってゴメンナサイ。
637622:04/08/05 03:35
>>622 訂正
>(b-ma)(c-ma)=2ma^2 ; 一定
===> (b-ma)(c-ma)= ma^2 + m^2 a^2 = (1+m)ma^2 ; 一定
638599:04/08/05 04:10
>>636 さん
訂正,ありがとうございました。

いま新たな疑問と格闘しているのですが,それは >>610 の最後の微分をする所で

「f'(α) = 0 となるようなα の値が極小値になる」

という部分です。
また「f(π/4-θ/2)」の値についてですが,和積の公式を利用されたんでしょうか?

>>635
遅くまで分かりやすい解答,本当にありがとうございます。
面積の公式を利用せずに解く方法が2通りもあったなんて,感動しています。


いまは4行目で苦戦していますが,そこさえ切り抜けられたら何とかなりそうです。
639636:04/08/05 04:29
>>638
あ、今気付いたんだけど、>>630
>f(π/4-θ/2) = 2r cosθ/sinθ + r cos(π/4-θ/2)/sin(π/4-θ/2) + r sin(θ-π/4+θ/2)/cos(θ-π/4+2θ)
は3項目が間違ってない?それで計算が合わないとか?
俺は本当にてきとーに計算したw(積→和の公式は1回だけ使ったかな?)。
恥ずかしながら、とりあえず書いてみると、
f(π/4-θ/2) = 2r cosθ/sinθ + r cos(π/4-θ/2)/sin(π/4-θ/2) + r sin(θ+π/4-θ/2)/cos(θ+π/4-θ/2)
= 2r{1/tanθ + (1/2)sin(π/4+θ/2)/cos(π/4+θ/2) + (1/2)cos(π/4-θ/2)/sin(π/4-θ/2)}
= 2r[1/tanθ + (1/2){sin(π/4+θ/2)sin(π/4-θ/2) + cos(π/4+θ/2)cos(π/4-θ/2)}/{cos(π/4+θ/2) sin(π/4-θ/2)}]
= 2r[1/tanθ + (1/2)cos{(π/4+θ/2)-(π/4-θ/2)}/{cos(π/4+θ/2) sin(π/4-θ/2)}]
= 2r[1/tanθ + (1/2)cosθ/{cos(π/4+θ/2) sin(π/4-θ/2)}]
= 2r{1/tanθ + cosθ/(1-sinθ)}(∵積→和の公式)
= 2r/{tanθ*(1-sinθ)}

なんか、グダグダした式変形で分かりにくいかもしれないけど、もし分からなかったら聞いてくれ。

>「f'(α) = 0 となるようなα の値が極小値になる」
については、増減表を書けばそうならない?
640599:04/08/05 05:06
レス遅くてすみません。

>>636 さん
レスしてくれた式をノートにかいて計算しました。おかげで最後まで解けました。
「2r/{tanθ*(1-sinθ)}」を「1+sinθ」で通分したら,>>599 の答えになりました。

>「f'(α) = 0 となるようなα の値が極小値になる」
増減表をかいて解決しました。

レスして下さった方々のように,数学ができるよう勉強します。ありがとうございました。
641132人目の素数さん:04/08/05 08:24
「3x^2+4xy-4y^2+4x-16y-28=0 を満たす整数xyの組を求めよ」
この問題が解けません。
私は上を因数分解して(3x-2y-5)(x+2y+3)=13となったのですが、
このばあい13は素数なので、xyが整数ということはありえないと
思うのですが・・・。
>>641
13=1×13=13×1
643642:04/08/05 08:28
おっと危ない
13=±1×±13=±13×±1(複号同順)
ここまで書かないと場合分けが足りない
644132人目の素数さん:04/08/05 08:55
>>642さん
ありがとうございます結果は。(4、3)(4、-3)(-3、-5/2)(-3、-1/2)
となったので(4、3)(4、-3)の二組が正解でしょうか?
645132人目の素数さん:04/08/05 09:42
>>632
ありがとうございますm(_ _)m
646132人目の素数さん:04/08/05 09:52
2次方程式の根の公式の導出に関して疑問があります。
ax^2+bx+c=0(a≠0)
a(x^2+(b/a)x)+c=0
a(x^2+2(b/(2a))x+b^2/(4a^2)-b^2/(4a^2))+c=0
a(x+b/(2a))^2-b^2/(4a)+c=0
a(x+b/(2a))^2=(b^2-4ac)/(4a)
(x+b/(2a))^2=(b^2-4ac)/(4a^2)
↓ココ!!
x+b/(2a)=±√(b^2-4ac)/(2a)
私が思う、自然な流れは
x+b/(2a)=±√(b^2-4ac)/√(4a^2)
x+b/(2a)=±√(b^2-4ac)/|2a|
±|x|は±xと同一視できるので
x+b/(2a)=±√(b^2-4ac)/2a
とするべきだと思うのです。
数学は厳密さが重要だと思うのですが、みなさんはどう思いますか?
647132人目の素数さん:04/08/05 10:00
>>646
> (x+b/(2a))^2=(b^2-4ac)/(4a^2)
> ↓ココ!!
> x+b/(2a)=±√(b^2-4ac)/(2a)

x+b/(2a)には絶対値つけなくてもいいの?
648132人目の素数さん:04/08/05 10:01
>>646
そういうのは厳密というよりただのアホ
649646:04/08/05 10:10
>>648
根拠なく非難するおまえの方がアホに見えるぞ。
アホな理由を言ってみろボケ!
非難されてる根拠がわからんなら相当のアホ。
651646:04/08/05 10:23
いや、だから理由をいってみろよボケ!
逃げるなよ卑怯者!
652132人目の素数さん:04/08/05 10:35
647は無視なのかw
653132人目の素数さん:04/08/05 11:04
数学板での曖昧な発言はハッタリを疑うべき。
少なくとも断言を避けてる時点で弱気なのは確か。
654132人目の素数さん:04/08/05 11:17
>>646
私は646以降の誰でもないが,一つ言いたい

> ±|x|は±xと同一視できるので

この一文に注目する限り,これはx=√-1のとき間違っている.
解でなく根の公式と言いつつ謎の解釈をしている>>646に萌え
(a+b)^2+2c(a+b)
をa+bでくくるとどうなりますか。曖昧なのでよろしく
雑魚しか釣れなかったね>646
>>656
Xと置くのはかったるいんだが
a+b=Xとおくと
与式
=X^2+2cX
=X(X+2c)
元に戻して
与式
=(a+b)(a+b+2c)
659132人目の素数さん:04/08/05 13:09
あほな質問ですが、det modってなんと読むんですか?
デットモッド
>>660
ありがとうございます。
662天然水:04/08/05 13:25
>>631
訂正

A⇔|a↑|^2 -1=(1/2)|a↑+b↑||a↑-b↑| >0
だから、|a↑|^2 >1
よって答えは
|a↑|=(√5+1)/2. …(答) ←負号の方は|a↑|^2 >1 を満たさないので不適。

>>658
ありがとうございます
664132人目の素数さん:04/08/05 13:35
なぜ今は根の公式ではなく、
解の公式と言うようになったのですか?
根が追放されたから
今は全部解
つーか解と根とは別の概念で
中学高校で根は教えないの
じゃあ、「平方根」と言い方もおかしいのかな?
あーそうかそこは単語として出てきますね
多項式に関しては根とは何かは教えない
668646:04/08/05 15:09
根と解については以下を参照してください。

>>654
当然実数の範囲で考えています。

>>655
萌えられたのは光栄ですが、なぞの解釈とは・・・どこかおかしいですか?
根の公式を自然な流れで導出するためにはやはり>>646のような過程が必要だと思いますよ。

>>657
釣りではないですよ。本気です。

>>650
出てこいやゴルァ!

>>652
低レベルすぎたのでねw

補)
x^2=aを解くとx=±√a
669646:04/08/05 15:09
a>0としても一般性を失わない。
相手にされてないのがわからんのか。夏だな
672132人目の素数さん:04/08/05 15:37
>>646
厳密性が重要なのはおまいの言う通り。しかし、おまいが言うおかしい部分に
厳密性が欠けていると言えるか?同値変形されていれば、厳密に成り立っている
ってこと分かってる?手順の省略と厳密性とは違うのだよ。とマジレスしてみるテスト。
673132人目の素数さん:04/08/05 15:41
>>646
> x+b/(2a)=±√(b^2-4ac)/√(4a^2)
> x+b/(2a)=±√(b^2-4ac)/|2a|

この変形は同値じゃないなw
674132人目の素数さん:04/08/05 15:43
>>673
なぜ?
675672:04/08/05 15:49
それに、別に省略されているわけじゃないぞ。分かりにくいならば、
両辺に4a^2をかけて同じことをしてみろ。そしたら納得できるか?
そんなことしなくても普通の人間なら理解できるんだがな。
この馬鹿はa^2=b^2⇔a=±bがわかんねえのか??
677132人目の素数さん:04/08/05 16:01
複素数α=2+2i β=(1+√3i)/4 γ=αβがある。

複素数平面上で3点O A(α) C(γ)は、
中心が_ア_+i 、半径が√_イ_の円周上にある。

この円周上を点P(z)が動くとき|z+a/2|の最大値は
_ウ_√_エ_であり、z+a/2の偏角の最小値は
_オ_°である。


よろしくお願いします。
678天然水:04/08/05 18:05
>>677
α=2√2(cos45°+isin45°)
β=(1/2)(cos60°+isin60°)
γ=αβ=√2(cos105°+isin105°)
図より∠OCA=90°となるのでOAの中点が円の中心。
∴1+i …ア
半径=|1+i|=√2 …イ
又、|z+α/2|が最大になるとき、α/2,円の中心、z が一直線になるから、
最大値は|α+α/2|=3√2 …ウエ
-α/2 から円に接線を引くと、図より偏角の最小値は15° …オ


(注)図は自分で書いてくれ。
679天然水:04/08/05 18:08
訂正

又、|z+α/2|が最大になるとき、「-α/2」,円の中心、z が一直線になるから、
680132人目の素数さん:04/08/05 18:29
>>678さん
ありがとうございます。
自分でも考えてみます。
実数係数の整式 f(x)=ax^3+bx^2+cx の係数は |a|+|b|+|c|≦1 を満たす。
(1)|x|≦1 である全てのxに対して|f(x)|≦|x|となることを示せ。
(2)|f(k)|=|k| (0<|k|<1)が成り立つ実数kが存在するようなf(x)を全て求めよ。

(1)は解けたのですが、(2)が方針も立ちません。
(1)を参考に解こうと考えているのですが・・・、誰かお願いします。
>>681
y=|f(x)| 、y=|x| のグラフを適当に描いて見よ。
683132人目の素数さん:04/08/05 21:14
f^2(k)=k^2->a,b,c=?
684132人目の素数さん:04/08/05 21:30
間違えて中学生のスレに書いてしまいました。
今度は間違えないように・・・

3点A(-2, 0), B(5, 0), C(0, 4)を通る放物線をTとし、弧BC上に点Pをとる。
放物線と弦BCおよびCPによりかこまれる図形の面積が最小となるときの点Pの座標を求めよ。

という問題なんですが、まったく手がつきません。
私は高一です。二次関数の応用らしいですけどどなたか教えてください。
685132人目の素数さん:04/08/05 21:37
>>684
それちゃんと問題あってる?
点Pを点Bにどんどんちかずければ、面積はどんどん0に近づいていってしまうぞ。
686132人目の素数さん:04/08/05 21:41
すみません。
問題間違えてました

↓正しい問題
3点A(-2, 0), B(5, 0), C(0, 4)を通る放物線をTとし、弧BC上に点Pをとる。
放物線と弦BPおよびCPによりかこまれる図形の面積が最小となるときの点Pの座標を求めよ。

でした。すみません
>>686
Pからx軸に垂線をおろして、その垂線の足をHとすると、
四角形OCPB=台形OCPH+三角形HPBとでもしてやって、これをPのx座標の関数で表す。
この四角形OCPBが最大値を取る時のPの座標が求める座標。
688132人目の素数さん:04/08/05 21:56
>>686

積分使わないと、紛らわしい問題になるね。

弦 BC と放物線で囲まれた部分の面積を $S$ としよう。そうすると、
「放物線と弦BPおよびCPによりかこまれる図形の面積」は、
$S - $ △ BCP で表される。なので、三角形 BCP の面積を P の $x$
座標を $t$ とでもして、求めて、それが最大になるときの $t$ の値
を求めてみては?
689132人目の素数さん:04/08/05 21:56
>>686
三角形PBCを最大化させてやればよい。

BCと平行な直線を考えて、上に移動させていく。一点になるところが求めるところだよ。
690リニアタン(;´Д`)ハアハア ◆O5M8Y2WWjk :04/08/05 21:58
>>681
|f(k)|=|k| (0<|k|<1) より 1=|ak^2+bk+c|≦ |a|+|b|+|c|≦1
(中略)
これを満たすのは、a=b=0、c=±1 のみ
691681:04/08/05 21:58
>>682
あ、かなり分かってきました。thanks
692689:04/08/05 22:01
【詳解】
題意からT:y=a(x+2)(x-5)とおけ、これが(0,4)を通ることからa=-2/5
∴T:(-2/5)(x+2)(x-5)
BCの傾きはすぐに-4/5とわかるから、直線y=-4/5+bを考えて、これとTの交点を考える。
つまり-4/5+b=(-2/5)(x+2)(x-5)
これを整理してD/4=0を適用するとb=13/2となる。
あとはもうわかるでしょ?
693132人目の素数さん:04/08/05 22:02
>>688
ちなみに積分使ってとくとどうなりますか?
俺も高一なもんでちょっとこの問題が気になったり
694689:04/08/05 22:09
Answer:(5/2,9/2)
695689:04/08/05 22:11
ちなみにこれが記述を求める問題じゃない場合(センターとかだったら)
OBの中点のところにあるな

x=5/2

yを求める

終了
696132人目の素数さん:04/08/05 22:34
>>686
BC=S,SH/2
P=(x,y)
H=(P-B)(1-*BC(BC))/(BC*BC)
dH/dx=((1,y')-B)(1-*BC(BC))/(BC*BC)=0
BP'=BP'*BC(BC)/(BC*BC)
BP'=BC
P'=C

697132人目の素数さん:04/08/05 22:44
>>686
BCと平行な線分で放物線と接する点がPで
最大の高さを持つってことか。。。

698132人目の素数さん:04/08/05 22:56
>>693
積分はわかるのかい?
699132人目の素数さん:04/08/05 22:58
lim[n→∞](3nCn/2nCn)^(1/n)
よろしくお願いします。
700689:04/08/05 23:00
logをとって考えてみよう!
701132人目の素数さん:04/08/05 23:02
logをとらない方法でお願いします!!
702689:04/08/05 23:03
>>701
なんで?
君は依頼者?
703777:04/08/05 23:03
この問題が分らないので教えてください!

a>0のとき、関数y=-x^2-6ax+2(0≦x≦2)の最大値を求めよ。

誰か教えてください。
下に凸だから両端を結んだ直線より下
ということは両端の大きい方が最大値
あとはどうとでもなるです
705689:04/08/05 23:09
>>699
この問題結構手ごたえあったわ。
いい問題だね。
答えは
27/16
706132人目の素数さん:04/08/05 23:11
>>699は今年になって何度も何度もコピペされてる問題
707689:04/08/05 23:12
>>706
そうなんだ。
デモトはなんなんですか?
708132人目の素数さん:04/08/05 23:13
>>703
どこの777かが気になるが・・・

平方完成して軸の場所で場合分け
答えはちょっと待って
709132人目の素数さん:04/08/05 23:16
>>703
関数y=-x^2-6ax+2(0≦x≦2)の最大値を求めよ

これは2次関数だからまず、平方完成して
y=-(x+3a)^2+9a^2+2  の形にする。
ここから 軸がx=-3a  頂点座標(-3a,9a^2+2)であることが分かる

ここでaの係数に関して場合分け

1)-3a<0すなわちa>0の時
最大値は 2(x=0)
2)0<-3a<2すなわち-2/3<a<0の時
最大値は 9a^2+2(x=-3a)
3)2<-3aすなわちa<-2/3の時
最大値は -12a-2(x=2)
710709:04/08/05 23:18
>>703
脳内計算だから間違ってる可能性あり
711777:04/08/05 23:26
せっかく答えてもらって申し訳ないですが、関数y=-x^2-6ax+2ではなく、
y=-x^2+4ax-aでした・・・。本当にスイマセン
712132人目の素数さん:04/08/05 23:30
>>711
問題が変わってもすることは何も変わらない
平方完成して、軸の位置で場合分け
ただし、上に凸になるんで3つの場合分けが必要になる
(1)
軸<定義域
(2)
軸が定義域内
(3)
定義域<軸

あとは説いてみな
>>771
数字が変わっただけだろうが……
714132人目の素数さん:04/08/05 23:31
>>711
関数y=-x^2-4ax-a(0≦x≦2)の最大値を求めよ

これは2次関数だからまず、平方完成して
y=-(x-2a)^2+4a^2+a  の形にする。
ここから 軸がx=2a  頂点座標(2a,,4a^2+2)であることが分かる

ここでaの係数に関して場合分け

1)2a<0すなわちa<0の時
最大値は a(x=0)
2)0≦2a≦2すなわち0≦a≦1の時
最大値は 4a^2+a(x=2a)
3)2<2aすなわち1<aの時
最大値は -7a(x=2)

同じく脳内計算なので間違いありかも
715132人目の素数さん:04/08/05 23:32
>ただし、上に凸になるんで3つの場合分けが必要になる
のただしを消して置いてください
716132人目の素数さん:04/08/05 23:32
a<0 のとき f(0)=a
0≦a≦1 のとき f(2a)=4a^2-a
a>1 のとき f(2)=7a-4
717714:04/08/05 23:34
>>709のを数字入れ替えたりしたから、すげえ、間違ってた・・・スマソ
718数学こそ青春:04/08/05 23:49
百・五十・十円がそれぞれ二枚ずつある。全部または一部で支払うこと
のできる金額は何通りあるか。  
>>718

そんくらい書き出せ
720132人目の素数さん:04/08/06 00:02
(10) 2枚、(50) 4枚
の場合と払える金額に違いはないよな?
2×4 - 1 = 7通り
721132人目の素数さん:04/08/06 00:03
まちがえた。
3×5 - 1 = 14通り
うそ教えちゃいかんよ。
>>720
> (10) 2枚、(50) 4枚
> の場合と払える金額に違いはないよな?

(50) 6枚 ?
724132人目の素数さん:04/08/06 00:24
3×7 - 1 = 20通りが正解か?
10,20,50,60,70,100,110,120,150,160,170
200,210,220,250,260,270,300,310,320
726数学こそ青春:04/08/06 00:31
>>724>>725
そおです!!解りやすい説明ありがとうございました。
支払い0円。
728132人目の素数さん:04/08/06 01:13
>>715
上に凸だろうが下に凸だろうが関係なくないか?
729132人目の素数さん:04/08/06 01:31
∫x/(x^2+2)dx
どうやったらいいべ
730132人目の素数さん:04/08/06 01:35
x/(x^2+2)=(1/2)(x^2+2)'/(x^2+2)
dd
732132人目の素数さん:04/08/06 01:40
(x+1)^1000をx^3+x^2+xでわったときのあまりを求めてください。
お願いします。
733132人目の素数さん:04/08/06 01:55
>>732
-2x^2-x+1
かな?
検算中
734132人目の素数さん:04/08/06 02:01
>>729
log(x^2+2)を微分してみてください
735132人目の素数さん:04/08/06 02:02
訂正
2x^2+x+1
736732:04/08/06 02:03
考え方を教えていただけるとありがたいのですが…
737132人目の素数さん:04/08/06 02:13
(x+1)^1000=P(x)*(x^3+x^2+x)+ax^2+bx+c
と書けることを利用してa,b,cを求めればいい.
x^3+x^2+x=0の解を代入するのが効率的.
幾つか計算テクニックがあるので,
決してハードな計算ではない(一回間違えたけど).

基本問題だから,まともな教科書みればやり方は載っていると思う


738732:04/08/06 03:02
ありがとうございます
>>737
あのーx^3+x^2+x=0の解を代入っていっても虚数解じゃん、、、?
虚数解だと何か問題があるんだろうか・・・?
>>740
739はまだ学校で虚数を習ってないんだよ。
nが正の整数のとき,0≦x≦1を満たす全てのxについて、次の不等式が成立することを証明せよ

0≦x^n-x^(n+1)≦n^n/(n+1)^(n+1)

f(x)=x^n-x^(n+1) とおく
@
x=0,1の時
f(x)=0
A
0<x<1の時
n<n+1であるから
x^n>x^(n+1)
∴f(x)>0 @,Aよりf(x)≧0 よって 0≦x^n-x^(n+1)
は導けましたが
ここから二辺と三辺の証明が分かりません よろです。
743132人目の素数さん:04/08/06 11:56
√24=2√6というのは
2^2*6=24だからて事でいいんでしょうか。
744132人目の素数さん:04/08/06 12:50
>>742
>ここから二辺と三辺の証明が分かりません よろです。
もう少し分かりやすく書いてください。

>>743
x>0, y>0 ならば、
√(x*y) = √x*√y
√(x^2) = x
なので、
√24 = √(2^2*6) = √(2^2)*√6 = 2√6 ■
>>744
x^n-x^(n+1)≦n^n/(n+1)^(n+1)
の証明ということでつ。
746132人目の素数さん:04/08/06 13:21
>>745
f(x) = x^n - x^(n+1) とすると、
f'(x) = n*x^(n-1) - (n+1)*x^n
   = x^(n-1)*(n - (n+1)x)
f'(x) = 0 のとき、
 x=0, n/(n+1)

f(0) = 0
f(n/(n+1)) = n^n/(n+1)^n - n^(n+1)/(n+1)^(n+1)
       = n^n/(n+1)^(n+1) > 0
f(1) = 0
なので、x=n/(n+1) のときに最大値 n^n/(n+1)^(n+1) をとり、
     x=0, 1 のときに最小値 0 をとる。
∴ 0≦x^n-x^(n+1)≦n^n/(n+1)^(n+1) (Q.E.D.)
>>746
なるほど
f(n/(n+1))で最後まで求まってしまうのか

サンクスです。
748132人目の素数さん:04/08/06 13:47
常に財布の中のコインの総数をできるだけ少なくするように買い物をするとき、
財布の中のコイン総数の期待値をEとする。ただし商品の値段はランダムであるとする。
(1)流通しているコインが1、5、10、50、100、500円玉のときEを求めよ。
(2)流通しているコインが1、a1、10、10*a2、100、100*a3円玉(1<a1,a2,a3<10でa1,a2,a3は整数)のとき、 Eが最も小さくなるa1,a2,a3を求めよ、またそのときのEを求めよ。
(3)流通しているコインが1、5、5*b1、50、50*b2、500円玉(1<b1,b2<10でb1,b2は整数)のとき、 Eが最も小さくなるb1,b2を求めよ、またそのときのEを求めよ。

(2)(3)をしらみつぶしでやる以外の方法を教えて下さい。
749132人目の素数さん:04/08/06 21:48
http://kazumi.jdyn.cc/cgi-bin3/stored/up0662.pdf
とりあえず、図はかけたのですが、それからサパーリです。
よろしくおねがいします。
751132人目の素数さん:04/08/06 22:53


問題:x,y,nを0または正の整数とする。不等式x+y≦nを満たす組(x,y)の個数を求めよ。

解答:x+y≦n, x≧0,y≧0から0≦x≦n
x,yは整数であるから、x=k(0≦k≦n)のとき、yの値の個数は0≦y≦n-kからn-k+1(個)
よって求める個数は
Σ[k=0,n](n-k+1)=Σ[k=1,n+1]k=1/2*(n+1)(n+2)

質問:最後のΣ[k=0,n](n-k+1)=Σ[k=1,n+1]kという式変形の過程がわかりません。
   よろしくお願いします。
kの動き方が違うが、どちらも1からn+1までの和。
753132人目の素数さん:04/08/06 23:01
m=n-k+1とおくと
k=0のときm=n+1
k=nのときm=1
だから
Σ[k=0,n](n-k+1)
=Σ[m=1,n+1]m
あるいは
=Σ[k=1,n+1]k
754751:04/08/06 23:03
>>752
あ、確かに
Σ[k=0,n](n-k+1)=(n+1)+n+(n-1)+・・・+1
ですね。どうもありがとうございます。
755132人目の素数さん:04/08/06 23:37
すみませんが
高1の数学Tの不等式の問題を教えてください

何本かの鉛筆を何人かの生徒に配るのに、
1人に2本ずつ配ると11本余り
5本ずつ配ると最後の1人には不足が生じるという。
生徒の人数と鉛筆の本数を求めよ。

以上が問題なのですが
生徒の数をxとおくと鉛筆の本数は2x+11(本)
5本ずつ配るとちゃんと5本もらえるのはx-1(人)
2x+11>5(x-1)を解いて
x<16/3

生徒の数は5人以下ということで
条件に合うのは
5人で21本
4人で19本

かな?と思ったのですが、何だか答えが変で
根本的に間違っている気がします

どなたか教えてください。お願いします。
756質問:04/08/06 23:41
問題
平行四辺形abcdがある。aの南西にb。bの真東にc。cの北東にdとする。

辺abの中点をe、辺bcの中点をf、辺adの中点をgとする。
線gcと線deとの交点をh、線gcと線dfとの交点をiとする。

三角形dhiの面積と平行四辺形abcdの面積の比を答えよ。








↑解けません。答えだけでなく、解くまでのプロセスを教えて〜。
757132人目の素数さん:04/08/06 23:42
すみませんが
高1の数学Tの不等式の問題を教えてください

何本かの鉛筆を何人かの生徒に配るのに、
1人に2本ずつ配ると11本余り
5本ずつ配ると最後の1人には不足が生じるという。
生徒の人数と鉛筆の本数を求めよ。

以上が問題なのですが
生徒の数をxとおくと鉛筆の本数は2x+11(本)
5本ずつ配るとちゃんと5本もらえるのはx-1(人)
2x+11>5(x-1)を解いて
x<16/3

生徒の数は5人以下ということで
条件に合うのは
5人で21本
4人で19本

かな?と思ったのですが、何だか答えが変で
根本的に間違っている気がします

どなたか教えてください。お願いします。
>>755
それのどこが変なの?
759755:04/08/06 23:57
ごめんなさい。ボタン2度押ししてしまいました。

>>758
変ということないですか?
その他の不等式の文章題は答えがみんな
○個以下とか△人以上とかになるので
どうもしっくりこなかったのですが…
同じ様な問題がないか手持ちの問題集もみたのですが
このテの問題はなくて、すっきりせずに困っていました。
変でなければ、眠れそうです。
>>755
最後の一人「だけ」が足りないんだから、5(x-1)<2x+11<5xとすべきだね。
そうするとxが自然数という条件の下ではx=4,5しかない。
761755:04/08/07 00:42
>>760
ありがとうございます!

やっと心からスッキリしました。
<5xが足りなかったのですね。
よくあるパターン以外の問題に当たると
そして「解答」がついていないと
すぐに困ってしまって…
助かりました。感謝
>>756
3/40 か。
763132人目の素数さん:04/08/07 00:56
>>756
問題文から面積は、平行四辺形dgfcでまず1/2になり
三角形dgcでさらに1/2になり、三角形dgiでさらに1/2になる。
よって三角形dgiの底辺の比gh:hiを求めればよい。

ここで、da↑=a↑、dc↑=c↑とおくと
dh↑=k*de↑=k*a↑+k/2*c↑
 dh↑=di↑+ih↑=di↑+m*ig↑=1/4*a↑+1/2*c↑+m/4*a↑-m/2*c↑
=(1/4+m/4)*a↑+(1/2-m/2)*c↑ となる。
a↑とc↑は一時独立だから、
k=1/4+m/4 、k/2=1/2-m/2
上の連立方程式を解くと k=2/5 m=3/5
よってih↑=3/5*ig↑ gh:hi=2:3 三角形dgh:三角形dhi=2:3
よって答えは、1/2*1/2*1/2*3/5=3/40 から  3:40
ad=bc=1、cdの中点をjとする。すると ij=1/4 だから、ei=1-ij=3/4
△dgh∽△hei より、gd:ei = 1/2:3/4 = 2:3 ⇔ gh:hi = 2:3
△dgc = (1/4)*abcd、またiはgcの中点になるから、△dgi = (1/2)*△dgc
よって、△dhi = {3/(2+3)}*△dgi = {3/(2+3)}*(1/2)*(1/4)*abcd = (3/40)*abcd
765132人目の素数さん:04/08/07 01:31
da=x
dc=y
dh=u
di=v
(全部ベクトル)
とおいたとき,平行四辺形abcdの面積と三角形dhiの面積はそれぞれ
abcd=|x×y|
dhi=(1/2)|u×v|
であることを知っていれば(×は外積),次のようにしても求まる

線分gc上の点は
kx/2+(1-k)y 0≦k≦1.......(1)

h,iはそれぞれde,df上の点だから
u=s(x+y/2) 0≦s≦1..........(2)
v=t(x/2+y) 0≦t≦1..........(3)

(1),(2)および(1),(3)から
u=2x/5+y/5
v=x/4+y/2

したがって
dhi=(1/2)|u×v|=(3/40)|x×y|

abcd=|x×y|だったから
その比は3/40
>>748
例えば5円玉の代わりに8円玉が使えるとすると、
27円は10×2+1×7だと9枚だが
10×1+8×2+1なら4枚で済む。
出題者はこの種の組み合わせも考えているのか?
そこまで考えると異様に難しくなると思うのだが。
767132人目の素数さん:04/08/07 17:35
>>766
そうです。a_2、a_3と無関係にa_1が決まっていくのかなと思いきやそうでもなさそうだし・・・
残る方法が10^3通り全て列挙するしかなくなって降参。。
768132人目の素数さん:04/08/07 17:53
297分の1 + 420分の1を計算してくれ〜〜っ
スロットの合成確立なんだ
>>768
マルチ
770132人目の素数さん:04/08/07 18:59
>>768
41580分の239。
電卓使えば簡単w
771132人目の素数さん:04/08/07 19:33
この計算の過程が???です。どなたか解説おねがいします。

7a/2(2-a)=7
772132人目の素数さん:04/08/07 19:34
すみません、7a/2(a-2)=7でした。おねがいします。
773132人目の素数さん:04/08/07 19:36
>>772
7a/(2*(a-2)) = 7
だったら、両辺に 2(a-2) をかける。
774132人目の素数さん:04/08/07 19:50
どうもありがとうございました。
われながらくだらない質問で本当に申し訳なくなりました。
775132人目の素数さん:04/08/07 19:54
774ゲトー!!
776132人目の素数さん:04/08/07 20:48
因数定理の宿題でわからないものが1つあり、にた問題を解説している
サイトを見つけて学習していたんですが、以下の問題の途中が解説を見
てもどうしてもわかりません…。

整式P(x)を(x−1)^2で割ると3x−7余り、x−2で割ると2余る。
この整式P(x)を(x−1)^2(x−2)で割った余りを求めよ。

[解]
P(x)を(x−1)2(x−2)で割った時の商をQ(x)、余りをax2+bx+cとおくと、
P(x)=(x−1)2(x−2)Q(x)+ax2+bx+c ・・・[1]

【P(x)を(x−1)2で割って3x−7余ることから、
ax2+bx+cを(x−1)2 で割っても、3x−7余る。
割った時の商は、aであるから、
ax2+bx+c=a(x−1)2+3x−7
とおける。これを[1]に代入して、
P(x)=(x−1)2(x−2)Q(x)+a(x−1)2+3x−7 ・・・[2]】

P(x)をx−2で割って2余る、という条件から、P(2)=2 ・・・[3]
[2]でx=2とおくと、[3]より、a−1=2  ∴a=3
よって、求める余りは、3(x−1)2+3x−7、すなわち、
3x2−3x−4である。 ・・・(答)

解答を見てもわからないなんて情けないですが、
【】の部分がなぜそうなるのかわかりやすく教えて頂ければ幸いです。
777132人目の素数さん:04/08/07 21:09
>>776
 P(x) = (x-1)^2(x-2)Q(x) + ax^2 + bx + c
なので、
 (x-1)^2(x-2)Q(x) は (x-1)^2 で割り切れるので、
 (x-1)^2 で割って余りが出るのは ax^2 + bx + c の部分。
 ax^2 + bx + c を (x-1)^2 = x^2 - 2x + 1 で割ったときの商は、実際に割り算をすれば a と分かる。
778132人目の素数さん:04/08/07 22:27
問:nが2以上の自然数のとき、次の不等式が成り立つことを数学的帰納法で
  証明しなさい。

(1/1^2)+(1/2^)+(1/3^2)+…+(1/n^2)<2−(1/n)

一応やってみましたがさっぱり…お願いします。
>>778

ご親切に「数学的帰納法を使いなさい」と指令があるんだから、
その通りやればよい。
780132人目の素数さん:04/08/07 22:32
$n = 2$ のときの証明をし、
$n = k(\geqq 2)$ のときを仮定した上で、$n = k + 1$ のときを証明
すればよいのだよ。
781132人目の素数さん:04/08/07 23:13
お願いします

x+(2y)=3,0≦x≦3のとき、x^2+2y^2の最大値と最小値を求めよ
>>781

最初の式からy = 1/2 * (3 - x)
と分かるので、x^2+2y^2に代入する。xだけの式になるので、後は適当に考える。
783132人目の素数さん:04/08/07 23:20
適当?
784132人目の素数さん:04/08/07 23:23
>>781

どっかで見たような、、、。
適当というのは、好きな方法でやってくださいという事だ。絵を書いてもいいし、
平方完成してもいいし、微分してもいいし。自分の知ってる範囲で。
786132人目の素数さん:04/08/07 23:24
次の問題が分からないのでどなたかお願いします!
(1)は480x+320
(2)は40人とでました。
(3)は全く分かりません・・・。
お手数かけますが説明も加えていただければ幸いです。

<一次方程式の文章題>
ある学校のK先生に赤ちゃんが生まれた。とてもめでたい話なので、
K先生が担任するクラスの生徒全員が参加して、パーティーを開くことになった。
会費を1人につき480円にすると、パーティーに必要な費用は320円不足する。
そこで、『生徒1人につき490円ずつ集めたところ、予定の費用より多く集まり、残金がでた。残金は80円より多くなる計算』だった。
ところが、招待したK先生から2000円いただいたので、ありがたく頂、
あまったお金を生徒全員に返すことにした。生徒一人当たり50円ずつ返してもまだ少し残る。
(1)クラスの生徒数をx人とするとき、パーティーの費用をxの式で出せ。
(2)下線部(『』)のことから、生徒数は何人より多いことが分かるか。
(3)クラスの生徒の人数を求めよ。
>>786

とりあえず、出来たところまで式も含めて書いてくれ。
>>786

マルチ。
789132人目の素数さん:04/08/07 23:37
次の等式を示せ。。。

lim   n^k / a^n = 0
n→∞


lim   ln n / n^k = 0
n→∞


lim   a^n / n! = 0
n→∞



Comparison Test
Limit comparison Test
Ratio Test

のどれかを使うらしいのですが、
どれにどうどう使えばいいのかわかりません。
教えろくだせい
>>789

Comparison Test
Limit comparison Test
Ratio Test
というのが何なのかよくわからん。説明してケロ。それを使って
解かなきゃいけないの?
791132人目の素数さん:04/08/07 23:43
Cが楕円(x^2)/(a^2)+(y^2)/(b^2)=1であるとき,グリーンの定理を
用いて積分∫[C]y^2dx+xdyを求めよ。(曲線の向きは正の向きにとるものとする。)
>>791
いい加減コピペうざい
793132人目の素数さん:04/08/08 00:08
>>791
πab
794132人目の素数さん:04/08/08 00:17
どなたか>>748をお願いします。
>>794

しらみつぶしでいいじゃん。
796132人目の素数さん:04/08/08 00:44
>>794
1, 5, 10, ... のときは 5 が 2 枚あるのが無駄だから 5 の枚数は 0 or 1 なのだが、
1, 8, 10, ... のときは 8 が 2 枚あることが必ずしも無駄ではないような気がする。
やっぱり無駄?
>>796

16円玉があるなら無駄。
798132人目の素数さん:04/08/08 00:54
1 = 1
2 = 1+1
3 = 1+1+1

7 = 1+1+1+1+1+1+1
8 = 8
9 = 8+1
10 = 10
11 = 10+1
12 = 10+1+1

15 = 10+1+1+1+1+1+1
16 = 8+8
17 = 8+8+1
18 = 10+8
19 = 10+8+1
20 = 10+10
21 = 10+10+1
22 = 10+10+1+1
23 = 10+10+1+1+1
24 = 8+8+8
25 = 8+8+8+1
26 = 10+8+8


20円を10円玉2枚で持ってる状態で4円の買い物をする時は、
やはり10円玉2枚で払って、8円玉2枚受け取るんだろうか。

この辺のことも考え始めると結構ややこしい。
800132人目の素数さん:04/08/08 08:35
Cが楕円(x^2)/(a^2)+(y^2)/(b^2)=1であるとき,グリーンの定理を
用いて積分∫[C]y^2dx+xdyを求めよ。(曲線の向きは正の向きにとるものとする。)
x=acost,dx=-asint
y=bsint,dy=bcost
∫-ab^2sint^3-abcost^2dt


801132人目の素数さん:04/08/08 08:51
S=∫fdx+gdy=∫(gx-fy)dxdy=∫(2y+1)dxdy
x=arcost,y=brsint
(2brsint+1)(-arsintdt+acostdr)∧(brcostdt+bsintdr)
=(2brsint+1)(abrcost^2+abrsint^2)drdt
=(2brsint+1)(abr)drdt
=((2ab^2r^3/2)sint+abr^2/2)dt
=-2ab^2r^3/2cost+abtr^2/2
=abπ
802132人目の素数さん:04/08/08 09:03
∫-ab^2sint^3+abcost^2dt
803132人目の素数さん:04/08/08 09:13
sint^3=((e^it+e^-it)^3)/8
=(e^it+e^-it)(3/8)=0
cost^2=((e^it-e^-it)^2)/-4
=-2/-4=1/2
=ab(1/2)2π=abπ
804132人目の素数さん:04/08/08 09:20
直接解くほうが簡単だな
805132人目の素数さん:04/08/08 09:43
線積分のうちに x=aξ、y=bη とか置換して、単位円周上の積分に直したほうがよくね?

S = ab∫[C'] (bη^2dξ+ξdη)
= ab ∫∫(1-2bη)dξdη
= ab ∫∫(1-2bsinθ)rdrdθ

先に θ で積分すれば簡単だべ。
806132人目の素数さん:04/08/08 11:07
x+(2y)=3,0≦x≦3のとき、x^2+2y^2の最大値と最小値を求めよ
807132人目の素数さん:04/08/08 11:08
m(Y/n)=1/n(mY) を示せ
808132人目の素数さん:04/08/08 11:32
>>806
既出&マルチ

>>807
m*(Y/n) = (1/n)*(m*Y) だったら、計算すれば明らか。
それも既出問題のコピペだろ。
810776:04/08/08 12:22
>>777
ありがとうございます。
す、すみません…まだ以下がよくわからないのですが、もうちょっと解説して
もらえませんでしょうか。すみません馬鹿で_| ̄|○

>(x-1)^2(x-2)Q(x) は (x-1)^2 で割り切れるので、
>(x-1)^2 で割って余りが出るのは ax^2 + bx + c の部分。

>>810
P(x) を (x-1)^2 で割った余りは ax^2+bx+c を (x-1)^2 で割った余りに等しい。の意
812132人目の素数さん:04/08/08 12:46
>>810

$P(x) = Q(x) (x - 1)^2(x - 2) + ax^2 + bx + c$ で、
$ax^2 + bx + c = a(x - 1)^2 + (2a + b)x + (c - a)$ なので、
$P(x) = (x - 1)^2 (Q(x)(x - 2) + a) + (2a + b)x + (c - a)$
であるから、$P(x) $ は恒等式なので、
$2a + b = 3 , c - a = -7$

どっかの教科書に載ってたような問題やな。
813132人目の素数さん:04/08/08 12:52
この問題が分かりません。誰か教えてくさい。

下の□の中に1〜9の数字を全て当てはめて引き算を完成させなさい。

 □□□□□
− □□□□
──────
  3 3 3 3 3
41268-7935 = 41286-7953 = 33333
815132人目の素数さん:04/08/08 13:54
12789-9453=3333
41286 - 7953 = 33333
817813:04/08/08 14:51
ありがとう。
818748:04/08/08 18:14
>>795-799
レスありがとうございます。
虱潰しを10通りぐらいに減らせないかと考えたけど無理でした。
ところでみんななんでa_1=8と予想?
これ、ひょっとして釣り問なんだろうか。
>>818

難しい問題だと思うよ。
820748:04/08/08 18:33
>>819
解けると思う?
>>820

パターンが有限しかないから解けるだろ。
822748:04/08/08 18:41
その通りだけど、コンピュータを使わず受験数学式で上手く解けるのかな、と。
入試で出たら、とりあえず捨てたほうがいいと思うが。
824132人目の素数さん:04/08/08 18:52
>818
はやくに有り金全部使い切ればいいってことでしょ
時間がたてば、Eは平均化されるし。つりはいらね〜よって
825132人目の素数さん:04/08/08 18:55

x+(2y)=3,0≦x≦3のとき、x^2+2y^2の最大値と最小値を求めよ
826132人目の素数さん:04/08/08 19:02
>>825

もうっ、やめてっ!!
827132人目の素数さん:04/08/08 19:06
NGワード推奨
「x+(2y)=3」「因数分解」
828132人目の素数さん:04/08/08 21:12
円C:x^2+y^2=a^2と点A(b,0)、B(0,b)がある。但しa/sqr(2)<b<aとする。
第一象限の円C上に点Pをとる。AP+BPが最小となる、点Pを求めよ。

 線分ABを直径とする円と円Cの交点が最小となるPのような気が
するのですがなぜそうなるかわかりません…
829132人目の素数さん:04/08/08 21:44
電車が駅AからBまでの距離 4.8km をはしります。

最大加速は 2.4m/s^2 で最大減速は 3.6m/s^2 です。

AB間での最大速度は 107km/h です。

AからBにかかる最短の時間をもとめなさい。

誰かお願いします。。。

830132人目の素数さん:04/08/08 21:45
>>829
マルチですが宜しく。
831132人目の素数さん:04/08/08 21:59
簡単のため、「多項式」は、複素係数 m 変数多項式で考える。
さて、多項式を要素とする n 次正方行列 A が、 A^2 = A を満たしているとする。
この時、多項式を要素とする n 次正方行列 B, C で、
(イ) 積 BAC が対角行列で、対角成分は 0, 1 からなる。 
(ロ) 積 BC は単位行列となる。・・・なる2条件を満たす物が存在する。
どうやって解くの?
832132人目の素数さん:04/08/08 22:35
よろしくお願いします。

関数f(x)=|x-a|の区間0≦x≦1における最大値をg(a)とする。
区間-1≦a≦3におけるg(a)の最大値と最小値を求めよ。

833132人目の素数さん:04/08/08 22:37
>>832
f(x)のグラフかけ。
a<0, 0≦a≦1, 1<a の3つの場合に分けて。
そしたら g(a) が求められるだろ。

g(a) = f(1) for a<0
g(a) = f(a) for 0≦a≦1
g(a) = f(0) for a>1

あとは g(a) のグラフ描いてお終い。
834833:04/08/08 22:38
ごめん、うそかいた。
835833:04/08/08 22:39
a<1/2, 1/2≦a の2つの場合の f(x) のグラフをかく。

g(a) = f(1) for a<1/2
g(a) = f(0) for a≧1/2

で g(a) のグラフ描いて…

ほんとにスマン。
↑でたらめ
837132人目の素数さん:04/08/08 23:39
w
838132人目の素数さん:04/08/09 00:01
どこがでたらめ?
839748:04/08/09 10:00
>>823
そうしますw

>>824
実は問題文だけからでは分からないけど(1000円以上は普通に札を使うので)
所持金自体は無尽蔵にあると考えます。

ここの住人さんでも無理か・・・困ったな
840132人目の素数さん:04/08/09 12:07
>>826

そそるわー。
841組合せnCr=n!/r!(n-r)!:04/08/09 12:35
組合せの問題からの疑問です

組合せの公式は nCr=n!/r!(n-r)! のはずですが

n=5 r=1 の時のnCr と n=5 r=5 の時のnCr が なぜ 1になるのか
理由がわかりません。

↑の答えは公式に当てはめるとどちらの答えもゼロ(0)になりそう
ですが僕が間違えているとすれば階乗(!)の理解で詰まっていると
いう事だと思います。階乗は計算して0になる時も答えを1とすべ
きなんでしょうか?どなたかご教授よろしくお願いいたします。
>>841
r=1のときは1にはなりません。
r=5のとき、5C5=5!/(5!*0!)=1ではありませんか?

まさか0!=0だなんて思っていないでしょう?
843832:04/08/09 12:42
誰か>>832の問題お願いします。
844組合せnCr=n!/r!(n-r)!:04/08/09 12:46
>>842さん。ありがとうございます。

>>まさか0!=0だなんて思っていないでしょう?

 いえ...0!=0だと思ってました。0!=1なら全体のつじつまが合います
が今度は0!=0の理由を教えていただきたく思います。

今の所、例えば3!の場合は 3!=3*2*1=6 なので 0!=0 だと誤解したまま
になってます。よろしくお願いいたします。
>>844
2!=3!/3
1!=2!/2
0!=1!/1=1。
846132人目の素数さん:04/08/09 12:51
次の条件を満たす放物線の方程式を求めよ。ただし、軸はy軸に平行とする。
問)頂点の座標が(3,-9)で、x軸から切り取る線分の長さが6である。

お願いします。問題の意味さえわからないので・・
847組合せnCr=n!/r!(n-r)!:04/08/09 12:51
>>845さん

ありがとうございます。これまでにないほど良くわかりました。
ほんとに基本的な事で失礼いたしました。
848132人目の素数さん:04/08/09 12:54
>>843
>関数f(x)=|x-a|の区間0≦x≦1における最大値をg(a)とする。
>区間-1≦a≦3におけるg(a)の最大値と最小値を求めよ。

場合分け。

関数 f(x)=|x-a| は、x-a<0 すなわち x<a のとき単調減少し、x≧a のとき単調増加するので、
f(x) の区間0≦x≦1における最大値は f(0) あるいは f(1) である。
区間-1≦a≦3における f(0) と f(1) の値は、
 f(0) = |-a| = |a|
 f(1) = |1-a| = |a-1|
なので、グラフを書くと、
 a<1/2 のとき f(0)<f(1)
 a≧1/2のとき f(0)≧f(1)
となることが分かる。
グラフ(略)より、最大値4、最小値1/2。
グラフは自分で書いて下さい(AAでグラフはきついので)
>>84
a≠0 として放物線は、y=a(x-3)^2-9 とかける。またx軸との交点は、
0=a(x-3)^2-9 ⇔ (x-3)^2 = 9/a ⇔ x=3±(3/√a)
(切り取る線分の長さ)=3+(3/√a)-{3-(3/√a)}=6/√a=6 ⇔ a=1
∴ y=(x-3)^2-9=x^2-6x
8501117:04/08/09 15:30
a(x-2)^2+b(x-2)+c=x^2+x+1

問:恒等式になるようにabcの値を求めよ。

誰か教えてください。
何が分からんのだ?何も考えずに計算しても必ずできると思うが。
楽なのは右辺をx-2について整理する方法だろうけど。
852132人目の素数さん:04/08/09 15:37
>>850
1117ってなんだよ。
つーか、何がわからないの?
始める前からわからな〜いじゃ話にならん。
x-2についてまとめてみたが、
結局普通に係数を決めていく場合とあんまり変わらん気がしてきた。
854132人目の素数さん:04/08/09 15:42
つーか、>>850程度の問題解けないやつって、ちゃんと教科書読んでるのか?
2chに問題丸投げして、答えがでてきたら、それを写すみたいなことやってんじゃねーのか?
そーゆーやつは氏んだほうがいいよ。
855132人目の素数さん:04/08/09 15:45
十分に長い半径rの円柱2本を直交させたときの共通部分の体積を求めよ。

分かる方いらしましたらよろしくお願いします。
856132人目の素数さん:04/08/09 15:46
lim_[x→+0](x^x) はどうやれば求められますか?
対数取って lim_[x→+0]log(x^x) = lim_[x→+0]{x*log(x)} としてみたのですが、これ以上わかりません
>>851 >>853
おすすめは、与式、与式を微分した式、与式を 2 回微分した式に 2 を代入。
858132人目の素数さん:04/08/09 15:49
>>856
おまえそれ自分で考えた問題じゃないか?
ちゃんとデモトの問題集なり、参考書なりはあるのか?
859132人目の素数さん:04/08/09 15:53
>>858
友だちに聞かれました。
一応 数学は得意な方なので 何とか解きたいです。
860132人目の素数さん:04/08/09 15:53
>>859
じゃー何とか解けよ
>>856
xが0に行くのとlogxが負の無限大に行くのとの比較なのだが、
logx=-tとでも置くと、指数関数と多項式との発散する速さの比較に帰着される。
>854
何言ってんだ!
質問スレは、問題入力したら答えが出てくるブラックボックスだろが!
863132人目の素数さん:04/08/09 16:06
>>856
1
>>861
ありがとうございます!

log x = -t とおくと x→+0 は t→∞ と同じで x*log(x) = -t*e^(-t) となりました。
指数関数と多項式との発散の速さを比較すると、指数関数の方が速い (?) ので、
x*log(x) = -t*e^(-t)→0 (t→∞) でしょうか?
そうならば x^x = e^{x*log(x)}→1 (x→+0) となりそうですが
865132人目の素数さん:04/08/09 16:15
>>855
一応問題の確認だけど、3次元空間を考えて
x軸を軸とする半径rの円柱と(y^2+z^2=r^2)
y軸を軸とする半径rの円柱と(z^2+x^2=r^2)
の共通部分の体積を求めればいいんだよね?
866132人目の素数さん:04/08/09 16:18
>>865
はい、そうです。
867132人目の素数さん:04/08/09 16:23
うーん、難しいね
>>856
gnuplotとかでx^xのx=0付近のグラフを描いてみるとおもしろいかも
869132人目の素数さん:04/08/09 16:36
lim[(x,y)→(0,0)]x^yも1ですか?
>>869
おまえはどう思うんだ?
>>850
a(x-2)^2+b(x-2)+c=x^2+x+1
⇔(a-1)x^2+(-4a+b-1)x+4a-2b+c-1=0
これがxの恒等式なのでa-1=-4a+b-1=4a-2b+c-1=0
>>869
近づき方により異なる。
873132人目の素数さん:04/08/09 17:07
>>870
わかりません・・・
>>872
ど、どういうことですか!?
874132人目の素数さん:04/08/09 17:08
>>855
答えとかだけでもわからんのか?
875132人目の素数さん:04/08/09 17:09
0^0は定義されてないだろうがよ
>>875
は?
877132人目の素数さん:04/08/09 17:16
>>855>>874
頻出だよ.切り口が正方形になるようにして積分.
東大では,楕円柱の直交の場合が出た事がある.
878132人目の素数さん:04/08/09 17:19
>>858
馬鹿晒しあげ
879因数分解:04/08/09 17:21
三次方程式の因数分解おしえてください!!
27A三乗−125
>>879
それは方程式ではないよ。
881因数分解:04/08/09 17:23
(○д○)え・・・・
882855:04/08/09 17:29
>>877
解答お願いできませんか?
883132人目の素数さん:04/08/09 17:35
>>882
>>865で 平面 z=a で切って積分すればいい.
ここから後はがんば.
884855:04/08/09 17:42
平面z=aで切ると一辺が2√(r^2-a^2)の正方形になって、
その面積はS=4(r^2-a^2)
これを-rからrまでaで積分して、
V=∫[-r,r]4(r^2-a^2)da=8∫[0,r](r^2-a^2)da=8[(r^2)a-(a^3)/3][0,r]=(16/3)r^3

こんな漢字でいいんでしょうか?
885132人目の素数さん:04/08/09 17:45
>>877
その東大の楕円柱の問題教えてくれない?
解いてみたい。
>>869
x=(1/2)^(1/y)のまま(x,y)→(0,0)としてみよ。
887132人目の素数さん:04/08/09 19:10
>>861
x*log(x) のあと x=1/n (n→+∞) と置換してもいいかもね。
>>869>>873
2変数の極限については高校では扱いません。
lim[x→0]{lim[y→0]f(x,y)} や lim[y→0]{lim[x→0]f(x,y)} というのはOKですが
lim[(x,y)→(0,0)]f(x,y) はこれらとは違う。値が一致するときはどれで書いてもかまいませんが。
このへんの微妙な話をするには、少なくとも大学生向けの微分積分の入門書が必要です。
背伸びしてよくわからない話を鵜呑みにしてわかったつもりになるよりも、
今のうちに地盤をしっかり固めておいて確実な知識を身につけていくことをお勧めします。
889132人目の素数さん:04/08/09 19:17
関数 f(x,y)=x^y は (x,y)∈(0,∞)x(0,∞) では連続だが、
(x,y)∈[0,∞)x[0,∞) では連続ではないってこった。
あーでもさー

0^0=1

とするのが妥当なんじゃないの?

0/0=0

ってのもイケル
891132人目の素数さん:04/08/09 20:09
夏だね
夏だな、香ばしいやつが多い。
でもさー不定といっても一番妥当なのはこれでしょ?
lim[y->0](lim[x->0] x^y) = 0^0 = 1
lim[x->0] x/x = 0/0 = 0
ってことですね!!
895132人目の素数さん:04/08/09 20:51
は?
数学A

・異なる3文字を選ぶ選び方
・異なる3文字を選んで一列に並べる並べ方
の違いを教えて下さい
897132人目の素数さん:04/08/09 21:47
>>896
A, B, C, D の4文字から
(1) 異なる3文字を選んたものを全て書いてみよ
(2) 異なる3文字を選んで並べたものを全て書いてみよ
>>896
上は順番がないけど下は並べる順番がある
899132人目の素数さん:04/08/09 21:59
>>855
(π^2r^2)/2だったりして?
900132人目の素数さん:04/08/09 22:20
>>855
2∫π(rsint)^2dt、0->π/2
lim[x→0]x^yはyによって結果が変わるから決定できない
lim[x→0]x/x=lim[x→0]1=1
902132人目の素数さん:04/08/09 22:27
903894:04/08/09 22:30
>>901
そんな事はもちろんわかってますよ。
>>890が阿呆な事逝ってるからノってみただけです。
904132人目の素数さん:04/08/09 22:51
lim[x→0]x^yはyによって結果が変わるから決定できない
lim[x→0]x^yはyによって結果が変わるから決定できない
lim[x→0]x^yはyによって結果が変わるから決定できない
lim[x→0]x^yはyによって結果が変わるから決定できない
lim[x→0]x^yはyによって結果が変わるから決定できない
lim[x→0]x^yはyによって結果が変わるから決定できない
lim[x→0]x^yはyによって結果が変わるから決定できない
lim[x→0]x^yはyによって結果が変わるから決定できない
lim[x→0]x^yはyによって結果が変わるから決定できない
lim[x→0]x^yはyによって結果が変わるから決定できない
lim[x→0]x^yはyによって結果が変わるから決定できない
lim[x→0]x^yはyによって結果が変わるから決定できない
lim[x→0]x^yはyによって結果が変わるから決定できない
lim[x→0]x^yはyによって結果が変わるから決定できない
順列 男子5人と女子3人が縦に一列に並ぶとき,女子の直後に必ず男子が来るような並び方は何通りあるか。

考えたは、女子と男子をひとまとまりとして考えるということでよろしいでしょうか?
>>897同じ
907905:04/08/09 22:55
すみません 下げてしまいました。
908132人目の素数さん:04/08/09 23:09
今読んでいるアメリカの数学の教科書に
「1以下の正の整数」という表現があるのですが
これは「1」を指しているということですよね。
TR909ゲット!
910132人目の素数さん:04/08/09 23:17
>>908
文脈にもよるけど、多分1以上の正の整数とかの間違いのような気がする。
もし誤植とかでなければ、その解釈(「1」のみ)でOK.
というか、原文キボン
>>906
違う。
912132人目の素数さん:04/08/09 23:49
>>910

どうもありがとうございます。

原文は次のようになっています。

n=positive integer less than or equal to 1
>>912
それなら確かにどう読んでも「1」だけとしか読めない・・・。
多分、何かの説明の一節だと思うけど。
>>905
男子を■、女子の「入り込める」場所を「↑」で表すと下のように書けるよ。
 ■ ■ ■ ■ ■
↑ ↑ ↑ ↑ ↑ 
                 
よって、(5P5)*(5P3) = 7200 とおり。
>>913

やはり何度考えてもそうですよね。
ありがとうございました。

関数の式の説明なのですが、内容は電気?のようです。
Il1
917132人目の素数さん :04/08/10 05:59
>>912

>n=positive integer less than or equal to 1
であることの証明の中でなら,なんの不思議もない.

したがって私はそのような文脈の中での言葉であろう予測する.
918132人目の素数さん:04/08/10 06:47
単位円上を点が一周する間に、それによってxy平面上のすべての点
に移動されるような変換て有りますか?
919132人目の素数さん:04/08/10 09:13
五角形ABCDEにおいて、CDとACの交点をKとして、
AB=AE BC=DE=√2 CD=4 ∠C=∠D=135度 AK=KC が成立している。

(1)ベクトルCEを、ベクトルCB、ベクトルBDを用いてあらわせ。

(2)Kから辺CDへ下ろした垂線の足をLとする。線分CLの長さを求めよ。
ベクトルCAをベクトルCB、ベクトルBDを用いてあらわせ。


(2)の、CLの長さを求める方法がわかりません。教えてください。
>>919
CDとACの交点KってCじゃないのかと
問題あってるのか?
921919:04/08/10 09:35
>>920

五角形ABCDEにおいて、BDとACの交点をKとして、
>>897
解説してくれよ
分からんから聞いてる
0≦a≦1である時
関数f(a)=∫[0,1]{|x(x-a)|}dx の最小値を求めよ

よろでつ。
>>923
f(a)=∫[0,1] {|x(x-a)|}dx = ∫[0,a] { -x( x - a ) }dx + ∫[a,1] { x( x - a ) }dx
>>918
変換自体に自由度(独立1変数)が付加されてなければ不可能。
x-1/x=1のとき
x^3-1/x^3はいくつになりますか?
(x-1)/x = 1 ? x-(1/x) = 1 ?
(x^3-1)/x^3 ? x^3-(1/x^3) ?
x^3-(1/x)^3 = (x-(1/x))^3+3*x*(1/x)*(x-1/x)
= 1^3+3*1
= 4
>>926
(x-1/x)^3 = x^3 - 3・x^2・1/x + 3・x・1/x^2 - 1/x^3
= x^3 - 1/x^3 - 3(x-1/x)
930132人目の素数さん:04/08/10 17:02
三角形の五心なんですが
チェバの定理の逆が成り立てば、
三角形の頂点から引いた直線は一点で交わるわけですよね?
ようするに三直線が交わるような点は無数(というかどの点でも交わるけど…)
にあるのに、なぜ五心は特別扱いなんでしょうか?
他の有名な点とかってありませんか?
>927
スマソ
>928,929
ありがとう!
932132人目の素数さん:04/08/10 17:14
方程式x^2+x+1=0の解の1つをωとする。

(1)ω^3=1であることを示せ
サッパリわかりません_| ̄|○お願いします。

(2)ω^10+ω^5+3の値を求めよ。
これはω^3を含めた式に変形して代入すればいいんでしょうか?
そのとき余ったωの処理についても教えて頂けると助かります。
933ペプシ工員:04/08/10 17:20
>>932
(1) ω^3−1=(ω−1)(ω^2+ω+1)=0
(2) ω^10+ω^5+3=ω(ω^3)^3+(ω^2)(ω^3)+3=ω+ω^2+1+2=2
>>932
蛇足だが、この手の問題は一般的に
「求めたい多項式」を「0になる多項式」で割った余りを求めれば解ける。
935132人目の素数さん:04/08/10 17:48
高1の二次関数の問題を教えてください。

関数y=−(x2−2x)2+2(x2−2x)について
(1)m=x2−2xとおくとき、mの値の範囲を求めよ
(2)(1)の結果を利用して、yの値の範囲を求めよ

半角の2は2乗の意味です。

(1)はy=−m2+2m
     =−(m−1)2+1と変形して
    mが1のとき最大値が1かなと思うのですが
問題がmの値の「範囲」を求めよーなのが気になります

もし、(1)が間違っていない場合(2)は
m=1のときyの最大値1ということで

x2−2x=1を解いてx=1±√2のときy=1?

どうも問題の意味がわからず困っています。
どなたか教えてください。
>>930

他にもあったかも知れないけど
五心はそれぞれ幾何学的な意味があるからこそ重要
>>935
横軸x、縦軸mのグラフや横軸m、縦軸yのグラフは書けるか?

知り合いに、計算問題は得意だが
一種の記号操作として理解しているみたいで、
文章題が苦手で数式の意味が分かってないらしい子供がいる。
君も似たタイプかな?

この辺りは解き方云々よりも君の中のイメージの問題だから、
ネットで聞くよりも友達と色々話し合う方がお薦めかと。
938918:04/08/10 19:05
>>925
ありがとうございます
939高1:04/08/10 19:19
>>935
m自体が二次関数になってるから、そこでmの値の範囲が決まるらしいよ。
参考書とかで、4次関数の最大・最小ってところにのってるよ。
僕も今日そこやったばかり。
>>936
幾何学的意味のある点は他にもたくさんあるみたいだよ
9点円とか
五心は歴史的に発見されたのが早かったからそう呼ばれただけじゃないかな
>>935
(1) m = x^2 - 2x
    = (x-1)^2 - 1
  よって、m≧-1

(2) x^2 - 2x = m とおくと、与式は
  y = -m^2 + 2m 
   = -(m-1)^2 + 1 と変形できる。
  (1)で示した通り、m≧-1 なので、yの範囲は
  y≦1 となる。
942935:04/08/11 04:25
>>941
ありがとうございます。
(1)で問題を取り違えたのが、そもそもの失敗でした
お恥ずかしい…ながらもとても助かりました
943132人目の素数さん:04/08/11 05:44
証明問題が出来ないので教えてください。

「関数f(x)はf(0)=0,∫[0〜1]f(x)dx=1を満たし、その導関数f'(x)は連続であるとする。
この時、区間[0,1]における導関数f'(x)の最大値をMとすればM≧2であることを証明せよ。」

一応途中まで考えましたので書きます。

f(1)≧2のとき平均値の定理から(f(1)-f(0))/(1-0)=f'(a)となる0≦a≦1が存在する。
f'(a)=2です。
f(1)<2のとき、∫[0〜1]f(x)dx(面積)が1なので、多分f(b)≧2bとなる0≦b≦1が存在すると思います。
でも、なんて書けばいいか(証明)が分かりません。お願いします。
944943:04/08/11 05:52
すいません。なんて書けばいいのか分からないのはf(1)<2のときです。
お願いします。
>>943
外してるかも試練が、f(b)=1 を満たす点 b を端点に平均値とか?
946943:04/08/11 06:36
>>945
ありがとうございます。
f'(a)=(f(b)-0)/(b-0)=1/bですか。
でもbがどこにあるか・・・難しいです。

f(b)≧2bとなる0≦b≦1が存在するのは図を描いて予想しました。
それで平均値の定理を使えばいいと思うのです。
>>946
[0,b] だけじゃなく [b,1] でも考えれば良いのじゃない?
948943:04/08/11 06:40
>>947
場合わけってことですか?
なんか出来そうです、ありがとうございます。
949943:04/08/11 06:49
何度もすいません。
僕が間違えてなければ、b>1/2のときに[b,1]で平均値の定理をつかって、
f'(c)=(f(1)-f(b))/(1-b)>2を示すことは出来ないみたいです。
f(1)<2としか分かってないので、例えばf(1)=1の時にf'(c)=0となって
しまいます。
>>949
c=0はダメなの?
951950:04/08/11 07:03
意味不明なことを喚いてしまった;
>>943
g(x) = ∫[0,x]f(t)dt - x^2 とすると、
g'(x) = f(x) - 2x, g(0) = g(1) = 0。
平均値の定理より g'(a) = 0 となる a (0<a<1) が存在する。
f(a) = 2a なので、平均値の定理より
f'(b) = 2 となる b (0<b<a) が存在する。
953132人目の素数さん:04/08/11 07:42
質問!
ある平面にそれぞれ高さの違う塔が三本立っている。
ひとつの塔の頂上と他の塔の頂上を結ぶ直線と平面との交点(合計三つ)は
同一直線上にある事を証明せよ。
わからーん
954943:04/08/11 07:47
>>952
おー!
ありがとうございます!
f(x)の面積∫[0,x]f(t)dt と 2xの面積x^2を式で比較するのですか。
僕には到底思いつきそうもありませんでした。難しい。。。

ところで、
g(x) = ∫[0,x]f(t)dt - x^n
としたら、f'(b)=nとなりますか??
955943:04/08/11 07:50
>>954
>g(x) = ∫[0,x]f(t)dt - x^n
>としたら、f'(b)=nとなりますか??

間違えました。撤回します。

とにかくありがとうございました!
>>943
0≦x≦1においてf(x)≦2xでf(x)<2xとなるxが存在するなら
∫_[0,1]f(x)dx<1となるので
∫_[0,1]f(x)dx=1ならば2x<f(x)となるxが存在するか
f(x)=2xとなるから2≦M。

>>953
三つの交点は三本の塔の頂上を通る平面と元の平面の交線上にある。
957132人目の素数さん:04/08/11 08:02
>>956
おっとこまえーー!
958943:04/08/11 08:08
>>956
またまた良いことを聞いてしまいました!
そうか、背理法で出来ましたね!
ありがとうございます!!
方べきの定理の証明の仕方教えてください
>>959
具具れ
961132人目の素数さん:04/08/11 11:30
{(N-1)/N}のN乗のNを無限に近づけると なんで0に近づくのか教えてください。 (N-1)/NのNを無限に近づけると1に近づくのに…
962132人目の素数さん:04/08/11 11:36
>>961
0にはいかないだろう
963132人目の素数さん:04/08/11 11:38
>>961

前半は $1/e $ では?

$( \dfrac{n - 1}{n} )^n = ( \dfrac{n - 1}{(n - 1) + 1} )^n
= \dfrac{1}{(1 + 1/(n - 1))^{n - 1} \cdot (1 + 1/(n - 1)) }$

なので、$n \to \infty $ だから、$n - 1 \to \infty$ で、
(与式)$\to \dfrac{1}{e \cdot 1} = 1/e $
>>961
数式ぐらい正確に書けよ。
965132人目の素数さん:04/08/11 14:50
zが複素数、iが虚数単位で
zの絶対値が2のとき

z^2-2(1+2ai)z+4(1+ai)=0

を満たす実数aはどうやって求めるのでしょうか?
>>965
z=p+qiとでもおいて
代入して、実部=0と虚部=0と |z|=2を連立させて解く
>>966
出ました。感謝です。
@xが-2≦x≦1の範囲を動くとき
y=(x^2+2x+3)(x^2+x-2)-5x^2-10x+2
の最大値、最小値と、そのときのxの値を求めよ。

A次の不等式を解け。ただし、aは定数とする。
x^2-(a+a^2)x+a^3<0

B次の式を因数分解せよ
t^3-t^2+t/3-1/27

の3つの問題の解き方がわかりません。
どなたか教えてください。。。
sin(しーた+90)、cos(180-しーた)等の覚え方教えて。
970132人目の素数さん:04/08/11 15:48
>>969
普通に加法定理使えば。
971969:04/08/11 15:55
>>970
思い出したからいいや
とりあえずサンクス(・∀・)
>>968
@は展開
Aは判別式
Bは3乗の式のやつかな?
多分ね。
質問です・・
∫r*e^(-r^2)dr
=-1/2(e^(-r^2))

と友達のノートに書いてあったのですがこれって間違って。。ますよね?
部分積分しなきゃいけないような気がするのですが
何分自分の数学力に自身がないので指摘できません。

どなたかご教授お願いします
>>972
出た答え微分してみ?間違いじゃないよ。
あと、そういう場合は部分積分じゃなくて−r^2を置換汁!
おぉっ
即レス感謝です。

人にノート借りときながら疑ってしまった
自分が情けないですorz
あの程度の問題がわからない方が情けないと思われ.
>>974
不定積分を求める問題なら積分定数も忘れずに
977132人目の素数さん:04/08/11 17:02
>>933-944
ありがとうございます!ばっちりわかりました。

…のですが、0になる多項式って何ですか?
す、すみません…(´Д`;)
978大学への名無しさん:04/08/11 20:21
数学って解かなくても何度も読めば良いのでは?
http://school4.2ch.net/test/read.cgi/kouri/1092221749/l50
ご意見お待ちしております。
あくまでも大学受験レベルですが。
979132人目の素数さん:04/08/11 22:15
>>972
>∫r*e^(-r^2)dr
>=-1/2(e^(-r^2))
は、
 ∫r*e^(-r^2)dr
 =-(1/2)*(e^(-r^2))
だと思うけど。

まず、-1/2(e^(-r^2))+C (Cは積分定数)を微分すると、
{-(1/2)*(e^(-r^2))+C}'
= -(1/2)*(-2r)*(e^(-r^2))
= r*(e^(-r^2))
なので、>>972の積分は正しい。

次に、積分方法としては、-r^2 = t とおくと、
 -2rdr = dt ⇔ rdr = -(1/2)dt
より、
∫r*e^(-r^2)dr
=∫-(1/2)(e^t)dt
=-(1/2)(e^t)+C
=-1/2(e^(-r^2))+C(Cは積分定数)■
980数学界のつぶやきシロー:04/08/11 23:17
高校数学 数U 平面図形と式 二直線の交点の軌跡

問題:mが任意の実数の値をとって変化するとき、2直線x-my+1=0、(m+1)x-my+2=0の交点Pはどんな図形を描くか。

自分の答え:
x-my+1=0 より my=x+1 y≠0のとき m=x+1/y ・・・・@
(m+1)x-my+2=0 より (x-y)m=-x-2 x≠yのとき m=-x-2/x-y ・・・・A
@Aよりmを消去して x+1/y=-x-2/x-y これを解いて y=-x^2-x
ここでy≠0より (0,0) x≠yより (-2.-2) は除く。
よって答えは 放物線y=-x^2-x (0,0)(-2.-2)は除く。

でも解答見たら(-2,-2)は除かれていませんでした。
僕の答えの出し方のどこがどうして誤っているのかお教えください。。
>>979
ヴァカ? 清書屋はいらネェよ。

>>980
自分で意図的に x=y のときを除いてるからだろ。
>>980
君は二箇所、計四つの場合に分けていながら、そのうちの一つしか見てないから。
983132人目の素数さん:04/08/11 23:26
>>979
こういう人の意図がわからん
何したいんだろ
984132人目の素数さん:04/08/11 23:33
>>980
>問題:mが任意の実数の値をとって変化するとき、2直線x-my+1=0、(m+1)x-my+2=0の交点Pはどんな図形を描くか。

>自分の答え:
>x-my+1=0 より my=x+1 y≠0のとき m=x+1/y ・・・・@
>(m+1)x-my+2=0 より (x-y)m=-x-2 x≠yのとき m=-x-2/x-y ・・・・A
>@Aよりmを消去して x+1/y=-x-2/x-y これを解いて y=-x^2-x
>ここでy≠0より (0,0) x≠yより (-2.-2) は除く。
>よって答えは 放物線y=-x^2-x (0,0)(-2.-2)は除く。
出来れば括弧を省略しないで書いて欲しいけど、それは置いておいて。

まず、(0, 0)は通らないのはOK。これは直線の式に(0, 0)を代入すると分かる。
次に、x=y の場合が抜けている。
x=y のとき、x = -2, y = -2 で、これを直線の式に代入しても問題は起こらない。

「y≠0 のとき」「x≠yのとき」と書いたときは、「y=0のとき」と「x=yのとき」のそれぞれの場合も吟味しないと。
>>980
y≠0 かつ x≠y のとき
y=0 または x=y のとき
の場合分けのうち1つ目の場合の結論としてはそれで誤り無い。
986979:04/08/11 23:38
>>983
分からないと書いてあったから、それに対するレスをしたまでだけど。
強いて言えば、「よって、>>972は間違っていないので指摘のしようがない」と付けるのを忘れた。
わざわざ意図が分からないというレスをつける方が意図不明。
煽りにしては中途半端だし。質問に回答レスすること自体を否定するなら、そもそもこの板に来なければいいし。
>>986
何で>>974で既に即時解決しているものに、あとからアホみたいに偉そうな
レスしてんのかって訊いてんだけど?
988132人目の素数さん:04/08/11 23:42
>>986
清書屋は数学板には必要ありませんので消えてください。
989132人目の素数さん:04/08/11 23:44
清書屋、TeX房はコテになってやれ
990979:04/08/12 00:32
>>987
見落としていた。
人の間違いを必死に非難する暇があったら、まず自分の目にある梁を取り除くことをお勧めする。

>>988
自分の練習を兼ねてレスするのを否定される筋合いはない。
レスをつけて益になることはあっても害になることはない。
強いて言えば本人のためにならないということがあるが、それだったらそもそも2ちゃんねるで
質問すること自体が間違っている(∵答えが欲しくて質問する人が殆どだから)。中には面白半分に
嘘を教えたりする人もいるが、そちらの方がよほど悪質だと言える。

それに、答えを清書しているつもりは毛頭ない。考えの仮定を書き連ねたら、
たまたま清書のようになっただけのこと。
中学の頃から、宿題やテストでなくても出来るだけ途中経過を詳しく書くように習慣付けてきた。
それがたとえ自分で考えた問題だとしても。
逆に、(自分が分かる問題に対して)自然に証明が書けないような香具師に数学をやる資格は無い。

もし必要があれば、清書行為を禁止するようにローカルルールを定めれば良い。
そういうローカルルールが出来たら、清書行為を辞める。それまでは辞めるつもりはないし、
辞める義務も無い。好きな時に好きなだけ清書させてもらう。
991132人目の素数さん:04/08/12 00:36
文が長いし、あとからどうでもいいことだらだら書き連ねてるから、葵玲だな
>>990
清書ならその辺のチラシの裏に書け。オナニーは隠れてやれや。
993数学界のつぶやきシロー:04/08/12 00:52
>>981 >>982 >>984 >>985
ありがとう!ありがとうございます!
完璧に理解できました!スッキリしました!!
偏差値3はあがりました!ありがとうございました!!
次のスレ

【sin】高校生のための数学質問スレPart11【cos】
http://science3.2ch.net/test/read.cgi/math/1092239756/
>>990
公開オナニーは悪趣味です。
オナニーは自分一人で、自分のノートでやりましょう
996132人目の素数さん:04/08/12 00:58
>>990
清書しか能の無い馬鹿は回答者やらんでいいよ
997984:04/08/12 01:23
>>992,995,996
>>993で、感謝のレスがついている。言っておくが、自作自演ではない。
いざとなれば、運営側のログを参照すれば分かること。

公開オナニーと言われようが何と言われようが、誰かの役に立っている以上は決して無駄レスではない。
そうやって真面目にレスした人間を根拠も無く叩く方がよほど趣味の悪い公開オナニーだろ。
仮にも数学板の住人だったら、こちらが間違ったことをしたということを数学的に証明してもらおう。
それが出来ない上に根拠の無い叩きを続けるんだったら氏ね。
所詮はまともに答えを順序立てて説明することも出来ない香具師の僻みだろうが。
>>980>>993
>>997
>>993で、感謝のレスがついている。
問題視されている>>979とは何の関係も無いわけだが。ひょっとして何が問題に
されているのかすらわからないヴァカですか?
ジャマだからほかいってくれます?
10011001
このスレッドは1000を超えました。
もう書けないので、新しいスレッドを立ててくださいです。。。